You are on page 1of 91

West Visayas State University – College of Medicine – Batch 2020

Block XX
Module 6 Introduction to Anesthesia; History; Pre-operative Evaluation
Lecture 1
05/ 14/ 19
Dr. Marcos Bito-onon

TOPIC OUTLINE ─ wife of Hypnos


I. Definition of Anesthesia Morpheus
II. History of Anesthesia ─ god of Dreams
a. Patrons of Anesthesia
b. Men of Anesthesia
─ brother of Hypnos
III. Anesthesiology as a Specialty • Biblical Origin
IV. Preoperative Evaluation So the Lord God caused the man to fall into a deep
a. American Society of Anesthesiology (ASA) Practice sleep; and while he was sleeping, he took one of
Advisory
b. Objectives of pre-op evaluation the man’s ribs and then closed up the place with
c. Goals of pre-op evaluation flesh. (Genesis 2:21)
d. Pre-anesthetic evaluation This was the first form of operation
e. Pre-anesthetic laboratory exam
• Earliest use of anesthesia was 2750 BCE
V. ASA Classification of the patient
VI. Anesthesiologist-Patient Relationship
VII. Informed Consent TYPES OF ANESTHESIA
VIII. Pre-operative preparation Table 1. Types of anesthesia. Source: Adeos and doc’s ppt
IX. Risk for aspiration
X. Types of Anesthesia Drug Methods Non-drug methods
XI. Factors Associated with Increased Risk for Aspiration
XII. Risk Factors of Post-operative Pulmonary Complications Alcohol Cold
Review Questions
References Opium (Poppy) Concussion
Appendices
Hyosine (Mandrake) Carotid Compression
LECTURER BOOK REFERENCE OLD TRANS Cannabis (Hemp) Nerve Compression
Cocaine (New World) Hypnosis
Blood Letting
I. DEFINITION OF ANESTHESIA
• “Without sensation”
• Etymology: Cocaine - first local anesthetic used for spinal
“an” (greek)- without anesthesia of all the anesthetics, only cocaine
“aesthesis” (greek)- sensation causes vasoconstriction others cause vasodilatation
• Bailey’s English Dictionary (1751) from poppy plant
“defect of sensation” Procaine
• Oliver Wendell Holmes (1846) first synthetic local anesthetic
“sleep-like state that makes painless surgery Introduced in 1903
possible”
This was attributing to anesthesia given with ether B. MEN OF ANESTHESIA
• Joseph Priestly
II. HISTORY OF ANESTHESIA Discovered Nitrous Oxide (1772)
• “At this time, the patients are unconscious (‘anasthetoi’) NO2 aka as laughing gas
of everything, deaf to what is spoken, blind to what is Patient unable to feel pain; very useful in labor
happening and insensible to pain (‘analgetoi’) anesthesia; adjunct with anesthesia during surgery
In the 19th century NO2 was used for entertainment
A. THE PATRONS OF ANESTHESIA akin to the use of illicit drugs in the present era. It
• Greek: initiates the feeling of being “high” and people
Hypnos discovered that they did not feel pain when under the
─ god of sleep influence of NO2.
─ messenger with wings attached to his temples • Humphrey Davy
or shoulder First to note the analgesic effects of nitrous oxide
Thanatos (50 years after its discovery)
─ god of Non-violent death • Horace Wells
─ twin brother of Hypnos Dec 10, 1844: First to use nitrous oxide in humans
Pasithea (tragic figure);
─ goddess of Hallucination

CCetC Group No. 29 1 of 15


MD 3 Respicio, Rivera, Samoro
He got addicted to chloroform and committed anesthetic plan with the primary goal of reducing
suicide before he was able to make the use of NO2 morbidity and mortality during the peri-operative
public period.
• William T.G. Morton Done at least 24 hours before elective/ scheduled
First to use ether publicly in Massachusetts, USA surgeries. If emergency, do rapid assessment only
(Oct 16, 1846)
Ether: prolonged induction of anesthesia resulted
in delayed emergence but with high incidence of
nausea and vomiting
First successful demonstration of NO2
• James Young Simpson
First to administer obstetric anesthesia with the
use of ether and chloroform for pain relief (January
17, 1847)
• John Snow
Figure 1. Anesthesiologist’s role in Pre-op evaluation. Source: Adeos
“Father of Anesthesia”
He administered chloroform as a form of The management of anesthesia starts as soon as
anesthesia on the Queen of England during labour you finish your pre-operative visits and upon giving
Popularized the use of anesthesia especially in pre-operative medications.
obstetrics
• August Bier A. AMERICAN SOCIETY OF ANESTHESIOLOGY
First to do spinal anesthesia (1898) (ASA) PRACTICE ADVISORY
• Theodore Tuffier • An interview with the patient or guardian to establish a
Identified the landmark (iliac crest which medical, anesthesia and medication history
corresponds to L4-L5 interspace) for epidural • An appropriate physical examination
anesthesia; known as Tuffier’s line Do not forget to perform a neurologic exam
• John Lundy especially if it is spinal/ epidural anesthesia (regional
Introduced the concept of Balanced Anesthesia anesthesia)
which is the use of multiple drugs- sedative, • Indicated diagnostic testing
narcotic analgesic, muscle relaxant, inhalation Pertinent diagnostic exams only that can be of
anesthetic, obliterator of reflex response of px. benefit to the patient
• Dr Quintin J. Gomez Pre-op tests should not be ordered routinely unless
“Father of Anesthesia in the Philippines” indicated
• Review of diagnostic data (laboratory, ECG,
* I only included the people doc glossed over or radiographs, previous consultations)
discussed. Refer to the supplementary notes for the full • Assignment of an ASA physical status score (ASA-PS)
list written in the old trans. • A formulation and discussion of anesthesia plans with
the patient or a responsible adult before obtaining
III. ANESTHESIOLOGY AS SPECIALTY informed consent
“Anesthesiologist’s job is to focus on the whole During the perioperative visits, decide whether the
patient, while the surgeon is focus is on… getting a patient is a good candidate for surgery. The
good outcome. We’re supposed to be parked there anesthesiologist has the final say whether a patient
behind our banks of monitors and managing the is fit or not for surgery. The surgeon just operates
patient’s overall physiology,…. Etc. (look at video)” but the perioperative course of the patient is on the
ANESTHETICS: The use of drugs which will render hands of the anesthesiologist. You can also
the patient unconscious, control pain, induce determine if the patient is a candidate for urgent,
amnesia (put to sleep, absence of pain and patient emergent or elective surgery.
will forget what transpired during the procedure) and Urgent if the patient has to be operated as soon as
prevent movement (4 functions of an ideal possible. Emergent if the patient can be operated on
anesthesia) within 24 hours. Elective if the surgery can wait

IV. PRE-OPERATIVE EVALUATION B. OBJECTIVES OF PRE-OPERATIVE EVALUATION


• Mandatory • To review database (know the history)
• The pre-op evaluation consists of gathering medical • To perform a physical exam
information from the patient and formulating an • To establish a doctor-patient relationship

CCetC
Block XX: Introduction to Anesthesia; History; Pre-operative Evaluation 2 of 15
MD 3
• To obtain consent (before the conduct of anesthesia) 3 aspects of physical examination
• To make an anesthesia plan (you cannot do anesthetic • Airway
procedure without the anesthesia plan) • Cardiovascular
REMEMBER: The approach to the patient should • Pulmonary
always begin with a thorough history and physical • (Neurologic)
examination. These two evaluations alone may be for patients undergoing spinal anesthesia or for those
sufficient (without additional routine laboratory tests). undergoing a surgery with a non-conventional
position
C. GOALS OF PRE-OPERATIVE EVALUATION example would be in kidney surgeries where the
• Reduce patient risk and the morbidity of surgery patient is extended torso down and also lower
• Promote efficiency and reduce costs extremities down
• Increasing resource utilization within the operating
room (OR) PRE-ANESTHETIC EVALUATION
The primary goal of pre-op evaluation should be to • Y- whY is the patient for surgery
minimize morbidity and mortality. Anesthesia is a • A- Anesthetic history
form of controlled poisoning. The medication that we • M- Medications including allergies
you should give need to be precise, within the • P- Past medical history/ Review of systems
recommended dose and within the bounds and limits • L- Last meal
of surgery and within your knowledge. • E- Examination (PE)- airway, cardio, respiratory
• Must also include:
D. PRE-ANESTHETIC EVALUATION Indications for surgical procedure
THE RULE OF THREES ─ Determines the urgency of the surgery
• Three aspects of acute history Response to previous anesthesia
• Three aspects of chronic history Medications/ allergies including herbal medications
• Three aspects of physical exam ─ Garlic, Gingko, Ginger, Ginseng

3 aspects of acute history Physical Exam


• HPI • Vital signs
• Exercise tolerance Height and weight
• Patient’s visit to primary physician ─ Dosage of anesthetic drugs are weight based;
BMI
3 aspects of chronic history Blood pressure measurement
• Medications Resting pulse
Including herbal medications Respiration
Herbal medications may cause bleeding episodes, Pain scoring
drug to drug interactions with the anesthetic agents • Airway
(intake for the last 2 wks or 2 months) • Head and Neck
Discontinue the use of herbal medications for 10-14 Mouth and Oral cavity
(2 weeks) days before an elective surgery ─ Extent of symmetry of mouth opening
• Social History ─ Health of the teeth
Drug abuse, alcoholism, smoking, etc. Loose tooth/teeth that predisposes the
• Past Medical and Family History patient to aspiration or airway obstruction,
Family history of allergies to anesthetic might break dentures
medications; malignant hyperthermia ─ Presence of dental appliances
Medical History including: ─ Size of the tongue
─ Allergies (recently patient has many reactions to Larger in pediatric patients than in adults
medications) ─ Palate
─ Drug intolerance Size of the mandible
─ Present therapy Temporomandibular joint (TMJ) function
─ Non-therapeutics (alcohol and smoking) Thyromental distance
─ Illicit drug use Prior surgeries, neck masses or tracheal deviations
Also remember to elicit a history of difficult airway Range of motion of the head and neck
• Cardiovascular system
• Pulmonary system
• Back (check the spine) (spinal, epidural anesthesia)

CCetC
Block XX: Introduction to Anesthesia; History; Pre-operative Evaluation 3 of 15
MD 3
• Neurologic examination • Obstructive sleep apnea assessment:

Metabolic Equivalents of Functional Capacity Table 3. Stop Bang Questionnaire. Source: Adeos
• 1 MET Loud enough to be heard
The amount of oxygen that is consumed while behind closed doors
Snoring
sitting at rest Associated with a high
𝑚𝑚𝑚𝑚 O2 Mallampati score
3.5 𝑥𝑥 𝑛𝑛𝑛𝑛𝑛𝑛𝑛𝑛𝑛𝑛𝑛𝑛 𝑜𝑜𝑜𝑜 𝑚𝑚𝑚𝑚𝑚𝑚𝑚𝑚𝑚𝑚𝑚𝑚𝑠𝑠
𝑘𝑘𝑘𝑘 (𝑏𝑏𝑏𝑏𝑏𝑏𝑏𝑏 𝑤𝑤𝑤𝑤𝑤𝑤𝑤𝑤ℎ𝑡𝑡) Daytime fatingue while
Simple, practical and easy to use procedure for Tired
talking and driving
expressing the energy cost of physical activities as a Has anyone seen you
multiple of the resting metabolic rate Observed stopped breathing or gasp
Convenient method that describes the functional during sleep?
capacity or exercise tolerance of an individual Pressure Do you have hypertension?
Important in assessing cardiovascular status of the BMI >35
patient Age >50
Collar size > men, 16 for
Table 2. MET Score definition. Source: Doc’s ppt Neck Size
women
Functional capacity/ exercise Gender Male
MET
tolerance
Daily self-care • Mallampati Classification
Eat, Dress or use toilet Let the patient open his mouth and protrude the
Walk indoors, around the house tongue without saying a word and examine
1 MET Walk a block or two on level ground at 2-
3 mph or 3.2-4.8 km/h Table 4. Mallampati Classification. Source: Adeos and Doc’s
Do light work around the house, like ppt
dusting or washing dishes Direct
Class Image
Climb a flight of stairs or walk up a hill Visualization
Walk on level ground at 4mph or 6 km/h
Run a short distance Soft palate,
Do heavy work around the house, like I fauces, uvula,
scrubbing floors or lifting or moving heavy pillars
4 METs
furniture
Participate in moderate recreational
activities like gold, bowling, dancing,
double tennis or throwing a baseball or Soft palate,
II
football fauces, uvula
Participate in strenuous sports like
>10 METS swimming, single tennis, football,
basketball or skiing
Soft palate,
III
Airway Examination uvular base
RULE OF 12345
1 finger breadth for subluxation of the mandible
2 finger breadths for adequacy of mouth opening
3 finger breadths for hyomental distance Hard palate
4 finger breadth for thyromental distance IV
only
5 movements: ability to flex the neck upto the
manibrium sterni, extension at the atlanto-occipital
joint, rotation of the head along the right and left
movement of the head to touch the shoulder If you have a patient with a Mallampati score of class
Rule of 3’s III or IV, prepare for a difficult airway. You may need
3 fingers in the interdental (inter-incisor) space necessary equipment and a skilled assistant during
3 fingers between the mentum and hyoid bone intubation.
3 fingers between the thyroid cartilage and • Findings that indicate difficult examination/ intubation
sternum Mallampati classification of ˃2
• Check for snoring, it is a symptom of airway obstruction Thyromental space of  3 fingerbreadths

CCetC
Block XX: Introduction to Anesthesia; History; Pre-operative Evaluation 4 of 15
MD 3
Diminished neck extension All patients over 60 years of age
Serum Glucose
Large tongue Diabetic patients
and Creatinine
Overbite Other specific clinical indications
Narrow-high arched palate All patients over 40 y.o. (but now it
Decreased TMJ mobility Electrocardiogram has been lowered to 35 y.o.)
Short thick neck (ECG) All patients with specific indications
Obese patient (i.e. HTN, palpitations, previous MI)
All patients over 60 y.o
Pulmonary System Specific clinical indications (i.e.
Chest Radiograph
• Inspections HTN, malignancy, acute pulmonary
symptoms)
Symmetry, deformities
Chest retractions
V. ASA- PHYSICAL STATUS CLASSIFICATION
• Palpation
*doc said this is important
Chest expansion
Tactile fremitus
Table 6. ASA-PS Classification. Source: Adeos and Doc’s ppt
• Percussion Class Definition Comments and Examples
• Auscultation Normal Healthy
Breath sounds Patient Without with good exercise
Adventitious sounds organic, tolerance
It is imperative that you auscultate and identify breath I
biochemical, or Exclude the extremes of
sounds first before intubation psychiatric age
disease
Cardiovascular System No functional limitations;
• Inspection has a well-controlled
Precordium disease of one body
• Palpation system; controlled
Point of maximal impulse A patient with
hypertension or diabetes
mild systemic
Thrills without systemic effects,
II disease and no
• Percussion cigarette smoking without
functional
Approximate the size of the heart chronic obstructive
limitations
• Auscultation pulmonary disease (COPD);
Heart sounds mild obesity
Murmur Extremes in age belong
to this class
E. PRE-ANESTHETIC LABORATORY EXAMINATION Some functional limitation;
• Routine Preoperative laboratory testing provides little has a controlled disease of
information beyond the results of history and physical more than one body system
examination to alter the management of otherwise or one major system; no
healthy patients A patient with immediate danger of death;
• ASA Recommendations moderate to controlled congestive heart
III severe systemic failure (CHF), stable angina,
Pre-operative tests should not be ordered routinely
disease that old heart attack, poorly
and should only be ordered if with indications
limits activity controlled hypertension,
present
morbid obesity, chronic
Pre-operative tests may be ordered, required, or
renal failure; bronchospastic
performed on a selective basis for purposes of disease with intermittent
guiding or optimizing perioperative management symptoms; cancer
Has at least one severe
Table 5. Sample labs and indications for testing. Source: A patient with disease that is poorly
Adeos and Doc’s ppt
severe systemic controlled or at end stage;
Test Indications for testing activity that is a possible risk of death;
IV
constant threat unstable angina,
All menstruating women to life or requires symptomatic COPD,
All Patients over 60 years of age intensive therapy symptomatic CHF,
Hemoglobin or
All patients likely to experience hepatorenal failure
hematocrit
significant blood loss and may
require transfusion

CCetC
Block XX: Introduction to Anesthesia; History; Pre-operative Evaluation 5 of 15
MD 3
Not expected to survive > • At a minimum, IC involved the indications for the
24 hours without surgery; treatment in terms a lay person can understand, and
A moribund
imminent risk of death; elucidation of alternatives.
patient who is not
multiorgan failure, sepsis • Discuss YAMPLE in the informed consent
V expected to
syndrome with
survive without
hemodynamic instability, VIII. PRE-OPERATIVE PREPARATIONS
the operation
hypothermia, poorly • Smoking cessation
controlled coagulopathy Increased risk for pulmonary and cardiac
A brain-dead complications as well as impaired wound healing
VI patient for organ The longer they are tobacco-free before surgery
donation the better, as their bodies will have more time for
REMEMBER: *The addition repair
of “E” denotes Emergency
Even only 12 hours of smoking cessation will
surgery: (An emergency is
Added for reduce levels of nicotine and carbon monoxide,
defined as existing when
E emergency improving blood flow
delay in treatment of the
operations
patient would lead to a
significant increase in the
IX RISKS ASSOCIATED WITH ASPIRATION
• Factors associated with increased risk for aspiration:
threat to life or body part)
Recent food intake
• More example scenarios are found in the
Elderly patient
supplementary notes
Decreased consciousness
Increased gastric pressure
Table 6. ASA-PS Classification Mortality Rate. Source: Adeos
and Doc’s ppt Increased acid production
Class Mortality Rate
Gastric and intestinal hypomotility
Impaired esophageal sphincter control
I 0.06-0.08%
Neuromuscular incoordination
II 0.27-0.4%
Presence of NGT
III 1.8-4.3%
Pregnancy
IV 7.8-23%
2 factors that predisposes pregnancy to aspiration:
V 9.4-51%
(1) Anatomic : pushes the stomach, instead of a “J”
configuration the stomach is flattened because of the
VI. ANESTHESIA-PATIENT RELATIONSHIP
• Organized interview gravid uterus (2) Endocrine: effect of progesterone,
• Reassuring the patient which is a relaxant, and relaxes the sphincters
• Events of the perioperative period:
NPO status F. 10 LEADING FACTORS PREDISPOSING TO
ASPIRATION
Estimated time of surgery
• Emergency Surgery
Need for premedication
• Inadequate anesthesia
Post-operative recovery
• Abdominal pathology
Plans for postoperative pain control
• Obesity
One of the most dreaded experiences is the pain
• Opioid medications
after surgery
Slows down motility of GI tract
• Neurologic deficit
VII. INFORMED CONSENT
• Lithotomy position
• Anesthetic plan
Pushes intra-abdominal contents cephalad
NPO status
• Difficult intubation/airway
Anesthetic Techniques
You have attempted intubation several times,
Pre-medications
during the intervals you ventilate patient and
Postoperative Recovery
introduce positive pressure resulting to some air
Postoperative pain control
going to the stomach and it refluxing
• Alternatives
• Reflux
• Potential complications
• Hiatal hernia
• Patient or Guardian needs to sign the consent
• Must be obtained for all non-emergency procedures
and is a legal requirement

CCetC
Block XX: Introduction to Anesthesia; History; Pre-operative Evaluation 6 of 15
MD 3
G. PREVENTION OF PERIOPERATIVE PULMONARY • Potential Procedure Related Risk Factors:
ASPIRATION Aortic aneurysm repair
• Patients who had a 25mL residual gastric volume with Thoracic surgery
a pH lower than 2.5 were at risk Abdominal surgery
Newer recommendations, they recommend up to 200 Upper abdominal surgery
mL intake 2 hours prior to surgery but just plain Neurosurgery
water. Prolonged surgery
Head and neck surgery
H. FASTING RECOMMENDATIONS TO REDUCE Emergency surgery
RISK OF ASPIRATION
Vascular surgery
Table 7. Fasting Recommendations. Source: Adeos, Doc’s ppt
General anesthesia
Minimum Fasting
Ingested Material Perioperative transfusion
Period (All ages)
Hip surgery
Clear liquids (water, pulp-free
Gynecologic or urologic surgery
juices, carbonated beverages,
2 hours Esophageal surgery
clear tea, black coffee) not more
• Pharmacologic Agents to Reduce Risk of Pulmonary
than 150 mL
Aspiration:
Breast milk 4 hours
Histamine-2 (H-2) Receptor Antagonists
Infant formula 6 hours Proton Pump Inhibitors
Non-human milk 6 hours Antacids
Light meal (toast and clear 6 hours Gastrokinetic Agents: Metoclopramide
liquids; meals that include fried or May be extended
fatty foods or meat may prolong to 7-8 hours in XI ANESTHESIA TECHNIQUES
the gastric emptying time) diabetic patients I. GENERAL ANESTHESIA
• Guidelines only apply to patients who are not at risk for • Broadly defined as a drug induced reversible
delay gastric arthropathy  these px should be on depression of the central nervous system (CNS)
longer NPO resulting in the loss of response to and perception of all
external stimuli.
PATIENTS AT RISK FOR DELAYING GASTRIC
EMPTYING INHALATIONAL
• Morbid obesity • most common general anesthesia, not necessarily
• Diabetes mellitus intubating the patient, we can also use mask
• Pregnancy • LARYNGEAL MASS AIRWAY: inserted below the
• History of gastroesophageal reflux tongue of the patient
• Surgery-limited stomach capacity
• Potential difficult airway TIV A
• Opiate analgesic therapy • giving anesthetic agents solely by IV route with the
absence of all inhalational agents
X RISK FACTORS FOR POSTOPERATIVE
PULMONARY COMPLICATIONS IV
• Type and site of surgery are the strongest predictors of • provides continuous infusion of sedative (i.e.
pulmonary morbidity in patients Midazolam)

Table 8. Risk Factors for Pulmonary Complications. Source: J. REGIONAL ANESTHESIA


Adeos and Doc’s ppt Test level through sensorial mapping
Potential Patient-Related Abnormal Findings on • Epidural Anesthesia
Risk Factors Chest Examination EPIDURAL ANESTHESIA: achieved by injection of
10 Diabetes local anesthetic solution into the space that lies
ASA Class II Obesity within the vertebral canal but outside or superficial to
CHF Asthma the dural sac (epidural space-above dura)
Functionally dependent Obstructive sleep apnea • Subarachnoid Block
COPD Corticosteroid used • Caudal Anesthesia
Weight loss HIV infection Variation of epidural anesthesia
Impaired sensorium Arrhythmia
Cigarette and alcohol use Poor exercise capacity

CCetC
Block XX: Introduction to Anesthesia; History; Pre-operative Evaluation 7 of 15
MD 3
K. LOCAL ANESTHESIA/ FIELD BLOCK XIII. PHARMACOLOGIC PREMEDICATION
In minor surgical procedures like circumcision, the M. PRE-OPERATIVE MEDICATIONS
dose of the anesthetic is determined. Legal requirement
loss of sensory perception on a specific area of the • Components:
body with the help of a hollow needle and syringe Psychological Preparation
The momentum for this development was the ─ This includes the pre-operative visit to the
discovery of several biologically active alkaloids, patient discussing the anesthesia technique
such as morphine, strychnine, atropine, and brucine, and what will transpire during the perioperative
which were relatively inactive when administered period. The visit must be conducted efficiently
orally but produced dramatic effects when deposited and must also be informative and reassuring.
into an open wound. Pre-op visit must be done at least 24 hours before
ELECTIVE SURGERY; if emergency just do rapid
L. MONITORED ANESTHESIA CARE (MAC) assessment just before you give anesthesia
• a planned procedure during which the patient ─ Patient’s mental and physical condition should
undergoes local anesthesia together with sedation and be assessed
analgesia ─ Satisfactory preparation lessens the patient’s
• conscious sedation + observation & management of (and family’s) anxiety and smoothens the
complication (i.e. midazolam/diazepam to calm patient) anesthetic induction
─ Psychological preparation alone may not relieve
you are just there to get vitals para lang may upod all anxiety
ang patient incase anything happens ─ After the patient interview, the use of pre-
operative medication in selected patients
XII CONSIDERATIONS THAT INFLUENCE THE serves to achieve sedation or amnesia as well
CHOICE OF ANESTHESIA TECHNIQUES as provide any needed analgesia
• Coexisting diseases Pharmacological Preparation
Patient may have tolerance If px is GCS 4-5 do not give pre-medication anymore
• Site of the surgery Ideal time to perform pre-operative preparations (pre-
• Position of the patient during surgery operative visit) is 24 hours to 1 hour prior to surgery.
• Risk of aspiration If 1 hour prior, usually for emergency cases.
• Age of the patient • Patient’s psychological condition, physical status, age
• Patient cooperation and prior response to depressant drugs must be
Write down on chart with patient signature if patient considered
prefers a different technique than recommended • The surgical procedure, expected duration, and
• Anticipated ease of airway management postoperative discharge plan are important factors as
• Coagulation status well
if the patient is taking blood thinners, it should be
discontinued N. PRIMARY GOALS OF PHARMACOLOGIC
─ Aspirin – stopped at least 7 days prior to PREMEDICATION
surgery • Relief of anxiety
─ Anticoagulant – LMWH- stopped for at least 24 • Sedation
-48 hours prior to surgery to lower perioperative meds and during induction
─ Protime >70 for regional anesthesia of anesthesia
• Previous response to anesthesia • Amnesia
history of prior surgery • Analgesia
patient might have a relative tolerance to anesthesia • Prevention of airway secretion
• Preference of the patient- follow wish of the patient • Prevention of autonomic reflex responses
REMEMBER: The choice of anesthesia (general, hypertension, tachycardia
regional or sedation), monitors, or specific anesthetic • Reduction of gastric fluid volume
drugs rarely alters outcome or risk. However, • Increase in gastric fluid pH
impressions from clinical experience continue to • Reduction of anesthetic requirements
influence beliefs and recommendations when • Prophylaxis against allergic reaction
devising a plan of anesthesia care.
O. SECONDARY GOALS OF PHARMACOLOGIC
PREMEDICATION
• Decrease vagal activity
Bradycardia especially in pediatric patients

CCetC
Block XX: Introduction to Anesthesia; History; Pre-operative Evaluation 8 of 15
MD 3
• Facilitation of smooth induction of anesthesia Sedative-hypnotic action - due to an inhibition of
If you are given pre-medication you need less dose conduction in the reticular formation resulting in a
of inducive drugs during anesthesia decrease in the number of impulses reaching the
• Post-operative analgesia cerebral cortex
• Prevention of post-operative nausea and vomiting capable of producing all degrees of depression from
mild sedation and hypnosis to general anasthesia,
P. DRUGS FOR PREMEDICATION deep coma and death
• Benzodiazepines GABA receptor (brain) - sedation
Diazepams, bedazolam etc. Glycine receptors (spinal cord) - muscle relaxation;
Most commonly given given for spasms and tetanic contractions
• Opioids Ketamine- only sedative drug that does not act on
tramadol, etc GABA receptor. It acts on NMDA receptors
• Antihistamines - anxiolytics
• Anticholinergics OPIOIDS
• Histamine receptor antagonists (H2 antagonists) • 3 Classes:
• Antacids Pure agonist
• Proton pump inhibitors Partial Antagonist
• Antiemetics Agonist – antagonist
• Gastrokinetic agents • Common side effects: nausea and vomiting, due to
• A2-adrenergic agonists effects on both the chemoreceptor trigger zone and the
vestibular system
BENZODIAZEPINES • Does not produce sedation or amnesia and are often
• Anxiolysis, amnesia and sedation combined with benzodiazepines for these effects
• Dose-dependent effect (depends on the percentage of
receptors occupied)
20% - anxiolytics
30-50% - sedation
>50%- unconsciousness
• Act on GABAA-receptor complex, causing membrane
hyperpolarization, increasing the frequency of channel
opening
• Relatively little depression of the ventilatory or
cardiovascular system with premedicant doses Figure 3. Opioid Classes. Source: Adeos
• Blocks common side effects of opioids: nausea and • Morphine
vomiting onset (IV): around 20 mins
• Examples: Analgesic and respiratory depressant effect
Midazolam (Dormicum) • Meperidine (Demerol)
─ 2-3x more potent than Diazepam, quicker onset 10x more potent than morphine
of action; Analgesic and respiratory depressant effect
─ water based, no irritation or phlebitis with • Fentanyl
injection; 100x more potent than morphine in analgesia
─ onset: 1-2 minutes Rapid onset and short duration
─ mental function usually returns to normal within In doses of 1-2 ug/kg IV it may be used as pre-
4 hours of administration and amnesia may only operative analgesia
last 20-30 minutes; Causes neither myocardial depression nor
─ tell patient not to drive histamine release
Diazepam (Valium) Associated with ventilatory depression and
Lorazepam (Ativan) profound bradycardia
─ 5-10x more potent than Diazepam but has slow • Tramadol (Tramal)
onset of action and longer duration 30% (1/3) of the potency of morphine
─ Profound amnesia and sedation • Nalbuphine (Nubain)
has a ceiling effect, if reached, no matter how
BARBITURATES (DOC GLOSSED OVER THIS) much the dose is given, it will not have an
Not often used because not widely available analgesic effect but side effects ensue;
Act on GABA receptors multiple sites agonist-antagonist;

CCetC
Block XX: Introduction to Anesthesia; History; Pre-operative Evaluation 9 of 15
MD 3
has the property of opioids at low doses but if GASTROKINETIC AGENTS
given at high doses it blocks opioid effects • Reduce gastric fluid volume
(reverses effects of other opioids) • Metoclopramide, a dopamine antagonist
given 5-10mg IV
NON-OPIOID ANESTHETIC Metoclopramide and hyoscine
Ketamine ─ given before opioids because opiods decreases
• Dissociative anesthesia gastric motility thereby decreasing the effectivity
rendered through the action of ketamine in the of gastrokinetic agents
limbic system; the patient appears awake with • Effects:
intact reflexes but is actually asleep/amnesic Stimulates upper GI motility
• Antagonist of the NMDA receptor Increases GE sphincter tone
• Profound analgesia and amnesia Relaxes pylorus and duodenum
• Bronchodilatory activity Antiemetic properties
preferred over barbiturates in patients who have • Given slowly via IV not as a bolus because it can
bronchial asthma INDUCE PARKINSON-LIKE REACTIONS in the
• Cardiovascular stimulating effects- caution for patient
hypertensive patients • Antagonized by opioids (slows gastric emptying time;
• Could be given IV especially for patients having opioids should be given first before giving
neuropathic pain metoclopramide
Can also be given as a sole anesthetic
ANTIEMETICS
OTHER SEDATIVE DRUGS • Reduce or abolish incidence of post-op nausea and
Hydroxyzine (iterax) vomiting
• Antihistamine and antiemetic • Ex: Metoclopramide, Ondansetron
• Sedative and anxiolytic properties • Risk Factors for Post-op N and V
• Given for its proposed additive effects to opioids Female gender
History of motion sickness/post-op nausea
Diphenhydramine Non-smoking- smoking is protective
Histamine receptor antagonist Use of postoperative opioids
Sedative and anticholinergic activity
Antiemetic properties ANTICHOLINERGICS
patient becomes tachycardic • Elevation of Gastric Fluid pH Level
• Cannot be relied to decrease H+ secretion
HISTAMINE 2 (H-2) RECEPTOR ANTAGONISTS replaced by H2 receptor antagonists
• Reduce gastric acid secretion • Induction before surgery:
• Increase gastric pH Antisialogogue
• Selective and competitive antagonism ─ May result to drying of airways
• Examples: Cimetidine, Ranitidine, Famotidine ─ Intraoral operations and instrumentations of the
airway
ANTACIDS Sedation and amnesia
• Neutralize the acid in gastric contents Vagolytic actions
• Increase gastric fluid pH above 2.5 ─ Blocking of ACTH at SA node
• No lag time Less likelihood of tachycardia
• Non-particulate antacid is commonly given before They used to give this in the past but with the advent
operation of the newer inhalation anesthetics
• SIDE EFFECTS:
PROTON PUMP INHIBITORS (PPI) Central Nervous System Toxicity
• Suppresses gastric secretion in a dose-dependent ─ Central anticholinergic syndrome: delirium,
manner by binding to the proton pump of the parietal restlessness, confusion, and obtundation
cell ─ Especially scopolamine and atropine
• Effect is dose-dependent ─ Administration of 1-2 mg of physostigmine IV
• Effect lasts as long as 24hours can successfully treat the syndrome
• Oral doses of 40mg to 80 mg must be given 2 to 4 Increased intraocular pressure
hours before surgery to be effective. ─ Mydriasis and cycloplegia and may place
• Examples: Omeprazole, Pantoprazole, Lansoprazole patients with glaucoma at risk for inc IOP

CCetC
Block XX: Introduction to Anesthesia; History; Pre-operative Evaluation 10 of 15
MD 3
─ Atropine and glycopyrrolate may be less likely when he’s asleep. Short-acting to have fast onset
to increase IOP than scopolamine and fast off-set
Hyperthermia Don’t give oral medication if outpatient
─ Sweat glands in the body use cholinergic
mechanisms; sympathetic nervous system S. MEDICATIONS
─ May also trigger tachycardia • Instructions to patients to continue or discontinue drugs
Be cautious with administering atropine to pxs with will likely improve outcomes
fever • Co-morbidities and the nature of the procedure are
considered when managing medications pre-
Α-2 ADRENERGIC RECEPTOR AGONISTS operatively
• Reduce tonic levels of sympathetic outflow Pre-rog of anesthesiologist
• Augment cardiac vagal activity
• Produce induced hypotension Table 9. Anesthesia Medication Instruction. Source: Doc’s ppt
Clonidine and Dexmedetomidine can cause sedation Discontinue on the Day of
Continue on Day of
at very high doses, however the point in which they Surgery Unless Otherwise
Surgery
can cause sedation is more than the point where it Indicated
produces hypotension. 1Antidepressant,
• Clonidine (Catapres) antianxiety, and
psychiatric medications
Sedation at a higher dose
(including monoamine
prevent hypertension and tachycardia
oxidase inhibitors)
Centrally acting
Antihypertensives
• Dexmedetomidin (Precedex)
potent sedative • Consider discontinuing ACE
inhibitors or ARB 12- 24 hrs
analgesic-sparing properties
before surgery if:
taken only for
Q. ROUTES OF ADMINISTRATION
hypertension
• Oral- preferred
lengthy procedures
• Intramuscular- in patients with no IV access; in case of
significant blood loss or
benzodiazepine, it would have an erratic absorption; IM
fluid shifts
diazepam would be painful because it is oil based • Generally to be
use of general
compared to midazolam which is water based continued
anesthesia
• Intravenous/Intravascularly multiple
• REMEMBER: Ideal time to give pre-op medication: At antihypertensive
least 1 hour before prior to induction of anesthesia medications
(oral); just before the patient is wheeled into the OR well-controlled blood
(IV) pressure
• REMEMBER: Diazepam- 2 hours before OR hypotension is
particularly dangerous
R. FACTORS/DETERMINANTS OF DRUG CHOICE Aspirin
AND DOSE IN PREMEDICATION
• Discontinue 5-7 days before
• Patient’s age and weight • Patients with known
surgery:
all patients must be weighed as much as possible, vascular disease
If risk of bleeding > risk
give meds depending on the weight of the patient • Patients with drug-
of thrombosis
• Physical status eluting stents for <12
For surgeries with
Use the ASA-PS months
serious consequences
• Level of anxiety • Bare metal stents for < 1
from bleeding
• Tolerance of depressant drugs month
Taken only for primary
• Before cataract surgery
especially patients with history of illicit drug use prophylaxis (no known
(if no bulbar block)
• Allergies vascular disease)
• Before vascular surgery
• Previous adverse experience with drugs used for pre- If surgery is emergency
• Taken for secondary
operative medications administer Fresh Frozen
prophylaxis
• Elective or emergency surgery plasma
• Inpatient or outpatient surgery Asthma medications
For outpatient surgery, give meds IV for faster Autoimmune medications
elimination of drugs, you cannot send the px home

CCetC
Block XX: Introduction to Anesthesia; History; Pre-operative Evaluation 11 of 15
MD 3
• Methotrexate (if with risk of Viagra or similar
renal failure) Steroids (oral or inhaled) medications (discontinue 24
• Methotrexate (if no risk
• Entanercept (enbrel), hours before surgery)
of renal failure)
infliximab, adalimumab : Thyroid medications Vitamins, minerals, iron
check with prescriber
Warfarin
Birth Control Pills
• Cataract surgery, no • Discontinue 5 days before
Cardiac Medications bulbar block surgery
• Clopidogrel (Plavix)
With drug-eluting XIV. RISK ASSESSMENT
stents for < 12 • Useful to compare outcomes, control costs, allocate
• Clopidogrel (Plavix)
months compensation, and assist in the difficult decision of
Patients not included in
Bare metal stents canceling or recommending a procedure not be done
group recommended for
for < 1 month when the risks are too high
continuation
Before cataract • Risks have traditionally been attributed to the patient’s
surgery (if no bulbar co-morbid conditions, general health status, age,
block) anesthetic technique, and the planned procedure
• COX-2 inhibitors It is according to the anesthesiologist whether the
• COX-2 inhibitors If surgeon is concerned
surgery will be continued or delayed. After all he is
about bone healing
the one that will be monitoring the patient
Diuretics
• Triamterene T. COMMONLY DISCLOSED RISKS OF
• Potent loop diuretics
• hydrochlorothiazide ANESTHESIA (WITH GENERAL ANESTHESIA)
Eye Drops FREQUENTLY OCCURING, MINIMAL IMPACT
Estrogen Compounds • Oral or dental damage
• Sore throat or feeling of lump in the throat
• When used for birth • When used to control
may still be present 2-3 days after surgery
control or cancer menopause symptoms or for
therapy osteoporosis • Hoarseness
Gastrointestinal Reflux Gastrointestinal Reflux • Postoperative nausea and vomiting especially in
medications medications (Tums) female patients
• Herbal and nonvitamin • Drowsiness/confusion
supplements • Urinary retention
7-4 days before surgery Especially in pts given opioids given
• Hypoglycemic agents (oral) perioperatively
Insulin
INFREQUENTLY OCCURING, SEVERE IMPACT
• Type I DM: take 1/3 of
• Awareness
intermediate to long
Benzodiazepines are useful as pre-medicant to
acting
induce amnesia
• Type II DM: take upto ½
• Visual loss
long acting or
combination (70/30) May be due to dehydration
• Regular insulin • Aspiration
preparations
• Discontinue of blood sugar especially if px was not placed on NPO
• Glargine (lantus):
level <100 • Organ failure
decrease if dose is >/=
1 unit/kg if you were not able to monitor patient well and
• With insulin pump hydrate the px
delivery, continue • Malignant hyperthermia
lowest night-time basal • Drug reactions
rate • Failure to wake up or recover after general anesthesia
Narcotics for pain or NSAIDs (48hr before • Death
addiction surgery)
Seizure medications U. COMMONLY DISCLOSED RISKS OF
Topical creams and ANESTHESIA (WITH REGIONAL ANESTHESIA)
Statins FREQUENTLY OCCURING, MINIMAL IMPACT
ointments
• Prolonged numbness/weakness
You may have given vasoconstrictors such as
epinephrine that prolongs excretion
CCetC
Block XX: Introduction to Anesthesia; History; Pre-operative Evaluation 12 of 15
MD 3
• Post-Dural Puncture Headache Patient unable to feel pain; very useful in
Presence of a puncture in the dura leading to labor anesthesia; adjunct with anesthesia
continuous leakage causing decrease in CSF, thus during surgery
decreased ICP
Manage: Humphrey First to note the analgesic effects of
─ Hydrate the patient Davy nitrous oxide (50 years after its
─ Maintain supine position discovery)
─ Apply pressure Gardner Medical student who first used nitrous
─ Give analgesic (NSAIDS) Colton oxide
─ Drink coffee or brandy Horace Wells Dec 10, 1844: First to use nitrous oxide
in humans (tragic figure);
─ Theophylline or aminophylline
He got addicted to chloroform and
─ Apply pressure or binder
committed suicide
─ Cover with patch (Epidural blood patch)
William T.G. First to use ether publicly in
Extract blood from the patient and introduce
Morton Massachusetts, USA (Oct 16, 1846)
it to the epidural space with an epidural
Ether: prolonged induction of anesthesia
needle to induce blood clotting and cover resulted in delayed emergence but with
the tear high incidence of nausea and vomiting
• Failure of technique First successful demonstration
James Young Professor of midwifery in Edinburgh from
INFREQUENTLY OCCURING, SEVERE IMPACT Simpson 1840
• Bleeding (1811-1870) Tried chloroform on himself and friends
• Infection from suggestion of David Waldie, a
• Nerve damage/paralysis chemist
• Persistent numbness/weakness January 17, 1847: First to administer
• Seizure obstetric anesthesia on with the use of
• Coma ether and chloroform for pain relief
• Death Popularized chloroform as clinical
anesthetic especially in obstetrics
XV. CONCLUSION
• Preoperative preparation can decrease the risk of John Snow Regarded as the Father of Anesthesia
complications and improve outcomes during and after Famous patient was Queen Victoria for
procedures requiring anesthesia anesthesia in labor (chloroform)
• Innovation in preoperative preparation needs to  Considered as the first anesthetist
 Became interested in anesthesia via
continue if patients are to receive the best preoperative
work in toxicology
services  Acknowldeged as “first full-time”
• Identification and modification of risk require anesthetist developing ways to
fundamentally good medicine; systems of care; clinical improve methods of Ether
assessment; and experienced, knowledgeable, and & chloroform administration
August Bier First to do spinal anesthesia (1898)
dedicated health care providers
*tuffiers line- where anesthesia is given
Achille Described epidural injections of local
SUPPLEMENTARY
Dogliotti anesthetics and the methods of
Table 10. Men in anesthesia copy pasted from trans. Source:
Adeos identification of epidural space (saline
Persona Work(s) technique)
Theodore  Loss-of-resistance technique
Identified the landmark (iliac crest which
Sir Ivan Used large bore endotracheal tube to
Tuffier corresponds to L4-L5 interspace) for
Whiteside allow plastic surgeons to operate on
epidural anesthesia; known as Tuffier’s
Magill facial injuries
line
Arthur Invented a device for suctioning
Crawford First to use ether in humans
Guedel secretions and for airway protection
Long &
John Lundy Introduced the concept of Balanced
William Clark
Anesthesia which is the use of multiple
Joseph Discovered Nitrous Oxide (1772)
drugs- sedative, narcotic analgesic,
Priestly NO2 still used for analgesia (laughing
muscle relaxant, inhalation anesthetic,
gas and initiates feeling of being “high”-
obliterate reflex response of px,
patients bang their heads on the wall
without feeling anything

CCetC
Block XX: Introduction to Anesthesia; History; Pre-operative Evaluation 13 of 15
MD 3
Dr. Quintin J. Father of Anesthesia in the Philippines  CVS- Ischemic or Failure symptoms
Gomez  GIT- Decreased liver function (pose a
 Henry Hill One of the first physicians to ablate problem with drug metabolism), Reflux
Hickman symptoms (risk for aspiration)
surgical pain via inhalation of CO2 which
 CNS- CVA, seizures, existing neuro
produced analgesia in animals deficits
 Alexander First to use the hollow needle and syringe  Renal- CKD, ↓Kidney Function
Wood combination for treatments of patients. In  Endo- DM, Thyroid problems
1858, he reported the use of hypodermic  Hema- coagulopathy
injections of morphine for treatment of  Musculoskeletal- RA, Neck stiffness
painful neuralgias. L Last Meal
 Confirm NPO status
 Carl Isolated cocaine from Erythroxylum coca
E Examination (Actual PE)
Koller in 1856, and was used in 1884 to produce
 3 Most important: Airway, Cadiovascular
reliable local anesthesia of the corneal and Pulmonary plus Neurologic for GA or
surface of the eye. Injections of cocaine Regional Anesthesia
directly into nerve trunks followed within
a year Table 13. Recommendations for Preoperative
 Alfred Introduced the less toxic local anesthetic Resting 12-Lead Electrocardiogram (ECG)
Einhorn “procaine” Pre-op 12 lead ECG Source: Adeos
 Fidel First identified the injection of local Class I
Pagés anesthetics into the lumbar and thoracic
(Procedure is indicated)
epidural space in 1921 but was
1. Pre-operative resting 12-lead ECG is
popularized a decade later by Dogliotti
recommended for patients with at least one
after perfecting the technique clinical risk factor who are undergoing vascular
surgical procedures
Table 11. Difficult Airway Assessment. Source: Adeos 2. Pre-operative resting 12-lead ECG is
Difficult Airway Assessment recommended for patients with known CHD,
LEMON peripheral arterial disease, or cerebrovascular
LOOK at the patient’s anatomy disease who are undergoing intermediate-risk
- small mandible surgical procedures
-large tongue Class IIa
-short bull neck (Procedure is reasonable to perform)
-obese 1. Pre-operative resting 12-lead ECG is reasonable
-abnormal facial/neck anatomy in persons with no clinical risk factors who are
EVALUATE – 3, 3, 2 finger widths between undergoing vascular surgical procedures
-incisors Class IIb
-hyoid and mentum (Procedure may be considered)
-hyoid and thyroid 1. Pre-operative resting 12-lead ECG may be
MALLAMPATI GRADE reasonable in patients with at least 1 clinical risk
OBSTRUCTION factor who are undergoing intermediate-risk
-secretions, stridor, muffled voice, mass, foreign body operative procedures.
NECK MOBILITY
-C-spine immobilization, RA, ankylosing spondylitis Class III
(Procedure should NOT be performed because it is
Table 12. YAMPLE. Source: Adeos
not helpful)
Y Why the patient is for surgery (Indication for 1. Preoperative and postoperative resting 12-lead
Surgery) ECGs are not indicated in asymptomatic persons
 Elective, emergent, urgent undergoing low-risk surgical procedures.
 Ex: Gallstones typically present as RUQ
pain but can also present as chest pain Table 14. Example Scenarios of ASA-PS Source: Adeos
A Anesthetic History
Example Scenarios
 Previous surgeries, issues with
anesthetic, post-op vomiting I healthy young man requires inguinal herniorrhaphy
 Family history- Malignant Hyperthermia IE Medical student, 24 years old, non hypertensive, no
 Will guide preventive measures bronchial asthma, non diabetic, for emergency
M Medications appendectomy
 Allergies 3rd year med student, gravida 1, 39 weeks for
 Medications taken emergency CS, non diabetic, non asthmatic, no
P Past Medical History (Review of Systems) gestational hypertension (pregnancy is stage I in the
 Respi- COPD, Asthma (use of puffers to
Philippines, stage II in US)
breathe), Smoking

CCetC
Block XX: Introduction to Anesthesia; History; Pre-operative Evaluation 14 of 15
MD 3
healthy young patient for emergency GI stimulants Metoclopramid 10 Oral, IV
hemorroidectomy or emergency CS e
II 24 yr old med student with chronic depression, with
psychotic features, on anti-depressant and anti- REVIEW QUESTIONS
psychotic medications, non hypertensive, non 1. Ms F, a 24 year old med student was diagnosed
diabetic, non asthmatic with schizophrenia this past year. She has
patient describes taking oral medications for diabetes continuously been taking her p-drug and got her
but has no end-organ damage and has never condition under control. However, she also
suffered ketoacidosis; ASA patient with psychiatric experienced heart problems in the previous year
condition like schizophrenia with an episode of MI that has now presented with
IIE 25 year patient with history of childhood bronchial angina that is usually controlled by her other
asthma for emergency appendectomy medications. What is her ASA-PS classification?
III the patient in (II) had a myocardial infarction last year a. II
and now has angina usually controlled by medical b. IIE
treatment c. III
IV patient has congestive heart failure and can walk less d. IE
than half a block 2. Which of the ff drugs is an opioid but also has the
V patient has infarcted bowel and is anuric, comatose, capability to reverse other opioid drugs if given at
and has a bood pressure of 70/40 with a dopamine
high doses?
infusion
a. Morphine
VE patient with GCS score 4 for emergency evacuation
b. Nalbuphine
of subdural hematoma
c. Tramadol
d. Fentanyl
VI 72 hours after motorcycle accident, a ASA PS 1
patient comes to the OR for liver and kidney donation 3. During A’s oral inspection you found out that only
E diabetic patient in (II) suffered a strangulated hernia the soft palate and uvular base are visible, What
during the years before he developed coronary mallampati classification will you put A under?
occlusion, and sought attention promptly (rated IIE) a. I
b. II
Table 15. Pre-medication drugs Source: Adeos c. III
Classification Drug Adult Route d. IV
Dose (mg) 4. Which of the ff is not a commonly disclosed risk
zodiazepines Midozalam 1-2.5 IV of general anesthesia that has minimal impact but
Diazepam 5-10 Oral, IV is frequently occurring?
Lorazepam 0.5-2 Oral IV a. Urinary retention
Opioids Morphine 5-15 IV b. Drowsiness
Fentanyl 25-100µg IV c. Sore throat
d. Post-dura Puncture Headache
Antihistamines Diphen- 12.5-25 Oral, IV
hydramine
5. Which of the ff is not a primary goal of
α-2 agonists Cholinidine 0.1-0.3 Oral,
anesthesia?
transde
rmal a. Facilitation of Smooth Induction Anesthesia
b. Increase in Gastric Fluid pH
Antiemetics Droperidol 1.25 IV c. Reduction of Anesthetic Requirements
Ondansetron 4 IV d. Prophylaxis Against Allergic Reactions
Anticholinergics Atropine 0.3-0.6 IV
Glycopyrrolate 0.1 IV Answers: 1.C 2.B 3.C 4.D 5.A

H2 antagonists Cimitedine 200-300 Oral REFERENCES


150 Oral • Upclass notes
Ranitidine 20-40 Oral • Doctor’s lecture
Famotidine
Antacids Nonparticulate 15-30 Oral

PPI Omeprazole 20 Oral


Pantoprazole 40 IV

CCetC
Block XX: Introduction to Anesthesia; History; Pre-operative Evaluation 15 of 15
MD 3
West Visayas State University – College of Medicine – Batch 2020
Block XX
Module 6 Inhalational Anesthetics
Lecture 2
05/ 14/ 19
Dr. Victoria Hofileña

TOPIC OUTLINE Before use of IV medications, man has inhaled


I. Introduction vapors to mitigate pain. Most commonly used inhaled
II. Classification of Inhalational Anesthetics (IA) anesthetics include volatile agents.
A. Outdated
B. Gases
C. Volatile Agents
III. Uptake and Distribution
IV. Mechanism of Action
V. Minimum Alveolar Concentration (MAC)
A. Factors that affect MAC
VI. Pharmacokinetics
VII. Alveolar Partial Pressure (PA)
A. Concentration Effect
B. Second Gas Effect
C. Factors that determine PA
VIII. Effects of IA on Organ Systems
A. Circulatory
B. Respiratory
C. Brain
D. Kidney
E. Liver
F. Uterine Smooth Muscle
IX. Recovery from Anesthesia
A. Tissue Concentrations
B. Metabolism Figure 1. Anesthetics used in clinical practice. Source: Doc’s Ppt
X. Inhaled Anesthetic Agents
XI. Toxicities III. UPTAKE AND DISTRIBUTION
Review Questions Your IA are delivered to the Respiratory system and
References its main target is the brain
Appendices
• IA’s action is extremely rapid due to fact that our lungs
receive 100% of our cardiac output (CO). So from the
LECTURER BOOK REFERENCE OLD TRANS
outside, Volatile agents are delivered into the lungs. It
then diffuses into the circulatory system to reach the
I. INTRODUCTION brain and is also excreted via the venous system and
• Inhalational anesthetics refers to the delivery of gases then exhaled out thru the lungs.
to the respiratory system to produce anesthesia
IV. MECHANISM OF ACTION
• Unknown: No single, accepted definition of what
II. CLASSIFICATION OF INHALATIONAL
ANESTHETICS (IA) constitutes the anesthetic state.
• Inhalational anesthetics may be classified into the • Operational Definition:
following: Immobility in response to surgical stimulation.
Outdated Amnesia for intraoperative events.
─ Ether
─ Trilene V. MINIMUM ALVEOLAR CONCENTRATION (MAC)
Immobility is measured by MAC
─ Methoxyflurane
• This is a measure of anesthetic potency.
─ Cyclopropane
• 1 MAC = Minimum Alveolar Concentration at which
─ Chloroform
50% of humans have no response to surgical stimulus
Gases
• The stimulus is usually skin incision and the response
─ Nitrous Oxide (N2O)
elicited is movement.
─ Xenon
Amnesia is however difficult to measure as well as
Volatile Agents
analgesia.
─ Halothane
It cannot be observed in an immobile patient who
─ Enflurane
cannot remember.
─ Isoflurane
─ Sevoflurane
─ Desflurane

CCetC Group No. 35 1 of 6


MD 3 Tatud, Te, Teruel
Only surrogate measures of pain (like increased BP, VII. ALVEOLAR PARTIAL PRESSURE (PA)
and HR) suggest that inhaled agent do not suppress • Partial pressure is the pressure exerted by an
perception of pain stimuli. individual gas in a mixture.
• Factors that affect PA:
PI (Inspired anesthetic partial pressure)
Concentration effect
Second gas effect
Alveolar ventilation
Solubility
Oil: Gas partition co-efficient
Meyer-Overton Rule
Partition co-efficient
FA/FI

A. INSPIRED ANESTHETIC PARTIAL PRESSURE


• A high PI is necessary during the initial administration
of inhaled anesthetics. This is called Concentration
effect.

Figure 2. Anesthetics in order by potency. Source: Doc’s Ppt


B. CONCENTRATION EFFECT
• The higher the concentration of an inhaled anesthetic,
• Nitrous Oxide has the least potency while having the
the faster the alveolar concentration approaches the
highest MAC while the Methoxyflurane has the most
inhaled concentration.
potency.
MAC is not affected by Sex and Thyroid disease.

Table 1. Factors that affect MAC. Source: Doc’s ppt


Factor Increases MAC Decreases MAC
Max: 6 months
Age With age
old
Temperature Max: 42°C With temperature
O2 Hypoxia
CO2 Hypercarbia
Alcholism Chronic Acute
Figure 3. Anesthetics used in clinical practice. Source: Doc’s Ppt
Barometric Inc Pressure – Inc
Pressure MAC To understand this better, let’s say, the capacity of
Hemoglobin Anemia patients lungs is 4 l. A patient inhales 2 liters of
IV Agents Yes nitrous oxide and 2 liters on oxygen, that is 50%
Pregnancy Yes concentration. Since, nitrous oxide diffuses very
Local rapidly compared to other gases, half of its volume,
Cocaine Others
Anesthetics that is 50% uptake… diffuses into blood, leaving only
Sodium Hypernatremia Hyponatremia 1L in the alveoli. The alveolar volume is now only 3 L
Magnesium Hypomagnesemia Hypermagnesimia and the new concentration of nitrous oxide in alveoli
Calcium Hypocalcemia Hypercalcemia is ~ 33%. This is the first part of concentration effect
and is termed as concentrating effect.
VI. PHARMACOKINETICS So because the alveoli is lacking 1L volume, this
• It describes the uptake of inhaled anesthetics from the creates sub-atmospheric pressure in alveoli and as
alveoli to the circulation, distribution and eventual the patient inhales… 1 l of gas containing 50%
elimination, and its metabolism. nitrous oxide and oxygen replaces this volume. Now,
• The primary objective of inhalation anesthesia is to the volume of gases in alveoli is again 4L but the
achieve a constant and optimal Brain partial pressure concentration of nitrous oxide in alveoli is up to ~
of anesthetic. 37.5%. This is the second part of concentration effect
• The brain and all other tissues equilibrate with the and is termed as ventilation effect.
partial pressure of the inhaled anesthetic delivered to
them by arterial blood (Pa) therefore the PA (alveolar
partial pressure) mirrors the Pbr and is an index of
anesthetic depth.
CCetC
Block XX: Inhalational Anesthetics 2 of 6
MD 3
C. SECOND GAS EFFECT of how an inhaled agent distributes itself between 2
• The ability of the large volume uptake of one gas (First phases at equilibrium
gas) to accelerate the rate of rise of the PA of a • See appendices for comparative characteristics of inhaled
concurrently administered companion gas (Second anesthetics.
gas)
This is a consequence of the concentration effect F. OIL: GAS PARTITION CO-EFFICIENT
where a “first gas” that is soluble in plasma, such as • Measure of lipid solubility and indicates Anesthetic
nitrous oxide, moves rapidly from the lungs to plasma potency. Further explained by Meyer-Overton Rule.
and also increases the rate of uptake of the “second • Indicates the amount of gas that is soluble in oil phase.
gas”.
But oxygen also behaves as a second gas. Although G. MEYER-OVERTON RULE
we frequently talk of inhalational kinetics as a single • The lower the Oil-gas partition coefficient the lesser the
process, there are multiple steps between dialing up potency. Therefore nitrogen being least potent and
a concentration and the consequent change in effect. halothane the most potent.
The key steps are transfer from the breathing circuit
to alveolar gas, from the alveoli to plasma, and then
from plasma to the “effect-site.”

Figure 5. Anesthetics used in clinical practice. Source: Doc’s Ppt

H. PARTITION CO-EFFICIENT
• Blood gas partition coefficient is the principal
determinant of the rate at which alveolar concentration
Figure 4. Anesthetics used in clinical practice. Source: Doc’s Ppt
increases toward a constant inspired concentration. It
correlates with the rate of induction.
We will use the same illustration to explain the
So it is how fast the agent can cause
second gas effect by adding isoflurane so that the
unconsciousness. This time nitrous oxide having a
inhalation mixture is 1% isoflurane, with the 4L of the
faster rate of induction compared with halothane
mixture inhaled by the patient.
Again, half of nitrous oxide diffuse quickly to blood,
and the alveolar volume is reduced to 3L making the
new alveolar concentration of isoflurane from 1% is ~
1.33%. Now as patient inhales 1L of the same
mixture gas, the new alveolar concentration gradient
becomes ~ 1.25%.
By second gas effect, nitrous oxide increases the
alveolar concentration of a second gas.. and
induction of anesthesia becomes more rapid.

D. ALVEOLAR VENTILATION
• Increase alveolar ventilation promotes input of more
inhaled anesthetic uptake into the blood.
• higher VA ~ more rapid rate of increase of PA
= faster induction
Figure 6. Anesthetics used in clinical practice. Source: Doc’s Ppt
E. SOLUBILITY
• Solubility of inhaled anesthetics in blood and tissues is • See appendices for table of partition co-efficients.
denoted by partition coefficient - or the distribution ratio

CCetC
Block XX: Inhalational Anesthetics 3 of 6
MD 3
I. FA/FI B. METABOLISM
• Ratio of alveolar agent to inhaled agent • important difference between induction and recovery
• Higher the blood/gas partition coefficient (solubility) the • impact of metabolism on the rate of the decrease of PA
greater the uptake from the alveolus • highly lipid soluble agents (ex methoxyflurane,
• The slower the rise of FA to meet the FI halothane)
• Factors affecting: principal determinant of rate of decrease of PA is
Minute ventilation, CO, FGF, IV agents metabolism
This is the ratio of the alveolar to the inhaled agent
and.. The more soluble or the higher the blood/gas X. INHALED ANESTHETIC AGENTS
partition coef.. The greater is the uptake from the A. NITROUS OXIDE (N2O)
alveolus meaning the slower the rise of the alveolar • History: used by John Culton during the public
concentration to meet that of inspired concentration.. demonstration of anesthesia
Ok so.. all these factors mentioned were all involved • Liquid gas, odorless and colorless
in how to achieve and maintain a constant and • still used in practice today
optimal brain concentration… and thus once you
achieve this.. surgery can begin.. and you have to B. ENTONOX
• It is used as ENTONOX which is a mixture of 50%
maintain this level of anesthesia during the duration
Nitrous and 50% O2.
of the surgery.
• 50% N2O + 50% O2
• Poyinting effect - where N2O is mixed with O2 and it
VIII. EFFECTS OF IA ON ORGAN SYSTEMS
• Circulatory System remains in gaseos state
Decrease blood pressure • Use:
Some decrease in cardiac output labour analgesia
• Respiratory System field analgesia
Decrease in tidal volume • It is 35 x more soluble in blood than nitrogen and can
Increase in respiratory rate easily fill and expand any air containing cavity like an
Depress mucociliary function = pooling of mucus air emboli or pneumothorax and should be used in
and atelectasis caution in high risk patients.
• Brain • N2O is 35x more soluble in blood than N2
Decrease in brain metabolic rate • Fills and expands any air-containing cavity -
Decrease in cerebral vascular resistance thus air embolism
increasing cerebral blood flow pneumothorax
• Kidney lung cysts
Decrease GFR tympanoplasty
Decrease in renal blood blow intraoccular bubbles
• Liver • May exacerbate pulmonary hypertension
Decrease in hepatic blood flow
• Uterine Smooth Muscle C. HALOTHANE
• Halothane is a very potent inhalation anesthetic ideal
Halogenated anesthetics are potent relaxants
for asthmatics as it causes bronchodilation
Lead to increase in uterine bleeding
• Thymol preservative
IX. RECOVERY FROM ANESTHESIA • Pleasant smell, non-irritant, bronchodilation
• Rate at which the PA decreases • Potent = induce anesthesia in single puff - for pedia
with time • Ideal for asthmatics
• Elimination - through lungs • MOST ARRYTHMOGENIC - sensitizes the heart to
• Influenced by: catecholamines
tissue concentrations • Max decrease in BP, SVR, CO, HR - bradycardia
metabolism • Uterus - atony
• post op shivering
A. TISSUE CONCENTRATION • Malignant hyperthermia
• serve as reservoir • Halothane hepatitis - caused by metabolite
• influenced by: Triflouroacetic acid
duration of anesthesia
solubility D. ENFLURANE
• Epileptogenic
• contraindications:

CCetC
Block XX: Inhalational Anesthetics 4 of 6
MD 3
MH susceptibility • cardiotoxic ~ ventricular fibrillation
seizure disorder • hepatotoxic, cause profound hyperglycemia
Preexisting kidney disorder, intracranial
hypertension XI. TOXICITIES
A. CARBON MONOXIDE (CO) TOXICITY
E. ISOFLURANE • All agents react with soda lime to produce CO
• Isoflurane is a newer inhalational agent • Desflurane > Enflurane > Isoflurane > Sevoflurane >
• It is the only agent that preserves baroreceptor reflex Halothane
• Causes coronary steal Carbon monoxide is a byproduct of all agents when
• Has minimal increase in intracranial pressure reacting with soda lime and desflurane is more toxic
• Agent of choice for cardiac and neuroanesthesia than other agents with regard to CO toxicity.

F. SEVOFLURANE B. FLUORIDE NEPHROTOXICITY


• More commonly used inhalation anesthetic • Fluoride - nephrotoxic by product of metabolism in liver
• Pleasant smell, non-irritant and bronchodilation and kidney.
• Choice for pediatric anesthesia • Opposes ADH leading to polyuria.
• 2nd choice for: • Methoxyflurane > Sevoflurane > Isoflurane >
Neuroanesthesia Desflurane
cardiac anesthesia • Results in potentially permanent renal injury.
asthmatics Fluoride nephrotoxicity can result in permanent renal
injury and methoxyflurane is the most nephrotoxic.
G. XENON
• The most ideal inhalation agent however, it is very C. MALIGNANT HYPERTHERMIA
expensive and it needs special equipment for delivery. • Autosomal dominant genetic disorder of skeletal
• Characteristics: Least blood:gas partition coefficient, muscle
least soluble, fastest induction, fastest recovery • Mutation in the ryanodine receptor
• Most cardiostable • Exposure leads to continual release of Calcium from
• Not metabolised, non-flammable sarcoplasmic reticulum
• Disadvantage: very costly, needs special equipment for • Sudden hyperthermia, tachycardia, hypertension,
delivery, causes bronchospasm muscle rigidity, hyperkalemia
• Life threatening emergency
H. ETHER • Treatment: Dantrolene
• Ether was the agent used during the 1st public
demonstration of anesthesia in 1886. REVIEW QUESTIONS
• It is a complete anesthetic agent: 1. Which of the following is the correct sequence of
It has muscle relaxation potency?
stimulates respiration a. methoxyflurane>enflurane>halothane>sevoflurane
preserves cilliary function. b. sevoflurane> methoxyflurane>halothane>enflurane
• However, it has the highest incidence of nausea and c. methoxyflurane>halothane>enflurane>sevoflurane
vomiting. d. halothane> enflurane>sevoflurane>methoxyflurane

I. METHOXYFLURANE 2. Mac is altered by the following except?


• most potent, slowest induction and recovery a. hypernatremia
• most nephrotoxic b. alcoholism
c. anemia
J. CYCLOPROPANE d. male sex
• most flammable and explosive
• causes cyclopropane shock 3. MAC is highest in:
a. 6 months old
K. TRICHLOROETHYLENE b. pregnancy
• most potent analgesic agent c. elderly
• reacts with soda lime: dichloroacetylene - neurotoxic d. hypoxia
phosgene - pulmonary toxicity ~ ARDS
4. Inhalation anesthetic that poses highest risk for
L. CHLOROFORM CO toxicity is
• 1st agent used for labor analgesia a. methoxyflurane
b. desflurane
CCetC
Block XX: Inhalational Anesthetics 5 of 6
MD 3
c. sevoflurane
d. nitrous oxide

5. A factor that prolongs the rate of induction is


a. high alveolar ventilation
b. high inspired concentration of anesthetic
c. high blood/gas partition coefficient
d. high potency

Answers: C D A B C

APPENDICES

REFERENCES
• Upclass notes
• Doctor’s lecture / ppt

CCetC
Block XX: Inhalational Anesthetics 6 of 6
MD 3
West Visayas State University – College of Medicine – Batch 2020
Block XX
Module 6 Regional Anesthesia
Lecture 3
05/ 14/ 19
Dr. Flores

TOPIC OUTLINE • The spine consists of 33 vertebrae, 24 true, and 9


I. Introduction fused/fixed
A. Anatomy 7 cervical
B. Preoperative Evaluation
C. Spinal vs Epidural Anesthesia
12 thoracic
i. Indications 5 lumbar
ii. Contraindications 5 fused sacral
II. Spinal Anesthesia 4 fused coccygeal
i. Important Landmarks
ii. Distribution
iii. Adjuvants Spinal Cord
iv. Sequence of Neural Blockade Begins at foramen magnum and ends at:
v. Sympathetic Block
─ L1 in adults
vi. Alternative Local Anesthestics for Short Duration
vii. Long Duration Spinal Anesthesia ─ L3 in infants
viii. Complications Terminal end is called the conus medullaris
ix. Treatment of Complications
A. Midline Approach – Subarachnoid Block (SAB)
B. Paramedian Approach – SAB
C. Lumbosacral Approach - SAB
III. Epidural Anesthesia
i. Indications
ii. Identification of Epidural Space
iii. Things to Remember
iv. Adjuvants
v. Advantages
vi. Complications
Review Questions
References
Appendices

LECTURER BOOK REFERENCE OLD TRANS

Figure 2. Important landmarks involved in spinal anesthesia. Source:


I. INTRODUCTION Doctor’s lecture
A. ANATOMY *See appendix for an enlarged picture

The identification of the vertebral spinous processes


can be used to differentiate between regions of the
vertebral column and facilitate the identification of the
position of deeper structures such as the inferior
end of the spinal cord in the subarachnoid space
C2 vertebra (spinous process) - identified through
palpation of the most superior bony protuberance in
the midline, inferior to the skull
Most of the other 5 processes cannot be palpated
because some tissues obscured it
C7 (spinous process) - prominent eminence in the
midline at the base of the neck
Ligamentum nuchae - extends between the external
occipital protuberance of the skull
In the picture, you can palpate the tip of coccyx (the
inferior end of the subarachnoid space) and to
identify L3 and L4 (landmark: posterior superior
iliac spine)
Figure 1. Spinal anatomy. Source: Internet

CCetC Group No. 32 1 of 9


MD 3 Solidarios, Subo, Sumbillo
• Relative
Bacteremia
May lead to meningitis
Preexisting neurologic disease
Cardiac disease
e.g. valvular heart disease and congestive
heart failure. High risk of complications if
anesthesia is done.
Abnormal coagulation
Risk for hematoma formation if Batson plexus is
injured, which may lead to paralysis if not
addressed.
Figure 3. Layers traversed by the needle in spinal anesthesia. Source:
Infection at the site of planned needle puncture
Internet Elevated ICP
Bleeding diathesis

Layers traversed by the spinal needle: Table 1. Spinal vs Epidural Anesthesia. Source: Doctor’s
1. Skin lecture
2. Subcutaneous tissue Spinal Epidural
3. Supraspinous ligament Injecting local anesthetic Injecting local
4. Interspinous ligament solution into the CSF within anesthetic solution into
5. Ligamentum flavum the subarachnoid or the epidural space
6. Epidural Space intrathecal space
7. Dura Anesthetic is
8. Subarachnoid space injected into the
epidural space before
Note: A more extensive review of spinal anatomy (which the solution comes into
was not discussed) can be found in ADEOS Notes. contact with nerve
fibers. Thus, epidural
B. PREOPERATIVE EVALUATION anesthesia needs more
• Discussion with the patients for benefits and potential volume of anesthetic
complications solution.
• Rare but serious complications: Limited to the lumbar region May be given at various
Nerve damage below the termination of the levels of the neuraxis
Bleeding spinal cord
Infection
• Common but minor complications: Limited to L2 to L3 to avoid
Post-dural puncture headache damaging the spinal cord
(which ends at L1)
C. SPINAL VS. EPIDURAL ANESTHESIA - Less time to perform - Ability to produce
INDICATIONS segmental sensory
• Spinal anesthesia Medication is given directly block
Lower abdominal area into the CSF
- Greater control over
Perineum the intensity of sensory
- Less discomfort
Lower extremities and motor block
• Epidural anesthesia Smaller gauge is used and
Abdomen - Allows titration of the
requires less anesthetic (4 cc)
Lower extremities block to the duration of
surgery, control post –
To supplement general anesthesia (especially for - Requires less anesthetic op pain
thoracic and upper abdominal surgeries)
- More intense sensory and - Decreased risk for
CONTRAINDICATIONS motor block postdural puncture
• Absolute headache
-Appearance of the CSF
Patient refusal

CCetC
Block XX: Regional Anesthesia 2 of 9
MD 3
II. SPINAL ANESTHESIA Supplementary Notes
• Preparation: Baricity
Equipment • Has something to do with your specific gravity of the
Drugs anesthetic as compared to the specific gravity of the
Monitors CSF. The normal specific gravity of the CSF is 1.007. If
Supplemental oxygen the anesthetic is of hyperbaric type, it has a specific
• To decrease discomfort, inclusion of an opioid in pre- gravity greater than 1.007. If Isobaric type, specific
op meds gravity is 1.007. If hypobaric type, specific gravity is
• Pre-op meds can be withheld provided that there is below 1.007.
adequate attention to infiltration of the skin and • Clinical Significance: Hyperbaric anesthetic
subcutaneous tissues with a local anesthetic solution (Mepivacaine) has tendency to gravitate towards the
• Sterile technique dependent portion. For example, in appendectomy, in
order to achieve a full block/sensory block up to the
IMPORTANT LANDMARKS level of T4 (nipple line) and we inject the local
Patient should be in lateral decubitus position (right anesthesia at the level of L3-L4. After injecting the
or left) local anesthesia, the patient must assume the
• Tuffier’s line Trendelenburg position or the Head-Down position. So
Line drawn between iliac crests that the anesthetic gravitates cephalad.
Traverses the body of L4 vertebra • If hypobaric type of Mepivacaine is utilized, after
Puncture is done at the level of L3 and L4 injecting the anesthetic and we want to assume the T4
• C7 spinous process block, the patient is placed in a Head-Up position.
Bony knob at the lower end of the kneck • If isobaric type, after injecting the anesthetic the patient
• T7-8 interspace assumes a flat-on-bed position. The anesthetic will just
Lower limits of the scapulae spread towards the cephalad area based on the
Terminal portion of the 12th rib intersects L2 volume given.
vertebral body • What influences the baricity of an anesthesia?
• Posterior iliac spines If Mepivicaine is mixed with D5 water, then it
Indicate S2 vertebral body becomes hyperbaric. Isobaric if we mixed it with
plain NSS. Hypobaric, if we mixed it with sterile
DISTRIBUTION water.
• These are the factors that affect distribution of the
anesthetic in spinal anesthesia Patient position
Baricity • Spine should be flexed by having the patient bend at
Patient position the waist and bring the chin toward the chest, which
Dose, volume, and concentration will optimize the interspinous space and interlaminar
Injection site foramen
Patient characteristics
Dose, Volume and Concentration
Baricity • The higher the concentration, the speedier the onset.
• Ratio of the density (mass/volume) of the local • The smaller the dose of the anesthetic, decreased
anesthetic solution divided by the density of CSF spread of the anesthetic.
• Predicts the direction of local anesthetic
• Hyperbaric (>1.007) Injection Site
Most commonly used • Epidural anesthesia can be performed above L3-L4,
Prepared using dextrose solution since the anesthetic is just inserted in the epidural
Ability to achieve greater cephalad spread space and not further into the subarachnoid space.
• Hypobaric (<0.997) Spinal anesthesia is strictly below L3-L4.
Generally reserved for patients undergoing • One advantage of epidural anesthesia is that it allows
perineal procedures in jack-knife position for Segmental blockade. For example is when we
Prepared using sterile water perform mastectomy, the block is usually at the level of
• Isobaric (0.998-1.007) thoracic area. If you want to block T1-T10 only, you
Prepared using NSS can perform epidural anesthesia at the level of T5.
More profound motor block and longer duration of Spinal anesthesia, however, can only block below or
action above the sacral area (never segmental).
Not influenced by patient position

CCetC
Block XX: Regional Anesthesia 3 of 9
MD 3
• Differential blockade is also an advantage of epidural Preganglionic
anesthesia. For example, in labor analgesia, only the sympathetic blockade
sensory is blocked. This is done by reducing the
concentration of the local anesthetic through dilution Arteriolar and venous
with plain NSS or sterile water or D5 water. dilatation

Patient Characteristics
Increased vascular
• Pregnant patients - lower dose needed capacitance
• Patients with acute abdomen - lower dose needed
• Elderly patients - osteopenic spines develop, the
caliber of spaces of the vertebra become smaller; Pooling of Blood
lower doses needed
End of Supplementary Notes
Decreased Venous
Return
ADJUVANTS
Vasoconstrictors
• Increase the duration of spinal anesthesia Decreased Cardiac
Output
Due to reduction in spinal cord blood flow, which
decreases loss of local anesthetic from the Figure 4. Sympathetic blockade in spinal anesthesia. Source: Doctor’s
lecture.
perfused areas and thus increases the duration of
exposure to local anesthetic
• Epinephrine (0.1-0.2mg) or phenylephrine (2-5mg) ALTERNATIVE LOCAL ANESTHETICS FOR SHORT
DURATION
Epinephrine is only effective when used with
• Mepivacaine
tetracaine. With bupivacaine, it does not affect the
Has incidence of transient neurologic symptoms
duration of action because bupivacaine per se has its
• Procaine
own vasoconstrictor property.
Very short duration of action with tendency to
cause nausea.
Opioids and other analgesic agents
• Chlorprocaine
• Enhance surgical anesthesia and provide postoperative
Phased out
analgesia
• Fentanyl (25ug) for short surgical procedures
LONG DURATION SPINAL ANESTHESIA
i.e. labor anesthesia; comes in 50-mcg
• Bupivacaine
concentration
Only available local anesthetic in practice is; has
• Morphine (0.1- 0.5mg) effective for ~24 hours
more profound sensory block
Most effective opioid; available in 10- and 16-mg
• Tetracaine
vials
No longer manufactured but has more profound
motor block
SEQUENCE OF NEURAL BLOCKADE
Both have a duration of 90 to 100 minutes but the
*in order
administration of epinephrine to tetracaine can
• Loss of pain and temperature sensation
prolong the action for 30 minutes to one more hour
• Loss of proprioception
• Loss of touch and pressure sensation
COMPLICATIONS
• Motor paralysis
• Hypotension
• Bradycardia
SYMPATHETIC BLOCK
• Post spinal headache
Upon the introduction of anesthetic into the
• High Spinal
subarachnoid space, there is a sympathetic block.
• Nausea
This is why we monitor the vital signs of the patient.
• Urinary Retention
• Backache
• Neurologic Sequelae
• Hypoventilation

CCetC
Block XX: Regional Anesthesia 4 of 9
MD 3
TREATMENT OF COMPLICATIONS • Prone position is rarely used except for perineal
Hypotension Treatment procedures. It is more challenging because of the
• IV fluid pre-loading at 10-20cc/kg limited flexion, contracted dural sac, and the low CSF
• Head-down position 5-10 degrees pressure.
• Sympathomimetics End of Supplementary Notes
• Ephedrine (5-10mg IV)
Indirect sympathomimetic PROCEDURE
• Colloids or additional crystalloids • Needle is inserted at the top margin of the lower
To hydrate and increase the preload because the spinous process of the selected interspace
sympathetic block will decrease the preload • Needle is progressively advanced in a slight cephalad
orientation
Spinal Headache Treatment • The needle is then advanced, in order, through the:
• Bed rest Subcutaneous tissue
• Oral analgesics Supraspinous ligament
• Hydration Interspinous ligament
• Caffeine sodium benzoate 500mg IV or caffeine Ligamentum flavum
containing beverages
• Epidural blood patch • Once the needle tip is believed to be in the
Done by getting 10 cc of blood from the patient subarachnoid space, the stylet is removed to see if
and injecting it to the epidural space CSF appears at the needle hub.

High Spinal Anesthesia Treatment • When redirecting a needle it is important to withdraw


• Ventilatory support the tip into the subcutaneous tissue.
• Circulatory Support (sympathomimetic drugs) If the tip remains embedded in one of the vertebral
• Head-down position ligaments, then attempts at redirecting the needle
will simply bend the shaft
A. MIDLINE APPROACH – SUBARACHNOID BLOCK Clue: CSF
• Easier
Up to the subcutaneous tissue, if the first attempt is
• Passes through less sensitive structures
not successful, the spinal needle can be reinserted
• Less local anesthetic infiltration
again
• Positioning of patient:
Sitting
• If the patient experiences a paresthesia, it is important
Lateral decubitus
to determine whether the needle tip has encountered a
Jack-knife position/prone position
nerve root in the epidural space or in the subarachnoid
space.
Supplementary Notes
If there is paresthesia, just pull out the spinal needle.
• Sitting: encourages flexion and facilitates recognition
If CSF is not visible at the hub, then the
of midline which may be of increased importance in
paresthesia probably resulted from contact with a
obese patients. Because lumbar CSF is elevated in
spinal nerve root traversing the epidural space.
this position, the dural sac is distended thus providing
This is especially true if the paresthesia occurs in
a larger target for the spinal needle.
the dermatome corresponding to the nerve root
With legs hanging over side of the bed
that exits the vertebral canal at the same level
Have patient hug a pillow
that the spinal needle is inserted.
Put feet up on a stool (no wheels)
In this case the needle has most likely deviated
Assistant must keep the patient from swaying
from the midline and should be redirected toward
Curve back like a “C”, Halloween Cat, Shrimp,
the side opposite the paresthesia. Occasionally,
Canon block
pain experienced when the needle contacts bone
For obese and pregnant patients
may be misinterpreted by the patient as a
• Lateral decubitus
paresthesia and the anesthesiologist should be
Needs to be parallel to the edge of the bed
alert to this possibility.
Legs flexed up to abdomen
Forehead flexed down towards knees
• The presence of CSF confirms that the needle
• Jack-knife Position/Prone Position
encountered a cauda equine nerve root in the
For ano-rectal surgery
suparachnoid space and the needle tip is in good
CSF will not drip from the hub of needle
position.
Use hypobaric solution
CCetC
Block XX: Regional Anesthesia 5 of 9
MD 3
CSF is gently aspirated to confirm that the needle is will be a sudden loss of resistance and the saline will
still in the subarachnoid space and the local be suddenly injected.
anesthetic slowly injected (≤0.5 ml/s-1). Once the block is placed, strict attention must be
After completing the injection, a small volume of paid to the patient's hemodynamic status
CSF is again aspirated. Block height should also be assessed early by pin
prick or temperature sensation
TECHNIQUE
B. PARAMEDIAN APPROACH – SUBARACHNOID
BLOCK
• Useful in situations where the patient's anatomy does
not favor the midline approach:
Inability to flex the spine or
Heavily calcified interspinous ligament
For patients with trauma (lower extremity), heavily
calcified spinous ligaments such as in geriatrics
• Best approach for the patient in the prone jackknife
position

PROCEDURE
• The first significant resistance encountered should be
the ligamentum flavum
Figure 5. Proper needle positioning in spinal anesthesia through • Bone encountered prior to the ligamentum flavum is
paraspinous (a) and midline (b) approaches. Source: Internet usually the vertebral lamina of the cephalad
vertebra and the needle should be redirected
Hold the needle in a pencil like manner. Anchor your
accordingly
hand at the back using the knuckle
Can encounter a lot of vasculatures; prone to
Spinal gauge 25 is used
bleeding
The lesser the gauge the lesser the postural
• An alternative method is to insert the needle
headache
perpendicular to the skin n all planes until the lamina is
contacted
• The needle is then walked off the superior edge of the
lamina and into the subarachnoid space

C. LUMBOSACRAL APPROACH
• Taylor approach
• Paramedian approach directed at the L5-S1
interspace

PROCEDURE
• The needle is inserted at a point 1 cm medial and 1
Figure 6. Proper needle insertion in spinal anesthesia – midline
approach. Source: Doctor’s lecture cm inferior to the posterior superior iliac spine
*See appendix for an enlarged picture • The needle is angled cephalad 45 to 55 degrees and
just medial enough to reach the midline at the level of
Free flow of CSF confirms correct placement the L5 spinous process
Needle is secured by holding the hub between the
thumb and the index finger III. EPIDURAL ANESTHESIA
Syringe is then attached and the CSF is aspirated to • The major site of action of local anesthetics placed in
reconfirm placement the epidural space appears to be the spinal nerve
Contents delivered to the space over an 3-5sec roots.
period • Anesthesia may also result from the extension to the
Aspiration and reinjection is done as the induction subdural area of the local anesthetic
nears end Local anesthetic delivered into the epidural space will
If the needle tip is properly engaged in the diffuse to the spinal nerve root. Doing this anesthesia
ligamentum flavum, it should be possible to would take 30 minutes to an hour for the onset of the
compress the air bubble without injecting the saline. effect.
As the needle tip enters the epidural space, there
CCetC
Block XX: Regional Anesthesia 6 of 9
MD 3
Fast onset local anesthetic is used to test if
approach is intrathecal
In epinephrine, if you are intravascular there is a
positive increase of 20 to 30 beats per minute of
cardiac rate.

Table 2. Onset of action and duration of several anesthetics.


Source: Doctor’s lecture
Drug Onset Duration- With epi
(min) plain (min) (min)
Figure 7. Needles used in epidural anesthesia. Source: Doctor’s
lecture Chloroprocaine 5-10 45-60 60-90
Lidocaine 10-15 60-120 90-180
Epidural anesthesia uses a larger needle gauge vs. Bupivacaine 15-20 120-200 150-240
spinal anesthesia Ropivacaine 10-20 120-800 150-20

INDICATIONS Ropivacaine is commonly used because the onset


• Analgesia and duration is more predictable than bupivacaine.
• Anesthesia (with variable blocks) pH is responsible for onset
• Labor and delivery Lipid solubility is related to potency
• Prolonged post-op pain relief Protein binding for duration of action
• All indications for spinal anesthesia
ADJUVANTS
IDENTIFICATION OF THE EPIDURAL SPACE • Epinephrine
• Loss of Resistance Technique Decreases vascular absorption
A syringe containing air/saline or both is attached Maintains effective anesthetic concentrations
to the needle • Opioids
The needle is slowly advanced while assessing To enhance surgical anesthesia and to control
resistance post-operative pain
An abrupt loss of resistance to injection signals Lipid solubility:
passage through the ligamentum flavum and into ─ Morphine - hydrophilic, spreads rostrally within
the epidural space the CSF
Usually used in practice because it is easily done ─ Fentanyl - lipophilic, rapidly absorbed and
• Hanging Drop Technique exhibits less rostral spread.
A small drop of saline is placed at the hub of the • Sodium bicarbonate
epidural needle Because local anesthetics are weak bases, they
The drop is retracted into the needle by the exist largely in the ionic form
negative pressure in the epidural space Adding sodium bicarbonate favors the non-
Associated with higher incidence of accidental ionized form of local anesthetic and favors faster
dural puncture onset of anesthesia

THINGS TO REMEMBER WITH EPIDURAL ADVANTAGES


PLACEMENT • Maintain continuous anesthesia after placement of
• Thread the catheter 3-5 cm catheters – suitable for prolonged procedure
Remove the needle while keeping positive 3 to 4 days
pressure in the catheter Check puncture site and dress well to avoid
Check position infections
Secure catheter • Post-op analgesia
For continuous epidural anesthesia, retain the
catheter into the epidural space. COMPLICATIONS
• Test dose • Backache
Aspirate for blood or CSF Needle trauma, local anesthetic irritation, and
Paramedian insertion usually results in higher ligamentous strain secondary to muscle
blood vessel puncture relaxation
1.5% lidocaine with epinephrine
vs. spinal anesthesia – no test dose

CCetC
Block XX: Regional Anesthesia 7 of 9
MD 3
• Post-dural puncture headache
Loss of CSF through the meningeal needle hole
resulting in decreased buoyant support for the
brain. In the upright position the brain sags in the
cranial vault putting traction on pain-sensitive
structures
• Hearing loss
• Systemic toxicity
Does not occur with spinal anesthesia because
the drug doses used are too low to cause toxic
reactions even if injected intravenously. Both
CNS and cardiovascular toxicity may occur
during epidural anesthesia
• Total spinal
Does not occur with spinal anesthesia because
the drug doses used are too low to cause toxic
reactions even if injected intravenously. Both
CNS and cardiovascular toxicity may occur
during epidural anesthesia
• Neurologic Injury
Transient Neurologic Injury defined as pain,
dysesthesia, or both in the legs or buttocks after
spinal anesthesia
• Spinal Hematoma
Coagulation defects are the principal risk factor
for epidural hematoma.

REVIEW QUESTIONS
FROM THE LECTURE
• Layers traversed by the Spinal Needle:
1. Skin
2. Subcutaneous tissue
3. Supraspinous ligament
4. Interspinous ligament
5. Ligamentum flavum
6. Epidural Space
7. Dura
8. Subarachnoid space

REVIEW QUESTIONS
True or false
1. Epidural anesthesia uses more volume of
anesthetics than the spinal anesthesia.
2. The identification of the vertebral spinous processes
can be used to differentiate between regions of the
vertebral column.

Fill in the blanks


The spinal cord begins at ________and ends at __ in
adults and __ in infants.
Answers: T,T,Foramen magnum, L1, L3

REFERENCES
• Upclass notes
• Doctor’s lecture

CCetC
Block XX: Regional Anesthesia 8 of 9
MD 3
APPENDICES

Figure 2. Important landmarks in spinal anesthesia. Source: Doctor’s lecture

Figure 6. Proper needle insertion in spinal anesthesia – midline approach. Source: Doctor’s lecture

CCetC
Block XX: Regional Anesthesia 9 of 9
MD 3
West Visayas State University – College of Medicine – Batch 2020
Block XX
Module 6 Intravenous Anesthesia
Lecture 7
05/ 15/ 19
Michael Y. Castanos MD, DPBA, FPSA

TOPIC OUTLINE IV DRUG DISTRIBUTION


I. Ideal IV Anesthetic Drug
II. Propofol
III. Barbiturates
IV. Benzodiazepines
V. Ketamine
VI. Etomidate
VII. Dexmedetomedine
Review Questions
References
Appendices

LECTURER BOOK REFERENCE OLD TRANS

IV vs Inhalational anesthesia Figure 1. Drug distribution in different tissues. Source: internet


In IV, drugs are administered directly through
intravenous while inhalational is given through your Upon entering the bloodstream, most of the drug will
trachea and alveoli be present in the plasma & will bind to the plasma
Inhalationaluses gas or volatile anesthetics. proteins. And the remaining unbound drug are free
IV induction drugs drug, which are the active component of the drug.
When given intravenously they will cause rapid Dictum in IV anesthetics: the more lipid loving the
loss of consciousness more potent the drug is, because the brains is mostly
1 arm brain circulation phospholipids.
The time for the drug to travel from the arm to the Three compartments
brain Vessel rich- heart, kidney, lungs & brain. 70% of
The target organ for general anesthesia is the brain the drug will be taken here.
Less- skeletal muscles & skin
I. IDEAL IV ANESTHETIC DRUG Minimal- adipose, bones & connective tissue
It should be without cardiac & respiratory Induction
complications. But there is no single drug that can High concentration of drug in the bloodstream will
attain this so we do multiple drug approach go to the brain which has a lower concentration
• Hypnosis Three characteristics of IV anesthetic drugs
• Amnesia Lipid solubility
• Analgesia Degree of ionization (when the drug is ionized it
Anesthesia vs Analgesia can’t enter the brain)
Anesthesia → no sensation, Analagesia → no pain Arterial concentration
• Muscle relaxation The particular receptor that the IV drugs interact with
Very important especially during abdominal is the GABA which is an inhibitory neuron. The drugs
surgery potentiates the activity of the GABA
Normally, abdominal muscle reacts to an invading
stimuli as part of the protective reflexes of the body II. PROPOFOL
If the anesthetic drugs does not have enough
muscle relaxation property, the surgeon will have a
difficulty in incising the abdomen kay ma tig-a

Figure 2. Propofol chemical structure. Source: internet

CCetC Group No. 37 1 of 6


MD 3 Velasco B, Velasco J, Villacrusis
A. PHYSIOCHEMICAL PROPERTIES C. PHARMACODYNAMICS
• 2, 6 diisopropylphenol • Acts through potentiation of the chloride current
• Alkyl phenol with HYPNOTIC properties through the γ–aminobutyric acid Type A (GABA-A)
• Insoluble in aqueous solutions receptor complex
• Formulated as emulsion
• Usually contains: CENTRAL NERVOUS SYSTEM
10% soybean oil Primarily hypnotic and NOT analgesic
2.25% glycerol Decrease cerebral bloodflow (CBF) and cerebral
1.2% lecithin metabolic rate for oxygen (CMRO) → results in
• The available formulation support bacterial growth → decrease intracranial pressure
GOOD sterile technique is important! Good for neuro procedures
Since lipid emulsions are good for bacterial growth, Monro-Kellie doctrine – any increase in the three
they added bacterial retardants substance of the brain (blood, CSF, brain tissue)
• Bacterial retardants: will cause a decrease of the other two leading to ↑
EDTA 0.05 mg/mL ICP
Metabisulfate 0.25 mg/mL Neuroprotective
Benzyl alcohol 1 mg/mL Anticonvulsant
• Milky-white
• Slightly viscous CARDIOVASCULAR SYSTEM
• pH – 7 Vasodilation
• 1% (10 mg/mL) propofol concentration ─ Occurs in both arterial and venous circulation
When IV propofol is given, it is very painful. Give ─ Reduction in preload and afterload
lidocaine before administering propofol. ↓ systemic blood pressure (SBP) is more
It is a short acting anesthetic. IV drugs should be pronounced
short acting so that there is a rapid onset and rapid ─ Especially with increasing age; patient with
offset. reduced intravascular fluid volume; and with
Onset of action starts 30 sec after drug administration rapid injection
Recovery within 2-4 mins Exacerbates hypotensive effect
Propofol is the ideal drug for IV general anesthetic. Profound bradycardia and asystole, even on
The drug that killed MJ healthy adults despite prophylactic anticholinergic

B. PHARMACOKINETICS RESPIRATORY SYSTEM


• Rapidly metabolized in the liver Potent respiratory depressant → produces apnea
• Excreted through the kidneys post-induction
• High plasma clearance and exceeds hepatic blood flow The brain can only survive 5 mins of apnea due to
Extrahepatic metabolism is important! decreased supply of O2
─ Lungs play the major role Reduction in tidal volume and respiratory rate
─ Accounts for the elimination of up to 30 % of a Reduction in upper airway reflexes → hence, the
bolus dose use of laryngeal mask airway
─ This explains a more complete recovery from Decreases incidence of wheezing and tracheal
propofol with LESS “HANGOVER” than intubation among healthy and asthmatic patients
thiopental
Hangover means the drug eliminated completely OTHERS
within an hour Antiemetic
• Continuous IV infusion Area postrema is the antiemetic area of the brain.
Rapid metabolism – efficient plasma clearance It is modulated by GABA and causes the decrease
Slow redistribution from poorly perfused of serotonin.
compartments back into the central compartments Unexpected tachycardia
Context-sensitive half-time is brief ─ Should prompt laboratory evaluation
The time taken for the blood plasma concentration ─ Possible metabolic acidosis
of the drug to decline by 50% after you stop giving ─ Propofol infusion syndrome
the drug Since propofol is acidic, prolonged infusion of the
drug causes propofol infusion syndrome which
leads to metabolic acidosis, cardiac failure and
kidney failure. It is often critical but rare.

CCetC
Block XX: Intravenous Anesthesia 2 of 6
MD 3
MJ chose propofol because it stimulates nucleus C. PHARMACODYNAMICS
accumbens which is the reward system of the brain CENTRAL NERVOUS SYSTEM
that releases dopamine. • Sedation to general anesthesia in induction doses
• No analgesic effect (reduces pain threshold)
D. CLINICAL USES • Potent cerebral vasoconstrictor
• Induction of anesthesia Decrease CBF, CBV, ICP, CMRD
• Maintenance of anesthesia Useful in management of patient with space-
• Sedation occupying intracranial lesions
• Antiemetic • Neuroprotection
For focal cerebral ischemia
III. BARBITURATES
CARDIOVASCULAR SYSTEM
• Decreases SBP
Vasodilation
Barbiturate-induced depression of the medullary
vasomotor center and decreased sympathetic
nervous system from the CNS

RESPIRATORY SYSTEM
• Depresses the respiration → leads to decreased
minute ventilation through reduced tidal volume and
RR
Figure 3. Barbiturate chemical structure. Source: internet • Induction doses induces transient apnea
• Decreases ventilatory response to hypercapnia and
The popular drug used in the hospital is Thiopental, hypoxia
so we will be focusing on it. Chemorecptor area in the brain which senses co2
• Slow breathing rate and decrease tidal volume
A. PHYSIOCHEMICAL PROPERTIES
• Lacks hypnotic effect SIDE EFFECTS
• Both are formulated as sodium salts mixed with • Severe tissue injury involving gangrene – accidental
anhydrous sodium carbonate intra-arterial injection
• After reconstitution with water and NSS, the solution Due to alkaline property
are alkaline with pH > 10 → prevents bacterial growth • Local tissue irritation – accidental subcutaneous
and helps increase the shelf-life injection
• Leads to precipitation when mixed with acidic drug • Life-threatening allergic reaction – RARE
preparation such as NMBD Barbiturates cannot be given to asthmatic patient
Can irreversibly block intravenous delivery lines
Accidental injection into artery will cause extreme D. CLINICAL USES
pain and may lead to severe tissue injury • Induction of anesthesia
NMBDs are often administered shortly after the
B. PHARMACOKINETICS barbiturate to produce skeletal muscle relaxation
• Undergo hepatic metabolism by oxidation, N- Thiopental + Succinylcholine: Classic drug
deakylation, desulfuration, and destruction of barbituric regimen for “rapid sequence induction of
acid ring structure anesthesia”
• Resulting metabolites are inactive and excreted You do rapid sequence induction of anesthesia in
through urine, and after conjugation through bile cases like intestinal obstruction. The patient is
• Should not be administered to patient with acute placed in supine position and he has intestinal
intermittent porphyria obstruction. His abdomen is big and the stomach
Chronic administration enhances barbiturate contents push the diaphragm upwards. During
metabolism giving of anesthetics, the lower esophageal
Production of porphyrin is increased through sphincter will relax and gastric contents will go up
stimulation of aminolevullinic acid synthase and may lead to aspiration. So in rapid sequence
induction, we do the Sellick maneuver(applying
pressure to the cricoid cartilage). We push the

CCetC
Block XX: Intravenous Anesthesia 3 of 6
MD 3
cricoid cartilage against the esophagus to seal it to
prevent the gastric content to go up. SPECTRUM OF EFFECTS:
• Neuroprotection • Mediated through the α1-subunitof the GABA
receptors
IV. BENZODIAZEPINES Sedative-hypnotic
Amnestic
Used to treat seizure
• Mediated through the γ-subunit of the GABA receptors
Anxiolysis

CENTRAL NERVOUS SYSTEM


• ↓CMRO, and CBF
• Potent anticonvulsant

CARDIOVASCULAR SYSTEM
• ↓↓SBP
Figure 4. Benzodiazepine chemical structure. Source: internet
RESPIRATORY SYSTEM
The popular drugs are the midazolam (brand name: • Minimal depression of ventilation
Dormicum) and diazepam(brand name: Valium). • Depression increases when co-administered with
We will only tackle midazolam because it has the opioids
highest lipid solubility, it has a rapid action, and it’s
the most common drug used in the hospital. SIDE EFFECTS
• Allergic reaction – rare
A. PHYSIOCHEMICAL PROPERTIES • Pain during injection
• Contains benzene ring fused to a seven-member
diazepine ring, hence the name D. CLINICAL USES
• Highly lipophilic • Preoperative medication
• Midazolam – highest lipid solubility; this speeds its • Intravenous sedation
passage across the blood-brain barrier and its onset of • Intravenous induction of anesthesia
action • Suppression of seizure activity
• Highly protein bound, mainly to serum albumin
V. KETAMINE
B. PHARMACOKINETICS
• Highly lipid soluble
• Rapid onset of action
• Metabolism in the liver through microsomal oxidation,
N-deakylation and aliphatic hydroxylation or
glucuronide conjugation
• Midazolam – has the shortest context-sensitive half-
time, which makes it the only one that is suitable for
continuous infusion

C. PHARMACODYNAMICS Figure 5. Keyamine chemical structure. Source: internet


• Activation of GABA, receptor complex and
enhancement of GABA-mediated chloride currents –
A. PHYSIOCHEMICAL PROPERTIES
leads to hyperpolarization of neurons and reduced • Phencyclidine derivative
excitability • Produces significant analgesia
Benzodiazepine is an anti-anxiety drug. It is given • Known as “dissociative anesthesia”
orally1 night or 1 hour before the procedure to relax It dissociates the thalamus (receives inputs from
the anxious patient. stimuli felt all over the body) from the cortex. When
The drug’s purpose is to make you forget the things it dissociates, the brain can’t interpret sensations.
that hurt you. (anterograde amnesia) When the drug is given before the operation and
• Minimal effects of these drugs outside the CNS → the surgeon starts to incise an area in the patient’s
magnitude of ventilation depression and hypotension body, he cannot feel the pain.
are rare
CCetC
Block XX: Intravenous Anesthesia 4 of 6
MD 3
Patient’s eye remain open with a slow nystagmus Causes adrenocortical suppression by producing a
gaze (cataleptic gaze) dose-dependent 11 β-hydroxylase, an enzyme
• Partially water-soluble, but highly lipophilic necessary for the conversion of cholesterol to
cortisol
B. PHARMACOKINETICS Suppression last 4-8 hours
• Lipid soluble Contraindicated to patients with adrenal
• Rapid onset of drug effect insufficiency
• Metabolism occurs primarily in the liver through N-
demethylation by CYP450 system D. CLINICAL USES
• This is the only IV anesthetic that has the low • Alternative to propofol and barbiturate
protein binding
Meaning, it is very potent because less drugs are VII. DEXMEDETOMEDINE
bound to plasma protein and more free drug • Highly selective α2-adrenergic agonist

C. PHARMACODYNAMICS A. PHYSIOCHEMICAL PROPERTIES


• Lacrimation and salivation are increased and • Active S-enantiomer of medetomidine
premedication with an anticholinergic is indicated to • Used in veterinary medicine
limit the effect
A popular anticholinorgic is Atropine B. PHARMACOKINETICS
• Emergence reactions: • Rapid hepatic metabolism
Vividcolorful dreams Conjugation, N-Methylation, and hydroxylation
Hallucinations Bullet 2
Out-of body experience
Increased and distorted visual, tactile, and auditory C. PHARMACODYNAMICS
• Hypnosis results from structure of α2receptor in the
sensation
locus ceruleus and the analgesic effect at the level of
Associated with fear and confusion
spinal cord
D. CLINICAL USES
• Induction and maintenance of anesthesia D. CLINICAL USES
• Short-term sedation
• Analgesia (CS and neuraxial anesthesia)
• Adjunct to general anesthesia to provide sedation
Ketamine vs Propofol vs Barbiturate
REVIEW QUESTIONS
Ketamine has both hypnotic and analgesic effect,
1. Patient scheduled for appendectomy has history
while propofol and barbiturate only has hypnotic
of asthma what will you give. Propofol but not
effect.
thiopental
So you can operate on a patient using ketamine only
a. Propofol or Thiopental
b. Thiopental
VI. ETOMIDATE
c. Propofol
• IV anesthetic with hypnotic but not analgesic properties
d. AOTA
and with minimal hemodynamic effects
2. Patient is scheduled for surgery, shehas
porphyria, can you give thiopental?
A. PHYSIOCHEMICAL PROPERTIES
• Carboxylates imidazole derivative a. Yes
• Has two optical isomers b. No
• Available preparation: c. Maybe
Contain only the active D-(+) isomer, contains d. I don’t know?
hypnotic properties 3. Patient came in with a stab wound on the chest
with BP of 50 palpatory, what is your drug of
B. PHARMACOKINETICS choice during induction of anesthesia.
• Continuous infusion can be safely administered a. Midazolam
Because of its minimal effects on hemodynamics b. Diazepam
and short context-sensitive half-time c. Ketamine
d. Etomidate
C. PHARMACODYNAMICS 4. Patient with adrenal insufficiency is scheduled for
• Effect on endocrine system bilateral knee amputation, what drug will you use
for general anesthesia?
CCetC
Block XX: Intravenous Anesthesia 5 of 6
MD 3
a. Etomidate
b. Propofol
c. Barbiturate
d. B or C
5. Hypertensive patient scheduled for general
surgery, what general anesthetic drug will you
use as maintenance to maintain the BP?
a. Dexmetomedine
b. Midazolam
c. Thiopental
d. Ketamine

Answers: CBDDA

REFERENCES
• Doctor’s lecture
• Audio recording
• Internet

CCetC
Block XX: Intravenous Anesthesia 6 of 6
MD 3
West Visayas State University – College of Medicine – Batch 2020
Block XX
Module 6 Intraoperative Monitoring
Lecture 5
05/ 15/ 19
Mercy Margot T. Yanson, MD, DPSA

TOPIC OUTLINE • The most critical 2 times during anesthesia are


I. Introduction INDUCTION and RECOVERY
II. The Four Basic Monitors • The aim is to achieve a smooth induction & a smooth
A. ECG
B. SpO2
recovery & a smooth intraoperative course
C. Blood Pressure Smooth induction for GA, the patient is asleep
D. Capnography during intubation
III. Individual System of Monitoring For regular intubation, the patient is awake
A. Correct Position of ETT
B. Respiratory Monitoring
C. Hemodynamic Monitoring B. ECG
D. CVS Monitors • Heart rate
E. CNS: Awareness • Rhythm (arrhythmias) best identified from lead II.
F. Temperature Monitoring
G. Monitoring after Extubation & Recovery • Ischemic changes & ST segment analysis.
Supplementary Notes • Timing of ECG monitoring is throughout the surgery:
Review Questions before induction until after extubation and recovery
References
If you see abnormal tracings, check if the
cautery is being used, a detached lead or if there
LECTURER BOOK REFERENCE OLD TRANS
is still abnormality in the tracings, confirm it with
the 12-lead ECG before giving any medications.
I. INTRODUCTION
• To understand & appreciate the value of clinical TYPES & CONNECTIONS OF ECG CABLES
monitoring • 3 – leads:
• RULE: your clinical judgement/assessment is much White: Right arm
better & much more valuable Black: Left arm
• To appreciate that modern monitors have made life Red: Left foot
much easier for us. They are present to make • 5 – leads:
monitoring easier for us. Green: Right foot
Brown: Anterior Chest
A. WHY DO WE NEED INTRAOPERATIVE
MONITORING
• To maintain the normal patient physiology &
homeostasis throughout anesthesia and surgery
• Surgery is a very stressful condition
• Most drugs used → hemodynamic instability,
myocardial depression, hypotension and arrhythmias
• Under GA → hypo or hyperventilated and may develop
hypothermia
• Blood loss → Anemia, hypotension; start blood
transfusion
• Making sure that all organs during surgery and under
anesthesia are perfused
Anesthetic agents - cardiopulmonary depressants
Patient’s response to (physio, pharma & surgical)
intervention

II. THE FOUR BASIC MONITORS


Figure 1. ECG Connections. Source: Upclass Notes
• ECG
• SpO2: arterial O2 saturation
HOW TO ATTACH ECG ELECTRODES
• Blood Pressure: NIBP (non-invasive), IBP (invasive)
• Choose a bony prominence
• Capnography – for patients under general anesthesia
• Avoid fatty and hairy areas (lead might fall off)

CCetC Group No. 30 1 of 9


MD 3 Sardua, Segovia, Shaikh
• Position them far away from each other to give higher • Can also be applied to the ear lobe (special probe)
voltage and better gain • In infants and children can be applied to 2 fingers or to
• Ensure good contact with the skin the hand (different probe than in adults)
If the electrodes are not accessible during surgery • Usually attached to the limb with the IV line (opposite
or are soaked during preparation of the patient the limb with the blood pressure cuff).
ensure that the leads are covered with plaster. If there are two blood pressure monitors (manual
• If there is no trace follow ECG cable from the patient to and NIBP), attached your pulse oximeter on the
the monitor: side which has the manual BP monitoring
Ensure good contact with the patient
Ensure proper fitting of cable connections
Ensure proper fitting of the cable to the monitor
Change monitor settings: try different leads (I, II,
III, avR, avR, avL, V1 – 6), filter, size (amplitude) of
ECG
Ensure that the ground is attached because you
will have a different tracing if it is not in place.

RULES: Figure 2. Pulse oximeter sensor. Source: Upclass Notes


• QRS beep ON must be heard at all times. NO SILENT
MONITORS. Fallacies & Inaccuracies occur when:
• Clinical judgment is much more superior to the monitor. • Misplaced on the patients finger, slipped.
Check peripheral pulsations. • Patient movement: shivering – low tracing
• Cautery → artefacts & fallacies in ECG (noise/electrical • Poor tissue perfusion (cold extremities, hypotension,
interference) → check radial (peripheral) pulsations. shock)
• Arrhythmias → check radial (peripheral) pulsations • Cardiac arrest
• Electrical interference from cautery in some monitors
C. SPO2 • Keep the sound of the pulse oximeter ON at ALL times.
• % of oxy-Hb / oxy + deoxy-Hb. • Pay attention to the sound of the pulse oximeter.
• Timing of SpO2 monitoring is throughout the surgery: • Check for cyanosis
before induction till after extubation & recovery. • If hypoxemia occurs immediately check the correct
• Waveform of pulse oximeter = plethysmography position of the probe on the patient and check the
(arterial waveform) patient’s color: nails & lips, then manage accordingly.
Indicates that the pulse oximeter is reading the Sometimes you would get a tracing of 60 but the
arterial O2 saturation. Normal - 99-100% O2 sat patient is not cyanotic, check for the position, pull
out the pulse ox and place it on another finger
PULSE OXIMETER If the patient is cyanotic with high blood pressure,
• Measures the perfusion of blood in the fingertips, check your oxygenation.
earlobe or toes. Because SpO2 measurements are averaged over a
• SpO2: arterial O2 saturation (oxygenation of the patient) few seconds to provide readings, there is some
• Heart Rate degree of delay in response time. Hypothermia, low
Check if there is cardiac arrest CO, and vasoconstriction secondary to drugs or
• Peripheral perfusion status (loss of waveform in peripheral hypoxia all increase bias, imprecision, and
hypoperfusion states: hypotension & cold extremities) response time for hypoxic episodes. This appears to
• Give an idea about the rhythm from the be more common with finger probes than with ear or
plethysmography wave (arterial waveform). (cannot forehead monitoring. Motion artifact and
identify the type of arrhythmia but can recognize if hypoperfusion are the most common causes of SpO2
irregularity is present) inaccuracy, both of which are less problematic with
• Cardiac arrest newer oximeters. Caution is advised in using pulse
• Pulse oximeter tone changes with desaturation from oximetry to not make inferences about gas exchange.
high pitched to low pitched (deep sound). So just by SpO2 should not be used to assess the adequacy of
listening to the monitor you can recognize: (1) HR (2) ventilation because SaO2 is only minimally affected
O2 saturation. by changes in PCO2 (via the Bohr Effect). In addition,
when PO2 is high, large decreases in oxygen tension
How to Attach produce only small changes (if any) in SaO2 and may
• To the finger or toe not be detected with pulse oximetry.

CCetC
Block XX: Intraoperative Monitoring 2 of 9
MD 3
D. BLOOD PRESSURE Palpation of superficial temporal artery – systolic
NIBP (NON-INVASIVE ABP MONITORING) – BP is greater than 80 mmHg
AUTOMATED
• Value: to avoid and manage extremes of blood IBP (INVASIVE ARTERIAL BLOOD PRESSURE
pressure MONITORING)
• Avoid decrease in MAP < 60 mmHg • Beat to beat monitoring of ABP via an arterial cannula
For cerebral and renal perfusion • Indicated in:
• Avoid decrease in diastolic pressure < 50 mmHg Major surgeries
For coronary perfusion During deliberate hypotensive anesthesia
• Timing of BP monitoring During the use of inotropes
Before induction until after extubation and recovery Cardiac surgery
• Frequency of measurement Surgeries involving extreme hemodynamic
Every 5 minutes changes/ instability
Every 3 minutes ─ Pheochromocytoma
─ Immediately after spinal anesthesia Repeated ABG sampling
─ Conditions of hemodynamic instability
─ During hypotensive anesthesia
Every 10 minutes
─ Monitored anesthesia care - awake patients
under local anesthesia
─ Minimal hemodynamic changes

HOW TO ATTACH BP APP


• Correct cuff size
Width of the cuff should be 1.5 times limb diameter
and should occupy at least 2/3 of the arm
• Tight cuff results to false high readings
• Loose cuff results to false low readings
• Better applied directly to the arm (remove sleeve)
Forearm – very obese individuals Figure 3. Components of an IBP Measuring System.
Calf – if arms are not accessible Source: Lecture
• Correct positioning
Hoses over the brachial artery E. CAPNOGRAPHY
• Attached to the limb opposite the IV line and pulse • Continuous CO2 measurement displayed as a
oximeter waveform sampled from the patient’s airway during
• Avoid attaching it to an arm with AV graft ventilation
Which nerve is at risk of compressive injury due to Gold Standard
frequent BP cuff use? Answer: Radial Nerve • EtCO2
A point on the capnogram
READING ERROR/FAILURE Final measurement at the endpoint of the patient’s
• Pressure line is disconnected expiration before the inspiration begins again
• Leakage from the damaged cuff Highest CO2 measurement during ventilation
• Line is compressed Phases:
• Line contains water from washing ─ Baseline: A-B
• Monitor error – cuff cannot inflate due to infant or Records expiration
neonate limits ─ Upstroke: B-C
─ Plateau: C-D
RULES ─ End-tidal: D
• Must check peripheral pulse volume from time to time Highest CO2 value
regularly every 10 minutes ─ Downstroke
Palpation of radial artery – systolic BP is greater
than 90 mmHg
Palpation of dorsalis pedis artery – systolic BP is
greater than 80 mmHg

CCetC
Block XX: Intraoperative Monitoring 3 of 9
MD 3
• After intubation, auscultation must be done in 5 areas
Right and left anterior chest wall
Right and left midaxillary
Epigastrium: to exclude esophageal intubation
• Always auscultate the chest after intubation for:
Equal air entry: to exclude endobronchial
intubation
Adventitious sounds: wheezes, crepitations,
pulmonary edema
• We must always auscultate the chest again after
Figure 4. Phases of Capnography. Source: Lecture repositioning to exclude:
Inward displacement – endobronchial intubation
Outward displacement – slippage and accidental
extubation
Sellick’s Maneuver
a technique used in endotracheal intubation to try
to reduce the risk of regurgitation. The technique
involves the application of pressure to the cricoid
cartilage at the neck, thus occluding the
esophagus which passes directly behind it.

CLINICAL MONITORING:
• Color: cyanosis: nails, lips, palms, conjunctiva
Figure 5. Normal Values in Capnography. Source: Upclass Notes
• Chest rise and fall (inflation)
V. INDIVIDUAL SYSTEM MONITORING • Vapor in the ETT
• Position of ETT • Airway pressure
• Respiratory System • Ventilator bellows
• CVS and Hemodynamic monitoring • Ventilator sound during respiratory cycle
• CNS: awareness Abnormal sounds (leakage, disconnection, high
• Temperature airway pressure, alarms)
• Monitoring after extubation and recovery • Never ignore an alarm by the ventilator!
Low airway pressure
F. CORRECT POSITION OF ETT High airway pressure
Low expired tidal volume
Apnea alarm
O2 sensor failure
Flow sensor failure

SURE SIGNS OF ENDOTRACHEAL INTUBATION


• Capnography
• Under vision

UNSURE SIGNS OF ENDOTRACHEAL INTUBATION


• Auscultation of both lungs and epigastrium – look for
the increase intensity
• Chest inflation
Figure 6. Correct position of Endotracheal Tube. Source: Lecture • Vapor in ETT
• Good compliance of bag
Table 1. Signs of Endotracheal Intubation. Source: Lecture
• O2 saturation
Sure Signs Unsure Signs
1. Capnography 1. Auscultation of both lungs &
G. RESPIRATORY MONITORS
2. Under vision epigastrium (where is max intensity?) • O2 saturation
2. Chest inflation
• Capnography EtCO2
3. Vapor in ETT
• Airway Pressure
4. Good compliance of bag
• ABG samples
5. O2 saturation

CCetC
Block XX: Intraoperative Monitoring 4 of 9
MD 3
H. HEMODYNAMIC MONITORING • It is necessary to avoid hypothermia, in order avoid
• Color: pallor complications
• Peripheral pulsations
• Capillary refilling time COMPLICATIONS OF HYPOTHERMIA
• Urine output • Cardiac arrhythmias
Ventricular tachycardia
URINE OUTPUT Cardiac arrest
• Values are indicators of • Myocardial depression
Good hydration • Delayed recovery – delays drug metabolism
Good tissue (renal) perfusion • Delayed enzymatic drug metabolism
Good renal function • Metabolic acidosis
• Indications: Tissue hypoperfusion  anaerobic glycolysis 
Lengthy surgery – longer than 4 hours lactic acidosis
Major surgery with major blood loss Hyperkalemia
C-section – monitor injury to the bladder or ureters • Coagulopathy
• Normal – 0.5-1.0 cc/kg/hr
• Note the baseline urine volume at the start of HOW TO AVOID HYPOTHERMIA
operation • Warm IV fluids
• Intermittently switch off air-conditioning especially
MANAGEMENT OF OLIGURIA OR ANURIA towards the end of surgery – increase ambient room
• Check that the line is not kinked or disconnected temperature
• Palpate the urinary bladder (suprapubic fullness) • Pediatrics – warming blanket
• Raise BP
MAP should be greater than 80 mmHg – for
adequate renal perfusion
• IV fluid challenge
• Diuretics
• Sometimes Trendelenberg position (head down)
causes decrease urine output.
Reversal of this position results in immediate flow of
urine

I. CVS MONITORS Figure 7. Pediatric warming blanket. Source: Internet


• ECG
• Blood pressure (NIBP, IBP) L. MONITORING AFTER EXTUBATION AND
• Central venous pressure RECOVERY
IV volume AFTER EXTUBATION
RV (right ventricular) function • Immediately fit the face mask on the patient (with
slight chin lift) and observe the breathing bag:
J. CNS: AWARENESS Good regular breathing with adequate tidal volume
SIGNS OF PATIENT AWARENESS transmitted to the bag
• Movement, grimacing (facial expressions) No transmission to the bag
• Pupils dilated ─ Respiratory obstruction
• Lacrimation Improve your support
• Tachycardia ─ Apnea
• HTN Attempt to awaken your patient by painful
• Sweating stimuli or jaw thrust
• Always check the concentration of the vaporizer and • BP – within 20% of the baseline
make sure that the vaporizer is not empty • SpO2 – greater than 92%
• Breathing – regular, adequate tidal volume
K. TEMPERATURE MONITORING • Muscle power
• Clinical monitoring – using hands Sustained head power for 5 seconds
• Monitors – temperature probe (nasopharyngeal, Good hand grip
esophageal) Tongue protrusion
• Avoid hypothermia – less than 36°C • Level of consciousness
• Especially in pediatrics and geriatrics Fully conscious
CCetC
Block XX: Intraoperative Monitoring 5 of 9
MD 3
─ Obeying orders/commands • Many problems with existing monitors (e.g., cost,
─ Eye opening complexity, reliability, artifacts)
─ Purposeful movement
• Patient must be able to protect his own airway ASA MONITORING GUIDELINES
• STANDARD I – Qualified anesthesia personnel shall
SUMMARY be present in the room throughout the conduct of all
• Never start induction with a missing monitor general anesthetics, regional anesthetics and
ECG monitored anesthesia care.
BP • STANDARD II – During all anesthetics, the patient’s
SpO2 oxygenation, ventilation, circulation and temperature
• Never remove any monitors before extubation and shall be continually evaluated.
recovery • Oxygenation
• Never ignore an alarm by the ventilator Inspired gas: During every administration of
• Always remember that your clinical sense and general anesthesia using an anesthesia machine,
judgment is better than any monitor the concentration of oxygen in the patient
• Always remember that there is no such as “all monitors breathing system shall be measured by an oxygen
disconnected” analyzer with a low oxygen concentration limit
• Immediately check peripheral and carotid pulsations to alarm in use.*
make sure that your patient is not arrested Blood oxygenation: During all anesthetics, a
quantitative method of assessing oxygenation such
SUPPLEMENTRY NOTES as pulse oximetry shall be employed. When the
What is normal airway pressure??? pulse oximeter is utilized, the variable pitch pulse
Normal urine output per hour tone and the low threshold alarm shall be audible
1 cc/hr for pedia to the anesthesiologist or the anesthesia care team
0.5-1 cc/hr for adult personnel.* Adequate illumination and exposure of
What is normal tidal volume? the patient are necessary to assess color.
In a healthy, young human adult, tidal volume is • Ventilation
approximately 500 mL per inspiration or 7 mL/kg Every patient receiving general anesthesia shall
of body mass have the adequacy of ventilation continually
How to calculate MAP? evaluated. Qualitative clinical signs such as chest
𝑆𝑆𝑆𝑆𝑆𝑆𝑆𝑆𝑆𝑆𝑆𝑆𝑆𝑆𝑆𝑆 𝐵𝐵𝐵𝐵+(2∗𝐷𝐷𝐷𝐷𝐷𝐷𝐷𝐷𝐷𝐷𝐷𝐷𝐷𝐷𝐷𝐷𝐷𝐷 𝐵𝐵𝐵𝐵)
𝑀𝑀𝑀𝑀𝑀𝑀 =
3
excursion, observation of the reservoir breathing
bag and auscultation of breath sounds are useful.
M. MONITORING IN THE PAST Continual monitoring for the presence of expired
• Finger on the pulse carbon dioxide shall be performed unless
• Visual monitoring of respiration and overall clinical invalidated by the nature of the patient, procedure
appearance or equipment. Quantitative monitoring of the
• Blood pressure (sometimes) volume of expired gas is strongly encouraged.
When an endotracheal tube or laryngeal mask is
HARVEY CUSHING inserted, its correct positioning must be verified by
• Not just a famous neurosurgeon … but the father of clinical assessment and by identification of carbon
anesthesia monitoring dioxide in the expired gas. Continual end-tidal
• Invented and popularized the anesthetic chart carbon dioxide analysis, in use from the time of
• Recorded both BP and HR endotracheal tube/laryngeal mask placement, until
• Emphasized the relationship between vital signs and extubation/removal or initiating transfer to a
neurosurgical events (increased intracranial pressure postoperative care location, shall be performed
leads to hypertension and bradycardia) using a quantitative method such as capnography,
capnometry or mass spectroscopy.* When
N. MONITORING IN THE PRESENT capnography or capnometry is utilized, the end
• Standardized basic monitoring requirements tidal CO2 alarm shall be audible to the
(guidelines) from the ASA (American Society of anesthesiologist or the anesthesia care team
Anesthesiologists), CAS (Canadian Anesthesiologists’ personnel.
Society) and other national societies When ventilation is controlled by a mechanical
• Many integrated monitors available ventilator, there shall be in continuous use a
• Many special purpose monitors available device that is capable of detecting disconnection of
components of the breathing system. The device

CCetC
Block XX: Intraoperative Monitoring 6 of 9
MD 3
must give an audible signal when its alarm O. ECG
threshold is exceeded.
During regional anesthesia (with no sedation) or
local anesthesia (with no sedation), the adequacy
of ventilation shall be evaluated by continual
observation of qualitative clinical signs. During
moderate or deep sedation the adequacy of
ventilation shall be evaluated by continual
observation of qualitative clinical signs and
monitoring for the presence of exhaled carbon
dioxide unless precluded or invalidated by the
nature of the patient, procedure, or equipment.
• Circulation
Every patient receiving anesthesia shall have the
electrocardiogram continuously displayed from the
beginning of anesthesia until preparing to leave the
anesthetizing location. General anesthetics: Volatile anesthetics, such as
Every patient receiving anesthesia shall have halothane or enflurane, produce arrhythmias,
arterial blood pressure and heart rate determined probably by a reentrant mechanism. Halothane also
and evaluated at least every five minutes. sensitizes the myocardium to endogenous and
Every patient receiving general anesthesia shall exogenous catecholamines. Drugs that block the
have, in addition to the above, circulatory function reuptake of norepinephrine, such as cocaine and
continually evaluated by at least one of the ketamine, can facilitate the development of
epinephrine-induced arrhythmias. In contrast, volatile
following: palpation of a pulse, auscultation of
anesthetics may have an antifibrillatory effect in
heart sounds, monitoring of a tracing of intra-
response to acute coronary occlusion and
arterial pressure, ultrasound peripheral pulse
reperfusion, at least in a canine model. Sevoflurane
monitoring, or pulse plethysmography or oximetry. may cause severe bradycardia and nodal rhythm
• Body Temperature when used in high concentrations during induction in
Every patient receiving anesthesia shall have infants, and desflurane may prolong the QTc within
temperature monitored when clinically significant the first minute of anesthesia in patients with a normal
changes in body temperature heart.

CAS MONITORING GUIDELINES Local anesthetics: Regional anesthesia by central


“The only indispensable monitor is the presence, at all neuraxial blockade, the goal of spinal or epidural
times, of a physician or an anesthesia assistant, under anesthesia, may be associated with profound, albeit
the immediate supervision of an anesthesiologist, with transient pharmacologic sympathectomy. This
phenomenon may cause parasympathetic nervous
appropriate training and experience. Mechanical and
system dominance leading to mild to very severe
electronic monitors are, at best, aids to vigilance. Such
bradyarrhythmias, especially when the blockade
devices assist the anesthesiologist to ensure the
extends to very high thoracic levels. Inadvertent
integrity of the vital organs and, in particular, the intravascular injection of a large dose of local
adequacy of tissue perfusion and oxygenation.” anesthetic may lead to asystole and cardiac arrest,
which are very difficult to treat.
BASIC MONITORING
• Cardiac: Blood Pressure, Heart Rate, ECG: Rate, ST Endotracheal intubation: This maneuver may be the
Segment (ischemia), Rhythm most common cause of arrhythmias during surgery
• Respiratory: Airway Pressure, Capnogram, Pulse and is frequently associated with hemodynamic
Oximeter, Spirometry, Visual Cues disturbances by eliciting autonomic reflexes.
• Temperature [pharyngeal, axillary, esophageal, etc.]
• Urine output (if Foley catheter has been placed)
ARTERIAL CANNULATION
• Nerve stimulator [face, forearm] (if relaxants used)
• With continuous pressure transduction and waveform
• ETT cuff pressure (keep < 20 cm H2O)
display remains the accepted reference standard for
• Auscultation (esophageal or precordial stethoscope)
blood pressure monitoring, even though it is more
• Visual surveillance of the anesthesia workspace and
costly, has the potential for more complications, and
some
requires more technical expertise to initiate and
maintain than noninvasive monitoring does. Direct

CCetC
Block XX: Intraoperative Monitoring 7 of 9
MD 3
arterial pressure monitoring should be used when PCO2 value at the end of exhalation is referred to as
moment-to-moment blood pressure changes are the end-tidal PCO2 (PEtCO2).
anticipated and rapid detection is vital. These
conditions typically apply to patients with pre-existing GENERAL ANESTHETICS
severe cardiovascular disease or hemodynamic • General anesthetics decrease the thresholds
instability or when the planned operative procedure is (triggering core temperatures) for vasoconstriction and
likely to cause large, sudden cardiovascular changes, shivering by 2°C to 3°C.
rapid blood loss, or large fluid shifts. • Anesthetic-induced impairment of thermoregulatory
control, combined with a cool operating room
Indications for Arterial Cannulation: environment, makes most patients hypothermic.
• Continuous, real-time blood pressure monitoring • The major initial cause of hypothermia in most patients
• Planned pharmacologic or mechanical cardiovascular is core-to-peripheral redistribution of body heat.
manipulation • Hypothermia during general anesthesia develops with
• Repeated blood sampling a characteristic pattern. An initial rapid decrease in
• Failure of indirect arterial blood pressure measurement core temperature is followed by a slow, linear reduction
• Supplementary diagnostic information from the arterial in core temperature. Finally, core temperature
waveform stabilizes and subsequently remains virtually
• Determination of volume responsiveness from systolic unchanged.
pressure or pulse pressure variation • Neuraxial anesthesia impairs both central and
peripheral thermoregulatory control and is associated
CAPNOGRAPHY with substantial hypothermia.
• Changes in the shape of the expired CO2 waveform in • Large randomized trials have proved that even mild
an intubated patient can provide very useful monitoring hypothermia (i.e., 1.5°C to 2.0°C) causes adverse
information. Capnometry is the measurement of outcomes, including a threefold increase in morbid
expired CO2 and has become increasingly popular as a myocardial outcomes, a threefold increase in risk for
diagnostic tool in a number of settings. It is now the wound infection, coagulopathy and need for allogeneic
confirmation method of choice in anesthesia for proper transfusion, prolonged recovery, and prolonged
placement of an endotracheal tube. CO2 concentration hospitalization.
is usually measured by infrared absorption with either a • Body temperature should be monitored in patients
mainstream or sidestream capnometer. Measurements undergoing surgery lasting longer than 30 minutes, and
can then be plotted against time or exhaled volume to core temperature should be maintained at 36°C or
generate a capnograph. Capnography has found many higher whenever possible. Forced-air warming
useful clinical applications, and in 1998 it was adopted currently offers the best combination of high efficacy,
by the American Society of Anesthesiologists as low cost, and remarkable safety.
standard care for all general anesthetics administered.
• The most commonly used type of capnograph plots RENAL FUNCTION MONITORING
Pco2 versus time. The tracing is traditionally divided • Perioperative acute renal failure (ARF), although
into an inspiratory phase and three (sometimes four) uncommon, is associated with extremely high morbidity
expiratory phases: and mortality rates.
Phase 0: inspiratory phase • The mechanism for perioperative ARF is complex and
Phase I: dead space and little or no CO2 most commonly involves multiple factors such as
Phase II: mixture of alveolar and dead space gas ischemia/reperfusion, inflammation, and toxins.
Phase III: alveolar plateau, with the peak • Notably, the importance of oliguria (<0.5 mL/kg/hr) as a
representing end-expiratory (end-tidal) CO2 predictor of AKI is very limited in the perioperative
• The waveform is conventionally subdivided into setting. Anesthesia and surgery influence normal renal
phases. During phase I, exhaled gas from the large function primarily through changes in GFR.
airways has a PCO2 of 0. Phase II is the transition Fluctuations in blood pressure have a major effect on
between airway and alveolar gas. Phase III (i.e., RBF and glomerular filtration. Anesthetic interventions,
alveolar plateau) is normally flat, but in the presence of whether involving volatile agents, intravenous drugs, or
mismatching, it has a positive slope. The downslope of regional blocks, generally reduce blood pressure and
the capnogram at the onset of inspiration is usually cardiac output, thereby diminishing RBF, leading to
referred to as phase 0, but sometimes there is a decreased glomerular filtration and urine formation.
terminal increase in the slope associated with the • Repeated direct perioperative assessment of renal
onset of airway closure (dashed line labeled IV). The hemodynamics, tubular function, or pathogenesis of
perioperative renal dysfunction is impractical;

CCetC
Block XX: Intraoperative Monitoring 8 of 9
MD 3
therefore, indirect assessments, such as serum
creatinine trends, are the best practical currently
available perioperative tool to assess renal function.
• Intraoperative urine formation depends on a number of
factors and is an insensitive and unreliable method for
assessing postoperative risk of renal dysfunction.
• Serum chemistries and urine indices such as blood
urea nitrogen, creatinine, fractional excretion of
sodium, and free water clearance are generally late
indicators of renal function deterioration and do not
enable the clinician to clearly delineate the cause of
renal failure.
• Creatinine clearance is the most sensitive and specific
clinical method for determining renal function, but it is
limited by time and measurement restrictions.

REVIEW QUESTIONS
1. Which of the ff is not a potential complication of
hypothermia?
a. Metabolic Acidosis
b. Ventricular tachycardia
c. Hypokalemia
d. Myocardial depression
2. In the management of oliguria/anuria, the MAP
should be increased to at least __ mmHg for
adequate renal perfusion?
a. 60
b. 70
c. 80
d. 90
3. Which of the ff is not an indication for Invasive
Arterial Blood Pressure (IBP) Monitoring?
a. Pheochromocytoma
b. During the use of chronotropes
c. During deliberate hypotensive anesthesia
d. Cardiac surgery
4. Most Critical Period during anesthesia
a. Induction
b. Sedation
c. Recovery
d. Both A and C
5. Which of the following with regards to BP
monitoring is true
a. Avoid Decrease in MAP of < 60 mmHG
b. Avoid Decrease in Diastolic Pressure of < 50
mm HG
c. Monitoring should be done before induction until
after extubation and recovery.
d. All of the above

Answers: CCBDD
REFERENCES
• Upclass notes
• Doctor’s lecture

CCetC
Block XX: Intraoperative Monitoring 9 of 9
MD 3
West Visayas State University – College of Medicine – Batch 2020
Block XX
Module 6 Muscle Relaxants
Lecture 6
05/ 15/ 19
Mark David Arbizo, MD

TOPIC OUTLINE IDEAL MUSCLE RELAXANT


I. Introduction • Fast onset
A. Clinical Uses • Short duration / easily reversible
B. Complications
II. Muscle Contractions
• Minimal side effects
A. Morphology • Minimal systemic complications
B. Physiology • Inactive metabolites
III. Pharmacology
A. Classifications
B. Mechanism of Action II. MUSCLE CONTRACTIONS
C. Characteristics A. MORPHOLOGY
D. Pharmacology • Corticospinal Tracts
IV. Reversal Agents Course of electrical signals
A. Mechanism of Action Ventral horns of the spinal cord
B. Characteristics
V. Sample Cases Contains the cell bodies and axons
Review Questions • Cell Bodies
References Receive and integrate information from the central
nervous system
LECTURER BOOK REFERENCE OLD TRANS
• Axons
Conduit of the impulse from the cell bodies to the
distal organ
OBJECTIVES
• Three main aims of anesthesia:
Relieve anxiety: Sedatives
Relieve pain: Analgesia
Immobility: Neuromuscular Blocking Agents
(Muscle Relaxants)

I. INTRODUCTION
A. CLINICAL USES
• Improve conditions for tracheal intubation
• Improve surgical working conditions
neurosurgery, ophthalmologic surgery (very
dangerous if patient moves)
• Facilitate mechanical ventilation at the OR/ICU

B. COMPLICATIONS
• Residual neuromuscular blockade
Airway obstruction
Attenuation of the hypoxic ventilatory response
(patient deeply inhales in response to low O2.
Muscle paralysis impairs skeletal muscles of
respiration, thereby rendering the patient unable to
inspire deeply)
Oxygen desaturation, patient compromise
• Increased incidence of awareness during general
anesthesia
you will not be able to know when the GA wears
off and the patient is conscious since the patient is
still paralyzed) Figure 1. Corticospinal Tract.

CCetC Group No. 36 1 of 8


MD 3 Tobias, Toledo, Valdez
B. PHYSIOLOGY
• Under resting conditions, the electrical potential of the
inside of a nerve cell is negative (-90 mV).
• Depolarization
Signal/stimulation
Sodium channels open and allow sodium ions to
enter the cell
Na influx is faster than K efflux
• Cascade/series of depolarization of the next segment
Propagation of an electric impulse or action
potential
1. Electrical signal depolarizes the nerve terminal
2. Opening of voltage-gated Ca2+ channels
3. Release of acetylcholine into the synaptic cleft
4. Binding of two molecules of acetylcholine to 2
alpha subunits of nAchRs
5. Conformational change in the nAchRs that opens
the ligand-gated channels
6. Na and Ca2+ influx, K efflux
7. Depolarization of the muscle membrane leading to
muscle contraction
8. Acetylcholinesterases hydrolyze acetylcholine to
acetic acid and choline
9. Repolarization

NICOTINIC ACETYLCHOLINE RECEPTORS


(NACHRS)

Figure 2. This is a cross-sectional view of a muscle bundle, which


contains the Fascicles. Inside the Fascicles, Motor Units are located.
The motor unit is the functional contractile unit and is composed of a
single myelinated alpha motor neuron and all muscle fibers that
receive innervation from this single neuron. The terminal portion of the
axon is a specialized structure, the synapse, designed for the
production and release of acetylcholine. The neuromuscular junction
(NMJ) is composed of the presynaptic motor neuron terminal, the
postsynaptic muscle surface, and a synaptic cleft (gap) that contains
the enzyme acetylcholinesterase.

Figure 4. NACHRS Subunits

• Subunits:
2 alpha
─ Binding sites for the acetylcholine,
succinylcholine and NMBA
Delta & Epsilon
─ Stabilize the closed state of the receptor
Beta
• The function of nAChR endplates depends on five
Figure 3. The NMJ has a highly ordered mechanism that converts the subunit proteins that combine to form the pentameric
electrical signal of the motor nerve (the action potential) into a
chemical signal (effected by the release of acetylcholine, which in turn unit consisting of two alpha subunits in association with
is converted into an electrical event (muscle membrane single beta, delta, and epsilon subunits.
depolarization), leading to a mechanical response (muscle
contraction). Nicotinic muscle-type Ach receptors (muscle-type • These subunits form a transmembrane pore as well as
nAChRs) are located in folds of the postsynaptic muscle membrane in the extracellular binding pockets for acetylcholine and
very high concentrations.

CCetC
Block XX: Muscle Relaxants 2 of 8
MD 3
other agonists (depolarizing neuromuscular-blocking Onset time
drugs) or antagonists (nondepolarizing neuromuscular- • Time to maximum blockade (disappearance of ST)
blocking drugs). • Directly proportional to potency (high potency, long
• In order for the conformational change to occur, 2 onset time)
molecules of Ach must attach to the 2 alpha subunits
of nAchRs. Duration
• On the other hand, an important function of the delta • Time from injection of NMBA to return to 25% twitch
and epsilon subunits is to stabilize this closed state. height
• 25% twitch height was chosen because rapid reversal
Table 1. Events Associated with Up-Regulation or Down- can normally be achieved at that level
Regulation of Nicotinic Acetylcholine Receptors (nAchRs)
nAchRs Up-Regulation nAchRs Down- Mechanism of Action (Depolarization Agent)
(Sensitive to Sch; Regulation (Resistant to 1. Binding of succinylcholine to alpha subunits of
Resistant to non- Sch; Sensitive to non- nAchRs
depolarizing agents) depolarizing agents) 2. Conformational change in the nAchRs that opens
Spinal cord injury Myasthenia gravis the ligand-gated channels
Cerebral vascular Anticholinesterase 3. Na and Ca2+ influx, K efflux
accident overdose 4. Depolarization of the muscle membrane leading to
muscle contraction
Thermal injury Organophosphate
5. Acetylcholinesterases cannot hydrolyze
poisoning
succinylcholine
Prolonged immobility 6. Neuromuscular blockade
Prolonged exposure to
neuromuscular-blocking Mechanisms of Neuromuscular Blockade
drugs • Desensitization
Multiple sclerosis Develops because Sch remains at the endplate
Guillain-Barre syndrome much longer
Depolarized post-junctional membrane cannot
III. PHARMACOLOGY respond to subsequent release of acetylcholine
Muscle paralysis starts at 75% occupancy of • Inactivation of Na channels
acetylcholine receptors Prevents the propagation of the action potential

Figure 5.
Figure 6. Neuromuscular Blocking Agents
C. NEUROMUSCULAR BLOCKING AGENTS (NMBA)
Potency
A. DEPOLARIZING NMBA
• Determined by constructing dose-response curve
Table 2. Succinylcholine
(Single Twitch Height)
Agent Succinylcholine
• ED 95 (effective dose) of a drug
Duration Ultrashort
• Half of the patients given with a particular NMBA based
Potency (ED 95 mg/kg) 0.25 – 3.0
on ED 95 will achieve 95% block of Single Twitch (ST),
Intubating Dose (mg/kg) 1.0 – 1.5
and half of the patients will achieve less than 95%
Onset Time (min) 1.0 – 1.5
block
Clinical Duration (min) 7 – 12
• Inversely proportional to the ED
Maintenance Dose N/A
• Small concentration of a drug produces muscle (mg/kg)
blockade Infusion Dose (ug/kg/min) Titrate to ST (single twitch)
muscle response

CCetC
Block XX: Muscle Relaxants 3 of 8
MD 3
Elimination Route / Plasma cholinesterase Due to depolarization or sustained muscular
Metablism (pseudocholinesterases / contractions
butyrylcholinesterases) • Increases Intracranial Pressure
Active Metabolites None Succinylcholine may increase intracranial pressure
Side Effects Myalgia, Laryngoscopy and tracheal intubation with
bradycardia/systole, inadequate anesthesia or muscle relaxation are
anaphylaxis likely to increase intracranial pressure even more
Contraindications Hyperakalemia, than succinylcholine
malignant • Increase IOP
hyperthermia, burn, Ach Intraocular pressure increases by 5 to 15 mmHg
receptor upregulation,
Recommended to avoid succinylcholine in open-
pseudocholinesterase
eye injuries
deficiency
Factors of inadequate anesthesia, elevated
Comments Fastest onset, most reliable
systemic blood pressure, and insufficient
NMBA for rapid tracheal
neuromuscular blockade during laryngoscopy and
intubation
tracheal intubation might increase intraocular
pressure more than succinylcholine
Succinylcholine
• Increases Intragastric Pressure
Only needs to occupy ½ of the acetylcholine
Lower esophageal sphincter tone is also increased
receptors to produce neuromuscular blockade, even
Thus, there is no increase in the risk of aspiration
if the other ½ is occupied by acetylcholine.
from the use of Sch
• Cardiac Dysrhythmias
Sinus bradycardia, junctional rhythm, and even
B. NON-DEPOLARIZING AGENTS
sinus arrest
Mimics the effects of acetylcholine at the cardiac
muscarinic cholinergic receptors
Tachycardia, hypertension
Mimics the effects of acetylcholine at the
autonomic nervous system ganglia
• Hyperkalemia
Channels remain open, maintaining efflux of K
from the cell
Serum K increases by approximately 0.5 mEq/L
Patients with pre-existing hyperkalemia (CKD
patients) do not have a greater increase in
Figure 6. Non-depolarizing Agents
potassium levels
Only INTERMEDIATE ACTING drugs are available
─ But the absolute level might reach the toxic
in the market
range
• Mechanism of Action (Non-Depolarizing Agent)
Severe hyperkalemia, occasionally leading to
1. Binds competitively to alpha subunits of nAchRs
cardiac arrest
2. No conformational change in the nAchRs ligand-
─ Has been described in patients with major
gated channels
denervation injuries, spinal cord transection,
3. No Na and Ca2+ influx, K efflux
peripheral denervation, stroke, trauma,
4. No depolarization of the muscle membrane
extensive burns, and prolonged immobility with
leading to muscle contraction
disease
5. Paralysis
• Muscle Pains
Common 24 to 48 hours after succinylcholine
d-TUBOCURARINE
administration
• 1st NMBA to undergo clinical investigation
Incidence is variable (1.5% to 89% of patients
• Long duration of action
receiving succinylcholine)
• Excreted in kidney and bile
Due to sustained muscular contractions
• Long duration and CVS effects have restricted its use
• Fasciculations
Prevalence is high (60% to 90%) after the rapid
injection of succinylcholine, especially in muscular
adults

CCetC
Block XX: Muscle Relaxants 4 of 8
MD 3
Table 3. Aminosteroids High doses of rocuronium can be used in place of
Agent Vecuronium Rocuronium succinylcholine
Duration Intermediate Intermediate
Potency (ED 95 0.05 0.3 Table 4.
mg/kg) Agent Pancuronium Pipecuronium
Intubating Dose 0.1 0.6 Duration Long Ultralong
(mg/kg) Potency (ED 95 0.07 0.45
Onset Time (min) 3–4 1.5 – 3 mg/kg)
Clinical Duration 25 – 50 30 – 70 Intubating Dose 0.1 0.1
(min) (mg/kg)
Maintenance 0.01 0.1 Onset Time (min) 2–4 4–6
Dose (mg/kg) Clinical Duration 60 – 120 80 – 140
Infusion Dose 1–2 5 – 12 (min)
(ug/kg/min) Maintenance 0.02 0.01
Elimination Renal (10-50%); Renal 30%; Dose (mg/kg)
Route/Metabolism Hepatic (30- Hepatic 70% Infusion Dose 20 – 40 (not N/A (not
50%) (ug/kg/min) recommended) recommended)
Active Metabolites 3-desacetyl- 17-desacetyl- Elimination Renal (40-70%); Renal (45% -
vecuronium rocuronium Route/Metabolism Hepatic (20%) 60%);
(60% potency) (minimal, 20% Unchanged
Can cause potency) 40%
prolonged Active Metabolites 3-OH- 3-desacetyl
paralysis pancuronium; metabolite (50%
Side Effects Vagal blockade Minimal 17-OH- potency)
with large doses pancuronium
Contraindications None None Side Effects Vagal block Minimal
Comments Not for Pain on (tachycardia);
prolonged ICU injection, catecholamine
administration reversible by release
(myopathy), Sugammadex Contraindications Short surgical Short surgical
reversible by procedures procedures
Sugammadex Comments Significant Can be
accumulation; reversed by
VECURONIUM prone to residual Sugammadex
• Onset is slower blockade
• Duration of action is governed by redistribution (hepatic
uptake) PANCURONIUM
• Potent metabolite 3-desacetyl-vecuronium (60% active • Slow onset limits its usefulness in Rapid sequence
of the parent drug) intubation
• Prolonged paralysis, myopathy • Long-acting drug
• No CVS effects after average doses, vagal blockade • Clearance is decreased in renal and hepatic failure
after large doses • Metabolized to 3-OH-pancuronium (50% NMB activity
• No histamine release of the parent compound)
Residual blockade
ROCURONIUM • Vagolytic effect at the post-ganglionic nerve terminals
• Fastest onset among Aminosteroidal NMBA Result of blocking the muscarinic receptors
Drug of choice for rapid sequence intubation if Tachycardia, hypertension, increased CO
Succinylcholine is contraindicated • No histamine release
• Duration of action is governed by redistribution (hepatic • Not available in the market
uptake)
High hepatic uptake decreases the plasma
concentration after an injection
• No hemodynamic changes
• No histamine release
• Major drawback is the long duration of action

CCetC
Block XX: Muscle Relaxants 5 of 8
MD 3
Table 5. Benzylisoquinolinium Increased protein binding
Agent Atracurium Cisatracurium Upregulation of receptors, causing resistance of
Duration Intermediate Intermediate the muscle end-plate
Potency (ED 95 0.25 0.05
mg/kg) CISATRACURIUM
Intubating Dose 0.5 0.15 – 0.2 • Potent isomer of Atracurium
(mg/kg) • Longer onset time
Onset Time (min) 3-5 4–7 In order to shorten onset time, intubating dose is
Clinical Duration 30 - 45 35 – 50 increased
(min) Dose is still well below the threshold of histamine
Maintenance 0.1 0.01 release
Dose (mg/kg) Duration of action is prolonged
Infusion Dose 10 - 20 1–3
No adjustment in dose in the elderly and pediatric
(ug/kg/min)
patients
Elimination Renal (10%); Hofmann
• Dose must be increased in burn patients
Route/Metabolism Hofmann (30%); (30%); Ester
Ester Hydrolysis Hydrolysis Increased protein binding
(60%) (60%) Upregulation of receptors, causing resistance of
Active Metabolites none None the muscle end-plate
Side Effects Histamine None;
release; Histamine Table 6. Comparison of Mivacurium and Doxacurium
Laudanosine and release at high Agent Mivacurium Doxacurium
Acrylates dose
production Duration Short Ultralong
Contraindications Hemodynamically None Potency (ED 95 0.08 0.02 – 0.033
unstable patients mg/kg)
due to Histamine Intubating Dose 0.2 0.05 – 0.08
release (mg/kg)
Comments Organ Minimal Onset Time (min) 3-4 3 – 10
independent Histamine,
Laudanosine, Clinical Duration 15 - 20 80 – 160
and Acrylates (min)
levels Maintenance Dose 0.1 0.01
(mg/kg)
ATRACURIUM / CISATRACURIUM Infusion Dose 5-8 n/a (not
• Organ independent metabolism (ug/kg/min) recommended)
• Degraded via 2 metabolic pathways: Elimination Route/ Plasma Hepatic/Renal
Hofmann reaction (1/3) Metabolism Cholinesterase
─ Non-enzymatic degradation dependent on Active Metabolites none None
Temperature and pH Side Effects Histamine release None
Ester Hydrolysis (2/3) Contraindication Pseudocholinester None
• End products: ase deficiency
Comments Reversal by No accumulation;
Laudanosine
Cholinesterase no cardiac effects
─ Causing seizures in animals inhibitors at high doses
─ No deleterious effects have been established in
humans MIVACURIUM
Acrylates • Short duration of action
─ Have been shown to inhibit human cell Hydrolyzed by Plasma Cholinesterases
production in vitro • Onset time is long
─ Requires high concentration and exposure to • Associated with histamine release
obtain clinical effects • In Burn patients:
can be given safely to patients with renal disease Upregulation of receptors
Decreased number of Plasma cholinesterases
ATRACURIUM Net effect is either normal or enhanced blockade
• Associated with histamine release • Not available in the Philippines
use cautiously in asthmatic patients
• No adjustment in dose in the elderly and pediatric DOXACURIUM
patients • Potent
• Dose must be increased in burn patients
CCetC
Block XX: Muscle Relaxants 6 of 8
MD 3
• Long-acting
• Limited clinical use due to slow onset and long duration
of action
• Not available in the Philippines

IV. REVERSAL AGENTS


• Unimpaired muscle strength Figure 7.
Breathe normally
• Rapidly binds all available unbound (free) rocuronium
Cough forcefully
molecules
Swallow
• Concentration gradient that favors the movement of
Maintain patent airway
steroidal NMBD molecules from the NMJ back into the
In most circumstances, all efforts should be made to
plasma
ensure that the patient leaves the operating room
• This removal of NMBD from the NMJ results in a fast
with unimpaired muscle strength. Specifically,
recovery of neuromuscular function
respiratory and upper airway muscles must function
normally so the patient can breathe, cough, swallow
V. SAMPLE CASES
secretions, and keep his or her airway patent. 1. 23/F, Asthmatic patient, with a known allergy to
seafoods. Patient was referred to Anesthesiologist for
NEOSTIGMINE Laparoscopic Surgery under General Anesthesia.
Acetylcholinesterase inhibitor What is your muscle relaxant of choice?
DO NOT GIVE THIS FOR SUCCINYLCHOLINE! A. Succinylcholine
• Inhibits Acetylcholinesterases at all Cholinergic B. Atracurium
synapses (both Nicotinic and Muscarinic) C. Rocuronium
• Increases Acetylcholine concentration at the D. Cisatracurium
neuromuscular junction (Nicotinic synapse) E. Vecuronium
• Increases the Acetylcholine concentration at the 2. 44/M patient with abdominal obstruction. Patient was
Muscarinic synapses referred to Anesthesiologist for Exploratory Laparotomy
• Side Effects: under General Anesthesia-Rapid Sequence
Bradycardia / Bradyarrhythmias Induction/Intubation. What is your muscle relaxant of
Increased Salivation choice?
Increased bowel motility A. Succinylcholine
Increase in airway resistance (Bronchoconstriction) B. Atracurium
• EFFECT ON SUCCINYLCHOLINE C. Rocuronium
Inhibition of Plasma Cholinesterases D. Cisatracurium
─ Sustained Succinylcholine effect E. Vecuronium
─ Prolonged duration of action 3. 55/M patient with open eye injury was referred to
use with caution in asthmatics because it increases Anesthesiologist for Emergency Eye Surgery under
airway resistance General Anesthesia-Rapid Sequence
Induction/Intubation because he had a full meal 2 prior
SUGAMMADEX the injury. What is your muscle relaxant of choice?
• FDA-approved Gamma-Cyclodectrin A. Succinylcholine
• Selective relaxant binding agent (SRBA)-selective for B. Atracurium
aminosteroidal agents C. Rocuronium
• eight-membered ring with a central cavity that perfectly D. Cisatracurium
encapsulates the steroid nucleus of steroidal E. Vecuronium
intermediate-acting NMBAs (rocuronium>vecuronium
>>pancuronium/pipecuronium) 4. 30/M, no co-morbid patient, stab wound victim was
• has no affinity for any of the other depolarizing or referred to Anesthesiologist for Emergency Exploratory
nondepolarizing NMBAs Laparotomy under General Anesthesia-Rapid Sequence
Because Sugammadex does not bind to any known Induction/Intubation. Minutes prior to admistration of
receptor, it is devoid of major cardiovascular or other muscle relaxant, tachycardia and BP spikes were noted.
side effects Patient is still moving despite giving Atracurium. What do
you think is happening?

CCetC
Block XX: Muscle Relaxants 7 of 8
MD 3
Answers: 1F, 2A, 3C, 4rocuronium, 5D

REFERENCES
• Upclass notes
• Doctor’s lecture /ppt

The resident injected the wrong vial since dopamine vial


and the atracurium vial looks very similar.

What Muscle Relaxant will you give for the patient in


order to intubate?
Answer: None
If you are not sure that you can intubate the patient
intame before hypoxia sinks in, you can just give opiods
and intubate the patient awake to make sure.

REVIEW QUESTIONS
1. T/F. Under resting conditions, the electrical potential
of t the inside of a nerve cell is positive (+90 mV).
2. An ideal muscle relaxant is:
a. Fast onset
b. Irreversible
c. Several side effects
d. Multiple systemic complications
e. All of the above
3. An example of a depolarizing neuromuscular
blocking agent is:
a. Mivacurium
b. Doxacurium
c. Succinylcholine
d. Acetylcholine
4. What has the fastest onset among aminosteroidal
NMBA?
5. Effect of neostigmine on succinylcholine except:
a. Enhances plasma cholinesterases, thus
reducing the succinylcholine effect
b. Enhances plasma cholinesterases which
would prolong the duration of action
c. Inhibits plasma cholinesterases, thus
reducing the succinylcholine effect
d. Inhibits plasma cholinesterases which would
prolong the duration of action

CCetC
Block XX: Muscle Relaxants 8 of 8
MD 3
West Visayas State University – College of Medicine – Batch 2020
Block XX
Module 6 Airway Management
Lecture 7
05/ 16/ 19
Dr. John Emmanuel S. Reyes

TOPIC OUTLINE II. AIRWAY ANATOMY


I. Indications
II. Airway Anatomy
III. Airway Management
A. History
B. PE
IV. Airway Management Techniques
A. Opening the Airway
B. Face Mask Ventilation
C. Laryngeal Mark Airway
D. Endotracheal Intubation
E. Rigid Laryngoscopy
V. Complications
A. During Laryngoscopy and Intubation
B. Tube in place
C. Extubation
Review Questions
References
Appendices

LECTURER BOOK REFERENCE OLD TRANS

Figure 1. Airway anatomy

I. INDICATIONS A. NASAL CAVITY


In emergency medicine Provides for the passage, filtration, humidification, and
Acute respiratory failure warming of inhaled air
Inadequate oxygenation or ventilation Anatomic borders:
Airway protection in a patient with depressed Superior: cribriform plate
mental status Inferior: hard palate
In preoperative setting Lateral: turbinates
Patients receiving general anesthesia The ophthalmic and maxillary divisions of the
Surgery involving or adjacent to the airway trigeminal nerve (cranial nerve V) provide innervation
Unconscious patients requiring airway protection to the nasal mucosa as the anterior ethmoidal,
Surgery involving unusual position nasopalatine, and sphenopalatine nerves.
Less frequently:
Short- term hyperventilation to manage increased B. ORAL CAVITY
intracranial pressure Includes dentition, anterior two-thirds of the tongue,
To manage copious secretions or bleeding in the floor of the mouth, and undersurface of the hard and
airway soft palates
Mandible
Primary bony structural support of the mouth
Horseshoe-shaped structure with two vertical rami
that articulate with the temporal bones of the
cranium
Temporomandibular joints

C. PHARYNX
U-shaped fibromuscular tube that is divided into 3
areas:
Nasopharynx
─ posterior to the nasal cavity and serves as an
air conduit
─ separated from the oropharynx by the soft
palate

CCetC Group No. 34 1 of 15


MD 3 Tan, Tansinco, Tañedo
Oropharynx Area above vocal cord
─ main passage of the aerodigestive tract Recurrent laryngeal nerve
─ level of the 2nd and 3rd cervical vertebrae Motor
─ Extends from soft palate to the level of the ─ Intrinsic muscle
hyoid bone inferiorly Sensory
─ Epiglottis – demarcates border between ─ Area below vocal cord
oropharynx and hypopharynx Innervation is by way of cranial nerves IX
─ Tongue - prominent cause of resistance; (glossopharyngeal) and X (vagus).
relaxation of genioglossus during anesthesia The vagus nerve provides sensation to the
Hypopharynx/ Laryngopharynx hypopharynx through the internal branches of the
─ continuation of the aerodigestive tract; extends superior laryngeal nerves.
from the epiglottis to the lower border of the Airway resistance may be increased by prominent
cricoid cartilage lymphoid tissue in the nasopharynx. The tongue is
─ begins at the level of the hyoid bone the predominant cause of resistance in the
─ its inferior end opens into the esophagus oropharynx. Obstruction by the tongue is increased
posteriorly and to the larynx anteriorly by relaxation of the genioglossus muscle during
anesthesia.
D. LARYNX
III. AIRWAY MANAGEMENT
Steps:
Airway assessment
Preparation and equipment check
Patient positioning
Preoxygenation
Bag and mask ventilation
Intubation
Confirmation of endotracheal tube placement
Intraoperative management and troubleshooting
Extubation

Figure 2. Anatomy of larynx A. AIRWAY ASSESSMENT


HISTORY
It connects the inferior pharynx with the trachea. Signs of Upper Airway obstruction/Airway distress:
Functions: Hoarse voice
It is a valve that separates the trachea from the Dyspnea
upper aerodigestive tract. Stridor
It helps maintain a patent airway. Suprasternal/supraclavicular/intercostal space
It is used for vocalization. retractions
It is located between the 3rd and 6th cervical vertebra. Tracheal tug
Adults: usually on the 6th cervical vertebra Restlessness
Preterm: 3rd cervical vertebra Cyanosis
Term: 4th cervical vertebra
The anteroposterior lengths of the vocal cords are 23 PHYSICAL EXAMINATION
mm & 17 mm in males and females respectively. Ease of open airway and maintenance
Larger in males (deeper voice) Ease of tracheal intubation
The vocal cords are 6 to 9 mm in the transverse plane, Teeth
but can expand to 12 mm. Neck movement
There are nine cartilages at the larynx. Three are sets Intubation hazards
of pairs and three exist singly. Signs of airway distress

E. UPPER AIRWAY INNERVATION


Vagus Nerve
Superior laryngeal nerve
─ External branch (motor)
Cricothyroid muscle
─ Internal branch (sensory)

CCetC
Block XX: Airway Management 2 of 15
MD 3
Table 1. Components of the airway physical examination

Figure 3. Mallampati classification of the oral cavity

Higher classification, more difficult airway


management
Class IV: hard palate is visible

Cormack & Lehane Score


Laryngoscopic view

Anatomic characteristics associated with diffcult


airway management
Short muscular neck
Receding mandible
Protruding maxillary incisors
Long high arched palate
Inability to visualize uvula Figure 4. Cormack and Lehane scoring of the larynx
Limited temporomandibular joint mobility
Grade I - visualization of the entire laryngeal aperture
Interincisor distance <2 FB or 3 cm
Grade II - visualization of just the posterior portion of
Normal: 4.6 cm
the laryngeal aperture
Difficult: >3.8 cm
Grade III - visualization of only the epiglottis
Grade IV - visualization of just the soft palate
Assessment of airway associated with difficult airway
management
Mallampati classification Table 2. Cornack and Lehane grading description
Likelihood
> Class III Appropriate
Grade Description of difficult
Atlano-occipital joint extension frequency
intubation
< 35o
1 Full view of 68% <1%
Hyoid mental distance
glottis
< 3 cm or FB
2a Partial view of 24% 4.3%
Thyromental distance glottis
< 6 cm or FB 2b Only posterior 6.5% 67.4%
Horizontal length of mandible extremity of
< 9 cm glottis seen of
Sternomental distance only arytenoid
< 12 cm cartilages
3 Only epiglottis 1.2% 87.5%
Mallampati Classification seen, none of
Examines the size of the tongue in relation to the oral glottis seen
cavity 4 Neither glottis Very rare Very likely
or epiglottis
seen

SUPPLEMENTARY
Based on common airway indexes measurement:
Thyromental Distance
─ mentum to thyroid notch in a neck-extended
position
─ helps identify whether the laryngeal axis falls
with the pharyngeal axis

CCetC
Block XX: Airway Management 3 of 15
MD 3
─ If <6-7 cm or <3 fingerbreadths ~ poor
laryngoscopic view
laryngeal axis makes an acute angle with the
pharyngeal axis making intubation difficult.
Interincisor gap
─ interincisor distance with the mouth fully opened
─ < 3 cm: poor laryngoscopic view
Atlanto-Occipital Extension/ Neck Mobility
Flexion of neck
─ elevating head ~ 10 cm aligns laryngeal and
Figure 7. The head is resting on a pad (which flexes the neck on the
pharyngeal axes chest) with concomitant extension of the head on the neck, which
─ to obtain line of vision during laryngoscopy brings all three axes into alignment (sniffing position).
Extension of the head on the atlanto-occipital joint
is important for aligning the oral and pharyngeal
axes
to obtain a line of vision during direct laryngoscopy.
Atlanto-occipital extension is quantified by the angle
traversed by the occlusal surface of the maxillary
teeth when the head is fully extended from the
neutral position.
More than 30% limitation of atlanto-occipital joint
extension from a norm of 35 degrees, or less than Figure 8. Extension of the head on the neck without concomitant
elevation of the head.
80 degrees of extension/flexion, is associated with
an increased incidence of difficult tracheal Submandibular compliance
intubation. ─ area in which pharyngeal soft tissue must be
displaced to obtain line of vision during
laryngoscopy
─ Ludwig’s angina, tumors, radiation scarring,
burns, and previous neck surgery are
conditions that can decrease submandibular
compliance
Body Habitus
─ Obesity; BMI > 30kg/m2
─ Neck circumference > 27 in
Figure 5. The head is in a neutral position with a marked degree of
nonalignment of the OA, PA, and LA. - END OF SUPPLEMENTARY -

IV. AIRWAY MANAGEMENT TECHNIQUES


A. OPENING THE AIRWAY
Non-equipment
Head tilt
Chin lift
Jaw thrust
With equipment
Oral/nasopharyngeal airway
Endotracheal intubation
Laryngeal mask airway
Figure 6. The head is resting on a large pad that flexes the neck on Cricothyrotomy
the chest and the LA with the PA.
Tracheostomy

B. FACE MASK VENTILATION


ONE-HANDED FACE MASK TECHNIQUE
THUMB and INDEX FINGER
exert downward pressure on the mask body to hold
the mask against the face

CCetC
Block XX: Airway Management 4 of 15
MD 3
MIDDLE FINGER
grasp the mandible to facilitate extension of the
atlanto-occipital joint
LITTLE FINGER
under the angle of the jaw and used to thrust the
jaw anteriorly

Figure 12. Sniffing position

Sniffing position allows alignment of all axes to


facilitate easier airway management

Figure 9. One- handed face mask technique

TWO-HANDED FACE MASK TECHNIQUE


For patients with difficult airway
THUMBS
Figure 13. Positioning for morbidly obese patients
hold the mask down
FINGERTIPS or KNUCKLES Patients with morbid obesity should be positioned on a
displace the jaw forward 30˚ upward ramp – avoid rapid deoxygenation

SUPPLEMENTARY
Bag-Valve Mask
Used to deliver oxygen
Approximately 1600 mL
Usually precedes intubation to oxygenate the patient
Avoided in rapid sequence inductions – causes
stomach inflation and increases risk of aspiration

Airway Adjuncts

Oropharyngeal Airway
Used to create an air passage by displacing the tongue
Figure 10. Two- handed face mask technique
from the posterior pharyngeal wall
Used when mandibular muscles have relaxed
Inserted between tongue and posterior pharyngeal wall
In awake patients, oral airway is less tolerated
because the gag reflex is initiated.

Nasopharyngeal Airway
Used if with clenched jaws which require relief of soft
tissue obstruction
Obstruction coexists with a preserved gag reflex.
Contraindicated in patients with coagulation and
Figure 11. Airway axes alignment and exposure of the glottic opening platelet abnormalities, basilar skull fractures
because it may cause bleeding.

CCetC
Block XX: Airway Management 5 of 15
MD 3
─ DAWD – function of the MVO2 and the oxygen
reservoir of the function residual capacity
(FRC), approximately 30 to 35 mL/kg.

Several techniques of breathing oxygen at the


beginning of anesthesia exist with the goal of
achieving an end tidal oxygen level above 90%:
Three minutes of tidal volume breathing of 100%
O2 – has been found to be superior to four deep
breaths in 30 seconds.
Eight deep breaths in 60 seconds – equivalent to
breathing 100% oxygen for 3 minutes. A patient
Figure 14. Oropharyngeal and nasopharyngeal airway adjuncts
who is not obese and has no pulmonary
dysfunction can maintain a SaO2 more than 90%
Facilitates delivery of oxygen or anesthetic gas by
for approximately 8.5 minutes.
creating an airtight seal around the patient’s face.
Effective mask ventilation requires:
Obesity can severely decrease the DAWD.
Gas tight mask fit
Breathing 100% oxygen in a 25- to 90degree head-
Patent airway
up position as well as 10 cm of continuous positive
Predictors of difficult face mask ventilation:
airway pressure (CPAP) followed by pressure mode
>55 yrs.
ventilation (peak pressure 14 cm, positive end
BMI > 26 kg/m2
expiratory pressure [PEEP] 10 cm) increase the
History of snoring
DAWD (45 to 50 sec above a baseline of 2 to 2.5
Beard
min) in obese patients by decreasing atelectasis and
Lack of teeth
improving ventilation perfusion matching.
Generate positive pressure ventilation by squeezing
The facemask should be held to the patient’s face
the breathing bag
with the fingers of the anesthesia provider’s left
Up to 20 cm of H20 – to avoid insufflation of the
hand lifting the mandible (chin lift, jaw thrust) to the
stomach
facemask.
Sniffing position
Pressure on the submandibular soft tissue should
allows alignment of the oral and pharyngeal axes
be avoided because it can cause airway
Jaw thrust
obstruction.
for patients with suspected cervical spine injury
The anesthesia provider’s left thumb and index
finger apply counter pressure on the facemask.
FACEMASK CHARACTERISTICS
Displacement of the mandible, atlanto-occipital
variety of sizes
joint extension, chin lift, and jaw thrust combine to
clear masks – allows visualization of fogging, skin
maximize the pharyngeal space. Differential
color, and signs of regurgitation.
application of pressure with individual fingers can
should fit over the bridge of the nose with the
improve the seal attained with the facemask.
upper border aligned with the pupils. The sides
The anesthesia provider’s right hand is used to
should seal just lateral to the nasolabial folds.
generate positive pressure by compressing the
bottom of the facemask should seat between the
reservoir bag of the anesthesia breathing circuit.
lower lip and the chin.
Ventilating pressure should be less than 20 cm
Most facemasks come with a hooked rim around
H2O to avoid insufflation of the stomach.
the 15- to 22-mm fitting that attaches to the
anesthesia breathing circuit. This rim allows straps
In instances in which the airway cannot be
to be used to hold the facemask in place when a
maintained with only one hand on the facemask, a
patient is breathing spontaneously or to improve
two- or three-handed facemask technique can be
the seal during mask ventilation.
used. If not trained in airway management, the
assistant can help by squeezing the reservoir bag
TECHNIQUE
while the anesthesia provider uses the right hand to
Prior to induction of anesthesia, breathing 100%
mirror the hand position of the left and improve the
O2 allows for a prolonged duration of apnea
facemask seal. When the second person is skilled in
without desaturation (DAWD).
airway management, the primary provider maintains
─ 3 mL/kg/min – minute ventilation for oxygen
(MVO2) in an adult with an ideal body weight
CCetC
Block XX: Airway Management 6 of 15
MD 3
the standard hand position and the assistant uses ADVANTAGES DISADVANTAGES
both hands to generate an optimal seal VS. Less invasive Increased risk of GI
tracheal aspiration
- END OF SUPPLEMENTARY - intubation Very useful in Less safe in prone
difficult intubations or jack-knife
C. LARYNGEAL MASK AIRWAY (LMA) positions
An LMA consists of a wide-bore tube whose
Less tooth and Limits maximum
proximal end connects to a breathing circuit with a
laryngeal trauma PPV
standard 15-mm connector, and whose distal end is
Less Less secure airway
attached to an elliptical cuff that can be inflated
laryngospasm and
through a pilot tube.
bronchospasm
SUPPLEMENTARY Does not require Greater risk of gas
Four types of LMAs are commonly used: muscle relaxation leak and pollution
Reusable LMA – most common Does not require Can cause gastric
Improved disposable LMA neck mobility distention
ProSeal LMA
Fastrach LMA - END OF SUPPLEMENTARY -

D. ENDOTRACHEAL INTUBATION
INDICATIONS:
Provide a patent airway
Deliver positive pressure ventilation
Protection of the respiratory tract
Maintenance of adequate oxygenation
All situations involving neuromuscular paralysis
Surgical procedures involving the head and neck or in
non-supine positions
Surgical procedures involving the cranium, thorax, or
abdomen
Route for emergency drug during cardiac arrest
Figure 15. LMA positioning and insertion
Table 4. Orotracheal Tube Size Guidelines
LMA is placed above the trachea AGE Internal Diameter Length (cm)
(mm)2
Advantages and Disadvantages of laryngeal mask Full Term 3.5 12
airway vs. other techniques
Child 4+ Age/4 12 + Age/2
Table 3. Advantages and disadvantages of laryngeal mask Adult
airway vs other techniques. Female 7.0 – 7.5 20 – 23
ADVANTAGES DISADVANTAGES Male 7.5 – 8.0 21 – 24
VS. Hands-free More invasive
face operation SIGNS OF TRACHEAL INTUBATION
mask Respiratory gas moisture disappearing on inhalation
Better seal in More risk of airway
and reappearing on exhalation
bearded patients trauma
Chest rise and fall
Less cumbersome Requires new skill
No gastric distention
in ENT surgery
ICS filling out during inspiration
Often easier to Deeper anesthesia
Reservoir bag having the appropriate compliance
maintain airway required
Breath sounds over chest wall
Protects against Requires some TMK No breath sounds over stomach
airway secretions mobility Hearing air exit from ET when chest is compressed
Less facial nerve N2O diffusion into cuff Large spontaneous exhaled tidal volume
and eye trauma “More reliable sign”
CO2 excretion waveform (Capnograph)
─ Gold Standard
CCetC
Block XX: Airway Management 7 of 15
MD 3
“Most reliable signs” Table 5. Available ET tube sizes and lengths for pediatric
ET visualized between vocal cords patients
Fiberoptic visualization of cartilaginous rings of the AGE Internal External French Distance
trachea and tracheal carina Diameter Diameter Unit Inserted
(mm)2 (mm)2 from Lips
TECHNIQUES FOR ROUTINE INTUBATION for Tip
Preoxygenation Placement
Administration of induction agent in the
Adequate mask ventilation Midtrachea
Administration of neuromuscular blocking agent (cm)2
Continue mask ventilation Premature 2.5 3.3 10 10
Intubation Term 3.0 4.0-4.2 12 11
Confirm ET in trachea newborn
1-6 mos 3.5 4.7-4.8 14 11
TECHNIQUES FOR “RAPID SEQUENCE” (CRASH) 6-12 mos 4.0 5.3-5.6 16 12
INDUCTION AND INTUBATION 2 yr 4.5 6.0-6.3 18 13
Preoxygenation 4 yr 5.0 6.7-7.0 20 14
5 mins / 8 deep breaths 6 yr 5.5 7.3-7.6 22 15-16
Administration of induction agent and neuromuscular 8 yr 6.0 8.0-8.2 24 16-17
blocking agent 10 yr 6.5 8.7-9.3 26 17-18
Cricoid pressure (Sellick’s maneuver)
12 yr 7.0 9.3-10 28-30 18-22
NO mask ventilation
≥14 yr 7.0 9.3-10 28-30 20-24
Intubation
(females)
Check ET in trachea
8.0 10.7-11.3 32-34
Release cricoid pressure
(males)
SUPPLEMENTARY
Average adult ET size:
Equipment:
Female: 6.5 to 7.5mm
Properly sized endotracheal tube – most often made of
Male: 7.5 to 8.5mm
clear inert polyphelene plastic
cuff prevents aspiration and facilitate positive
Laryngoscope
pressure ventilation
used to evaluate the larynx and facilitate intubation of
Laryngoscope
the trachea.
Functioning suction catheter
the choice of laryngoscope depends on physician’s
Appropriate anesthetic drugs
preference and patient’s airway anatomy.
Equipment providing positive pressure ventilation of the
lungs with oxygen

Figure 16. Parts of an endotracheal tube

CCetC
Block XX: Airway Management 8 of 15
MD 3
Hold the laryngoscope in the left hand with the 5th
finger close to the hinge and the handle pointing right
of the sagittal plane, introduce the blade along the right
margin of the tongue
Rotate the laryngoscope into a parasagittal plane to
displace the tongue to the left
Advance the blade until the right tonsillar fossa is
identified
Medial to the tonsillar fossa, locate the epiglottis
Maintaining the wrist rigidity to avoid using the
maxillary teeth as the fulcrum, expose the glottis by
Figure 17. Parts of a laryngoscope and types
displacing the tongue and epiglottis in the
anteroinferior direction.
Table 6. Comparison of Macintosh and Miller laryngoscopes
If necessary, bring the glottis into better view using the
Macintosh (curved) Miller (straight)
right hand to apply backward, upward, and rightward
Less trauma to the teeth Better exposure of glottis
opening pressure on the thyroid cartilage.
More room for passage of Adult: Miller 2 or 3
the tube
Less bruising of the
epiglottis
Adult: Macintosh 3 or 4

Positioning
Patient’s face is near the level of the xiphoid cartilage
Elevate patient’s head 8-10 cm with pads under the
occiput and extension of the head at the atlanto-
occipital joint (sniffing position)

Figure 20. Laryngoscope insertion


Figure 18. Positioning for laryngoscope insertion
Insert the tracheal tube from the right, keeping it from
Patient’s mouth may be manually opened by counter blocking the view of the glottis until the last moment
pressure of the right thumb on the mandibular teeth possible.
and right index finger on the maxillary teeth Advance the tube until the proximal margin of its cuff
Patient’s lower lip rolled away with the left index finger lies 1-2 cm beyond the vocal cords.
to prevent bruising by the laryngoscope blade

Figure 19. Hand maneuvers in laryngoscope insertion

CCetC
Block XX: Airway Management 9 of 15
MD 3
Pulse oximeter, Capnograph, ECG
Local anesthetic infiltration / spray

V. COMPLICATIONS
A. DURING LARYNGOSCOPY AND INTUBATION
Physiologic reflexes
Hypertension (HPN)
Arrythmia
Intracranial HPN
Intraocular HPN
Bronchospasm
Tube malfunction
Cuff perforation
Figure 21. laryngoscope parts and insertion
B. WHILE TUBE IS IN PLACE
Care after ET insertion
Malpositioning
Confirming tracheal intubation
Unintentional extubation
Bilateral breath sounds in the chest (apical and
Endobronchial intubation
mid-axillary) – if asymmetrical, it means the tube is
ET cuff malposition
inserted too low, thus we should adjust the length
Airway trauma
of the inserted tube.
Mucosal inflammation
Also auscultate the epigastric area to rule out
Excruciation of nose
esophageal intubation
Tube malfunction
Ignition
Obstruction/kinking
Aspiration

C. FOLLOWING EXTUBATION
Airway trauma
Edema
Stenosis
Hoarseness/sore throat
Laryngeal trauma/malfunction
Physiologic reflexes
Laryngospasm
Aspiration

SUPPLEMENTARY
A. HISTORY
Figure 22. Areas to be auscultated to confirm tracheal intubation Signs or symptoms related to the airway should be
elicited:
Maintaining tracheal intubation Snoring
Tube is taped or tied to secure its position Chipped teeth
Removing tracheal tube Changes in voice
Dysphagia
- END OF SUPPLEMENTARY Stridor
Bleeding
E. PREPARATION FOR RIGID LARYNGOSCOPY Cervical spine pain or limited ROM
Suction machine
Temporomandibular joint pain/dysfunction
Airway
Previous problem with airway management
Laryngoscope
Endotracheal tube
B. ANATOMIC DIFFERENCES BETWEEN
Stylet PEDIATRIC AND ADULT AIRWAYS
Anesthetic machine / Breathing system / Self inflating Preterm infants’ cricoid cartilages are usually located at
bags the 3rd cervical vertebrae. Term infants are at the 4th,
Monitoring and adults are at the sixth.

CCetC
Block XX: Airway Management 10 of 15
MD 3
An infant’s airway is funnel-shaped, while an adult’s is position (Mallampati class
flask-shaped. higher than II)
Pediatric patients are considered to have difficult Shape of the palate Highly arched or very
airways due to their existing features. narrow
These anatomic features make neonates and most Compliance of the Stiff, indurated, occupied
young infants obligate nasal breathers until about 5 mandibular space by a mass, or nonresilient
months of age. One millimeter of edema will have a Thyromental distance Less than three
proportionately greater effect in children because of fingerbreadths
their smaller tracheal diameters. Length of the neck Short
The infant’s larynx: Thickness of the neck Thick
is more anterior Range of motion of the Patient cannot touch the tip
the epiglottis is longer, stiffer and further away from head and neck of the chin to the chest or
the anterior pharyngeal wall cannot extend the neck
the narrowest portion is at the cricoid cartilage, not
the vocal cords Table 8. Congenital syndromes associated with difficult
the tongue is relatively larger endotracheal intubation
Vocal cords: narrowest portion for adults, the tongue SYNDROME DESCRIPTION
is relatively larger. Trisomy 21 Large tongue, small mouth
These characteristics put children under the make laryngoscopy difficult
classification of patients with difficult airway (difficulty Small subglottic diameter
in ventilation and intubation). possible
Laryngospasm is common
Goldenhar Mandibular hypoplasia and
(oculoauriculovertebral cervical spine abnormality
anomalies) make laryngoscopy difficult
Klippel-Feil Neck rigidity because of
cervical vertebral fusion
Pierre Robin Small mouth, large tongue,
mandibular anomaly
Treacher Collins Laryngoscopy is difficult
(mandibular dysostosis)
Turner High likelihood of difficult
tracheal intubation

D. ASSESSMENT AND PREDICTABILITY OF


Figure 23. Image showing the relative positions of the airway DIFFERENT MASK VENTILATIONS
components in an adult and infant
Table 7. Criteria for difficult mask ventilation and
C. AIRWAY ASSESSMENT independent risk factors
Table 7. Components of preoperative airway physical
examination
AIRWAY EXAMINATION NONREASSURING
COMPONENT FINDINGS
Length of upper incisors Relatively long
Relationship of the Prominent overbite
maxillary and mandibular (maxillary incisors anterior
incisors during normal to the mandibular incisors)
jaw closure
Relationship of the Patient cannot bring the
maxillary and mandibular mandibular incisors
incisors during voluntary anterior to (in front of) the
protrusion of the mandible maxillary incisors
Interincisor distance Less than 3 cm
Visibility of the uvula Not visible when the
tongue is protruded with
the patient in a sitting

CCetC
Block XX: Airway Management 11 of 15
MD 3
PROBLEMATIC/FAILURE IN VENTILATION: Bushy beard
Oxygen Saturation <90% Blood/Vomit
Significant gas leak in the patient’s mask Facial Trauma
Indicating the need for more airflow
Absence of chest movements II. OBESITY OR OBSTRUCTION
Two-handed face mask technique required Obesity
Change of operator required Heavy chest
Abdominal contents inhibit movement of the
INDEPENDENT RISK FACTORS WHICH RENDER diaphragm
PATIENTS DIFFICULT TO VENTILATE: Increased supraglottic airway resistance
Body Mass Index: >26 Billowing cheeks
Age: >55 years old Difficult mask seal
History of Snoring Quicker desaturation
Any Facial Abnormalities (Structure)
Different Airway Adjuncts: 3rd trimester pregnancy
Oropharyngeal Airway Increased body mass
Nasopharyngeal Airway Quick desaturation
Increased Mallampati Score
CONTRAINDICATIONS FOR NASOPHARYNGEAL Gravid uterus inhibits movement of the diaphragm
AIRWAY ADJUNCT:
Patients with coagulopathies Obstructions
Patients with fracture of the maxilla or the Foreign Body
sphenopalatine area Angioedema
Abscesses
OROPHARYNGEAL AIRWAY Epiglottitis
Puts the tongue in place so that adequate air would go Cancer
into the mouth to your trachea and down to your lungs Traumatic Disruption/Hematoma/Burn
Videl Airway
least soft tissue trauma when inserted to the III. AGE GREATER THAN 55
oropharyngeal area because of the smooth edges Associated with BVM difficulty, possibly due to loss of
Patients subjected to general anesthesia or inhaled tone in the upper airway
anesthetics have relaxed muscles of the soft
mandibular area (genioglossus). The tongue would fall IV. NO TEETH
backward and could cause airway obstruction which Face tends to “cave in”
makes oropharyngeal really important for anesthetized Consider leaving dentures in for BVM and remove for
patients intubation

NASOPHARYNGEAL AIRWAY V. STIFF


When avid airway access is needed Neck or lungs
refers to poor compliance
E. AIRWAY DIFFICULTIES reactive airway disease
Difficult to ventilate with a BVM COPD
Difficult laryngoscopy Pulmonary edema/ advanced pneumonia
Difficult to intubate History of snoring/ sleep apnea
Also predicts a higher Mallampati score
F. DIFFICULT TO BAG (M.O.A.N.S.)
Mask Seal G. DIFFICULT LARYNGOSCOPY AND INTUBATION
Obesity or obstruction (LEMONS)
Age > 55 Look Externally
No Teeth Evaluate 3-3-2
Stiff Mallampati Score
Obstruction
I. MASK SEAL Neck Mobility
Small hands Scene and Situation
Wrong mask size
Oddly shaped face

CCetC
Block XX: Airway Management 12 of 15
MD 3
I. LOOK EXTERNALLY Two finger widths mandibulohyoid distance.
Beards or facial hair Measured from the mentum to the top of the hyoid
Short, fat neck bone.
Morbidly obese patients The epiglottis arises from the thyroid and remains
Facial or neck trauma dorsal to the hyoid bone.
Broken teeth (can lacerate balloons) Therefore, the position of the hyoid bone marks the
Dentures (should be removed) entrance to the larynx.
Large teeth
Protruding tongue
A narrow or abnormally shaped face

II. EVALUATE 3-3-2


Ask:
Will patients mouth open wide enough to
accommodate 3 fingers?
Will 3 fingers fit between the mentum and hyoid
bone?
Will 2 fingers fit between the hyoid and thyroid
notch?
Figure 26. Mandibulohyoid distance of 2 finger widths
If not, expect a difficult intubation

Mouth opens at least 3 finger widths. III. MALLAMPATI SCORE

IV. OBSTRUCTION
Using Cormack and Lechane Grading

V. NECK MOBILITY
Ideally the neck should be able to extend back
approximately 35°
Problems:
Cervical Spine Immobilization
Ankylosing Spondylitis
Figure 24. Mouth opening assessment of at least 3 finger widths
Rheumatoid Arthritis
Three finger widths thyromental distance. Halo fixation
Distance from the mentum to the thyroid notch.
Ideally done with the neck fully extended. Can be done VI. SCENE/SITUATION
in-line Scene safety
Helps determine how readily the laryngeal axis will fall Environment
in line with the pharyngeal axis. Do you have a reasonable chance to get to the
If the thyromental distance is short, <3 finger widths, tube?
the laryngeal axis makes a more acute angle with the Consider space, positioning, access
pharyngeal axis and it will be difficult to achieve Egress
alignment. Will you be able to ventilate during egress?
Less space to displace the tongue A respiratory rate of 4 is better than a rate of 0!
Enough meds for a long extrication?

Table 9: Pathological States that influence airway


management.
PATHOLOGIC DIFFICULTY
STATE
Epiglottitis Laryngoscopy may worsen
(infectious) obstruction
Abscess Distortion of the airway renders
Figure 25. Thyromental distance of atleast 3 finger widths (submandibular facemask ventilation or

CCetC
Block XX: Airway Management 13 of 15
MD 3
retropharyngeal, tracheal intubation H. COMMANDMENTS OF AIRWAY MANAGEMENT
Ludwig’s angina) extremely difficult 1. Remain Calm!!!
Croup, bronchitis, Airway irritability with a 2. The First Priority Is ALWAYS DO BAG-MASK
pneumonia tendency for cough, VENTILATION
3. Call for Help Early
laryngospasm, bronchospasm
4. If You CAN’T VENTILATE: INTUBATE
Papillomatosis Airway obstruction
5. If You CAN’T INTUBATE: VENTILATE
Tetanus Trismus renders oral tracheal 6. Keep Track of Time
intubation impossible 7. Practice, Practice, Whenever You Can
Traumatic foreign Airway obstruction 8. When in doubt, pull it out!
body
REVIEW QUESTIONS
Cervical spine injury Neck manipulation may
1. Inferior anatomic border of nasal cavity.
traumatize the spinal cord
a. Cribriform plate
Basilar skull Nasotracheal intubation
b. Hard palate
fracture attempts may result in
c. Tongue
intracranial tube placement
d. Middle turbinate
Maxillary or Airway obstruction, difficult 2. Indication for airway management except.
mandibular injury facemask ventilation and
a. Acute respiratory failure
tracheal intubation
b. Surgery involving unusual position
Cricothyroidotomy may be
c. Patient under local anesthesia
necessary with combined
d. Inadequate ventilation
injuries
3. Poor laryngoscopic view if:
Laryngeal fracture Airway obstruction may worsen a. Thyromental distance of 2 fingerbreadths
during instrumentation b. Interincisor gap of 3 mm
Endotracheal tube may be c. Thyromental distance of 8cm
misplaced outside the larynx d. Atlanto-occipital extension of 85 degrees
and worsen the injury 4. What Mallampati class is correctly described?
Laryngeal edema Irritable airway a. Class II- only soft palate, fauces, uvula, and
(after intubation) Narrowed laryngeal inlet tonsillar pillars are visible
Soft tissue neck Anatomic distortion of the b. Class III- only soft palate. Fauces, and base of
injury (edema, upper airway uvula are visible
bleeding, Airway obstruction c. Class IV- soft palate is not visible
subcutaneous d. Class V- soft palate is not visible
emphysema) 5. What is true about Miller laryngoscope and not in
Neoplastic upper Inspiratory obstruction with Macintosh?
airway tumors spontaneous ventilation a. Miller 3 or 4 is used in adults
(pharynx, larynx) b. Less bruising of epiglottis
Lower airway Airway obstruction may not be c. Less trauma to the teeth
tumors (trachea, relieved by tracheal intubation d. Better exposure of glottis opening
bronchi, Lower airway is distorted 6. Gold Standard for sign of tracheal intubation
mediastinum) a. Laryngoscopy
Radiation therapy Fibrosis may distort the airway b. Fiberoptic visualization
or make manipulation difficult c. Capnography
Inflammatory Mandibular hypoplasia, d. NOTA
rheumatoid arthritis temporomandibular joint 7. How much air should be used to inflate the
arthritis, immobile cervical endotracheal tube cuff?
vertebrae, laryngeal rotation, a. 5-10cm H20
and cricoarytenoid arthritis b. 10-15cm H20
make tracheal intubation c. 15-20 cm H20
difficult d. No exact volume, as long as it inflates properly
Ankylosing Fusion of the cervical spine 8. What is the most common complication that can
spondylitis may render direct laryngoscopy possibly be seen in all the stages of intubation?
impossible a. Trauma
Temporomandibular Severe impairment of mouth b. Aspiration
joint syndrome opening c. Infection

CCetC
Block XX: Airway Management 14 of 15
MD 3
d. Obstruction
9. Which of the following is true?
a. Sensory branch of recurrent laryngeal nerve is
above vocal cord
b. External branch of superior laryngeal nerve is
the sensory portion
c. Motor branch of the superior laryngeal nerve
innervates the cricothyroid muscle
a. NOTA
10. Which of the following is true regarding the
required length from the orotracheal tube size
guidelines?
a. 8 yr old child – 14 cm
b. 25 yr old female – 23 mm
c. 0 yr old male – 20 cm
d. NOTA

Answers: BCACDCDBCA

REFERENCES
Upclass notes

CCetC
Block XX: Airway Management 15 of 15
MD 3
West Visayas State University – College of Medicine – Batch 2020
Block XX
Module 6 Local Anesthetics
Lecture 8
05/ 16/ 19
Dr. Jenny Sisnorio-Chan

TOPIC OUTLINE LIDOCAINE


I. Introduction and History • Prototype amide
II. Biochemistry • Derived from an aromatic amine (xylidine) and an
III. Mechanism of Action
amino acid
IV. Physiochemical Properties
V. Order of Blockade • Has Amide bond
VI. Selective Pharmacological Properties • Averts allergic reactions
A. Cocaine In 1948, lidocaine was introduced, the first amide and
B. Benzocaine
C. Bupivacaine with less allergies
D. Ropivacaine Lidocaine is also the most widely used anesthetic,
E. Lidocaine especially because it isn’t very allergogenic, as
VII. Systemic Toxicity compared to amino esters
VIII. Clinical Applications
A. Surface Anesthesia After lidocaine, the next most commonly used
B. Infiltration Anesthesia anesthetics are your bupivacaine & ropivacaine,
C. Nerve/Field Block respectively
D. Spinal Anesthesia
E. Epidural Anesthesia
IX. Adverse Effects BIOCHEMISTRY
Review Questions LOCAL ANESTHETIC AS WEAK BASES
References

LECTURER BOOK REFERENCE OLD TRANS

INTRODUCTION
LOCAL ANESTHESIA
• Drugs which produce a transient and reversible loss of
sensation in a circumscribed region without loss of
consciousness.
This is in contrast to general anesthesia, which does Figure 1. LA – Weak Bases. Source: Doc’s PPT
induce a loss of consciousness
• Produces by a narrow class of compounds; recovery is Local anesthetics consist of a lipid-soluble, aromatic
normally spontaneous, predictable and complete. benzene ring linked to an amine group (tertiary or
In many instances, local anesthetics are also used quaternary depending on pKa and pH) via either an
when performing spinal or epidural anesthesia amide or ester linkage.
The type of linkage separates the local anesthetics
HISTORY into either aminoamides, metabolized in the liver, or
COCAINE aminoesters, metabolized by plasma cholinesterases.
• First local anesthetic (late 19th century)
• Decreased fatigue and promoted the feeling of well- TWO TYPES OF LINKAGES
being • Types of linkages gives rise to 2 chemical classes of
Cocaine’s systemic toxicity, its irritant properties Local Anesthesia:
when placed topically or around nerves generated ESTER LINKAGE:
new interest in creating new ones. ─ Procaine:
procaine (Novocaine)
PROCAINE tetracaine (Pontocaine)
• First useful injectable local anesthetic benzocaine
• Prototype ester cocaine
• Derived from aromatic acid and an amino acid alcohol AMIDE LINKAGE
• Made of ester linkages ─ Lidocaine:
The first useful injectable local anesthetic, procaine, lidocaine (Xylocaine)
can be considered the prototype on which all mepivacaine (Carbocaine)
commonly used local anesthetics are based. bupivacaine (Marcaine)

CCetC Group No. 39 1 of 9


MD 3 Yap, Ycon, Ypil, Zarate
etidocaine (Duranest) transport of 3 sodium ions in exchange for the inward
ropivacaine (Naropin) transport of 2 potassium ions
With excitation of the nerve, there is an increase in
the membrane permeability to sodium ions, causing a
decrease in the transmembrane potential. If a critical
potential is reached (i.e., threshold potential), there is
a rapid and self-sustaining influx of sodium ions
resulting in depolarization, after which the resting
membrane potential is reestablished. From an
electrophysiologic standpoint, local anesthetics block
conduction of neural transmission by decreasing the
rate of depolarization in response to excitation,
preventing achievement of the threshold potential.
In short, local anesthetics work by stopping the action
MECHANISM OF ACTION
potential via blocking the sodium ion influx
• Block the transmission of action potential by inhibition
of voltage-gated sodium ion channels.
PHYSIOCHEMICAL PROPERTIES
• Reversibly binding to and inactivating sodium channels. PROPERTIES
• Variation of affinity by local anesthetics to sodium LIPID SOLUBILITY AND POTENCY
channels: • LA with lower pK have more rapid onset of action
Open or activated state because more unchanged forms result to more rapid
Inactivated state diffusion to the cytoplasmic side of the Na channel
Resting state • Potency reflects the ability of the LA molecule to
Primary target of LA is the open state; bind more permeate lipid membranes. So the more lipid soluble
readily to these channels; onset of neuronal blockade the drug is, the more potent it becomes
is faster in rapidly firing neurons
Local anesthetics bind to the activated and PROTEIN BINDING (DURATION OF ACTION)
inactivated states more readily than the resting state, • Degree of protein binding also affects activity of local
attenuating conformational change. anesthetics, as only the unbound form is free for
• Ionized forms determine the action of local anesthetics pharmacologic activity. In general, the more protein-
bound a local anesthetic is, the longer its duration of
action
Protein binding of local anesthetics is primarily
through alpha-glycoproteins, and to albumin (but to a
lesser extent)

PKA (IONIZATION CONSTANT) AND


SPEED OF ONSET
• pKA is the pH at which the fraction of ionized and
nonionized drug is equal. The pKa provides a useful
way to describe the propensity of a local anesthetic to
exist in a charged or an uncharged state. The lower the
Figure 2. Mechanism of Action of LAs. Source: Doc’s PPT pKa, the greater is the percent of un-ionized fraction at
a given pH and the faster is the onset of the anesthetic
Under normal or resting circumstances, the neural
This relationship is indirectly proportional, unlike the
membrane is characterized by a negative potential of
previous two which are directly proportional
roughly –90 mV (the potential inside the nerve fiber is
negative relative to the extracellular fluid). This
negative potential is created by active outward FORMS
transport of sodium and inward transport of CHARGED FORM
potassium ions, combined with a membrane that is • Responsible for predominant action of local anesthetics
relatively permeable to potassium and relatively • Highly hydrophilic
impermeable to sodium ions. • Relatively incapable of penetrating the nerve
The resting membrane potential is caused by the membrane to reach its site of action
sodium-potassium pump, which causes the outward The charged form is the one most responsible for the
action of your local anesthetics

CCetC
Block XX: Local Anesthesia 2 of 9
MD 3
NEUTRAL BASE more rapid nerve conduction. The presence of myelin
• Permits the penetration to the nerve membrane to gain also increases conduction velocity.
access to the receptor This order of blockade is actually more relevant to
All clinically used local anesthetics are weak bases spinal anesthetics (e.g. appendectomy), and not so
that can exist as either the lipid-soluble, neutral form much with regards to local anesthetics
or as the charged, hydrophilic form. The combination In spinal anesthesia, Tuffier’s line (a virtual line
of pH of the environment and pKa, or dissociation connecting the tops of the iliac crests) is used as a
constant, of a local anesthetic determines how much landmark for the anesthetic. This is at the level of the
of the compound exists in each form L3-L4 intervertebral space
Spinal anesthetics also typically affect up to the level
FACTORS THAT AFFECT POTENCY of T4 (level of nipples)
• Hydrophobicity
It should be more lipid soluble to increase potency Supplementary Notes on Classification of Nerve Fibers
• Hydrogen ion balance Diameter
Conduction

More basic, more rapid onset of action (such as Type Subtype


(mm)
Velocity Function
(m/sec)
sodium bicarbonate) A Alpha 12-20 80-120 Propioception
• Vasoconstrictor/vasodilator properties (myelinated) , Large Motor
Affect the rate of vascular uptake / Epinephrine’s Beta 5-15 35-80 Small Motor,
vasoconstrictive effects prolongs the LA block, Touch,
increases block intensity and it decreases systemic Pressure
absorption of LA Gamm 3-8 10-35 Muscle tone
• Fiber size, type and myelination a
Delta 2-5 5-25 Pain,
The thicker the fiber, the faster it could be blocked /
Temperature,
myelinated nerve fibers are easily blocked
Touch
• Frequency of nerve stimulation B 3 5-15 Preganglionic
• pH (Myelinated) Autonomic
Acidic environments antagonize the anesthetic C 0.3-1.5 0.5-2.5 Dull pain,
block, that’s why anesthetics are very ineffective (Unmyelinat Temperature,
for abscesses and cellulitis (acidic environment) ed) Touch
• Electrolyte concentration
Hypokalemia and hypercalcemia antagonize
blockade SELECTIVE PHARMACOLOGICAL PROPERTIES
A. COCAINE
ORDER OF BLOCKADE • Medical use limited to surface or topical anesthesia
Order of blockade according to books: • Avoid epinephrine because cocaine already has
AUTONOMIC vasoconstrictor properties.
PAIN • A toxic action on heart may induce rapid and lethal
TEMPERATURE cardiac failure.
TOUCH • A marked pyrexia is associated with cocaine overdose.
DEEP PRESSURE
MOTOR (unmyelinated, type A fiber)
B. BENZOCAINE
Autonomic fibers are the smallest, and blockage will
• Available in many preparation for relief of pain and
manifest as a drop in BP and other vital signs
irritation
Blockage of fibers assoc. with temperature will
• For surface anesthesia (topical) only
manifest as warmth
• Used as ointments, sprays, etc.
Recovery then is in reverse which means motor
• Used to produce anesthesia of mucous membranes
returns first then goes upward
• Methemoglobinemia
Follow the order above for exam purposes, but
clinically the order is ASTVMT (autonomic, sensory, C. BUPIVACAINE (MARCAINE)
temperature, vibration, motor, touch) • No topical effectiveness
There is evidence to suggest that large, myelinated • Slower onset and one of the longer duration agents
nerve fibers are more sensitive to local anesthetic • Unique property of sensory and motor dissociation can
blockade than smaller, unmyelinated fibers. provide sensory analgesia with minimal motor block
Usually, it is the nerve fibers diameter that influences has been popular drug for analgesia during labor
conduction velocity; a larger diameter correlates with • More cardiotoxic than other LA

CCetC
Block XX: Local Anesthesia 3 of 9
MD 3
D. ROPIVACAINE Usually given to patients before IV insertion to
• Enantiomer of bupivacaine (S stereoisomer) avoid pain, used in big hospitals like St. Luke’s
• No topical effectiveness
• Clinically equivalent to bupivacaine B. INFILTRATION ANESTHESIA
• Similar sensory versus motor selectivity as bupivacaine • Direct injection into tissues to reach nerve branches
with significantly less CV toxicity and terminals
• Can be superficial as well as deep
E. LIDOCAINE • Used in minor surgery.
• Effective by all routes. • Immediate onset with variable duration
• Faster onset, more intense, longer lasting • This type involve skin region as deep as intraabdominal
• Good alternative for those allergic to ester type tissue
• More potent than procaine but about equal toxicity
• More sedative effect than others C. NERVE BLOCK / FIELD BLOCK
• Interruption of nerve conduction upon injection into the
Recommended dose for lidocaine is 5-7 mg/kg BW region of nerve plexus or trunk
5 mg/kg if without epinephrine; 7 mg/kg if with • Used for surgery, dentistry, analgesia
The concentration of the solution should also be • Given within specific nerve area such as brachial
considered (e.g. 2% lidocaine solution = 20 mg/mL) plexus, within intercostal nerves, abdominal nerves are
FORMULA: (dose x body weight) / concentration targeted, cervical plexus when neck region is targeted
Example: 20 kg child and 2% lidocaine solution
[(5 mg/kg) x (20 kg)] / (20 mg/ml) D. SPINAL ANESTHESIA
= (100 mg) / (20 mg/ml) • Injection into subarachnoid space below level of L2
= 5 mL vertebra to produce effect in spinal roots and spinal
cord
SYSTEMIC TOXICITY OF LOCAL ANESTHETICS • Use hyperbaric or hypobaric solutions depending on
• Systemic toxicity of local anesthetics results from area of blockade
excessive plasma concentrations of these drugs, most • Used for surgery to abdomen, pelvis or leg when can’t
often from accidental intravascular injection during use general anesthesia
performance of peripheral nerve blocks.
That is why, it is very important to solve for the E. EPIDURAL / CAUDAL ANESTHESIA
• Injection into epidural space usually at lumbar or sacral
maximum dose that should be given to the patient.
levels or near dura matter where nearly most nerves
pass closely
• Lower part of the body
• For painless child birth

Question: What is the difference between Spinal and


Epidural Anesthesia
Remember the layers:
Skin
Subcutaneous fats
Supraspinous ligament
Infraspinous ligament
Figure 3. Systemic Toxicity. Source: Doc’s PPT Ligamentum flavum (Hard, has Popping sound)
Epidural Space
CLINICAL APPLICATIONS Dura Space
A. SURFACE ANESTHESIA
Subarachnoid Space
• Topical Anesthesia
Injecting epidural anesthetics typically require larger
• For the ears, nose, mouth, bronchial tree,
volumes because the epidural space is larger; on the
nasopharynx, cornea, GIT and urinary tracts
other hand spinal anesthetics tend to have more
Lidocaine, tetracaine, Benzocaine
potent effects and a faster onset as it is injected
EMLA cream (Eutectic Mixture of Local
directly to the CSF (subarachnoid space)
Anesthetics) lidocaine 2.5% + prilocaine 2.5%
permits anesthetic penetration of keratinized layer
ADVERSE EFFECTS
of skin as deep as 5mm, producing localized • CNS (1st stimulation, then depression)
numbness. • Local Neurotoxicity (cauda equina syndrome
associated with continuous spinal anesthesia – CSA)
CCetC
Block XX: Local Anesthesia 4 of 9
MD 3
• CVS (bupivacaine – most cardiotoxic) In OR, what is available are lidocaine, etidocaine,
• Motor Paralysis ropivacaine. Procaine is not available in our setting.
• Hematological Effects If we give local anesthetic, there will be sodium
• Hypersensitivity reactions channel blockade which will lead to no conduction,
so there will be no contraction causing paralysis.
Supplementary Notes on Prevention of Toxicity Local anesthetics can be given through axillary block
• Inquire about history of allergy. or epidural.
• Caution in presence of liver/myocardial damage.
• Proper site (correct knowledge of nerve course). PROCAINE
• Minimal effective dose usage (avoid I/V adm). • 1st synthetic ester in 1905
• Wait after injection.
• Observe the face for any twitching, excitement, and LIDOCAINE
pulse for tachycardia. • 1st synthetic amide in 1943
• Observe post – op for allergic reactions. Lidocaine is used in local anesthesia since it has
• Avoid food intake at least 04 hrs prior to anesthesia to preservatives which can cause complications
prevent vomiting. mistakenly used for spinal and epidural

Upclass Notes with Dr. Joselito Villa (ADeoS) SODIUM CHANNEL BLOCKADE
SUPPLEMENTARY NOTES (UPCLASS)
A. LOCAL ANESTHETICS
• Produce reversible blockade of neural conduction by
their actions on the Na channel of neurons
Once Na enters, there is a conformational change
that would eventually lead to muscle contraction.
With anesthetics, the Na channels are blocked and
there would be no depolarization nor contraction
thus leading to paralysis
Mechanism of action
Local anesthetics cause a loss of sensation in a
body part either through:
─ A blockade of excitation of nerve endings, or Figure. Sodium Channel Blockade
─ Inhibition of the process of conduction in
peripheral nervous tissue The more local anesthetics come into contact with
nerve fibers, the longer is the duration of local
DEPOLARIZATION anesthesia
• Inward movement of sodium ions from the extracellular
to intracellular FREQUENCY DEPENDENT BLOCKADE
• As the frequency of stimulation increases, more sodium
REPOLARIZATION channels remain open for longer periods of time, with
• flow of K ions from intracellular to extracellular more opportunity for local anesthetics to enter the
Ampules have no preservatives, so they can be channel
used for spinal anesthesia. If anesthesia with
preservatives is administered, convulsions may DIFFERENTIAL NEURAL BLOCKADE
occur. • Partial sensory blockade but retained motor function
What should you do before you give local • Effects in the following order:
anesthetic? Loss of pain and temperature sensation
─ Before we give anesthetic, we must first Loss of proprioception
aspirate to avoid intravascular administration, Loss of touch and pressure sensation
which can cause systemic toxicity. Motor paralysis

COCAINE In giving local anaesthetics via spinal or epidural, the


• 1st recognized anesthetic (used commonly by first to be blocked are the smaller fibers
ophthalmologists) (proprioception and pain) and last to be blocked are
• natural alkaloid; introduced into use in 1880 the larger fibers which are for motor function.
• Used by Koller as a topical ophthalmologic anesthetic

CCetC
Block XX: Local Anesthesia 5 of 9
MD 3
Table. Classification of Nerve Fiber Types LIPID SOLUBILITY
Fiber type Size (U) Function • The more lipid-soluble local anesthetic molecules
A alpha 12-20 Somatic, motor, penetrate the nerve membrane more easily and have
proprioception
greater intrinsic anesthetic potency.
beta 5-12 Touch, pressure
gamma 3-6 Motor to spindle muscles • Ex. Bupivacaine and tetracaine
delta 2-5 Pain, temperature, touch • Directly related to potency
B <3 Autonomic(preganglionic)
C 0.3-1.4 Pain, autonomic(post- PROTEIN BINDING
ganglionic) • The duration of action of a local anesthetic depends on
binding to the protein components of the nerve
SENSITIVITY OF NERVE FIBERS DEPENDS ON: membrane
Size Poor protein binding = short duration of action (ex.
• Small A-delta & unmyelinated C fibers – most sensitive Procaine)
• Sensory F & large myelinated motor fibers – most Strong binding = long acting (ex. Tetracaine,
resistant bupivacaine and lidocaine)
• Directly related duration of action
Location within the nerve
• Close to nerve surface PKA
Easily and promptly blocked • the greater the fraction of the drug that is in the lipid-
soluble uncharged free base form, the faster is the
Myelination
onset of anesthesia
In differential neural blockade, if the motor blockade
• the relationship between the concentrations of the
is at T6, the sensory blockade is at T4 and
charged and uncharged forms depend on the hydrogen
autonomic is at T2 because the sensory is 2
concentration:
dermatomes higher while autonomic is 4
log ([B]/[BH+])= pH – pKa
dermatomes higher.
• when the pH of the solution is the same as the pKa of
The first to be blocked are those unmyelinated C
the drug, the ionized and un-ionized forms are present
fibers and the ones that are closer in proximity to the
in equal amounts
site of the local anaesthetic.
• lesser pKa, more rapid onset
B. CLASSIFICATION OF LOCAL ANESTHETICS i.e. amide group (prilocaine, lidocaine,
ESTERS mepivacaine, etidocaine)
• Procaine Onset
• Cocaine Lidocaine = 5-10 min, Mepivacaine = 10-20 min
• 2-chlorprocaine ─ The more basic the pH, the faster the onset,
• Tetracaine which is why FENTANYL is sometimes added.
• Inversely related to onset
AMIDES
• Lidocaine INTRINSIC VASCULAR ACTIVITY
• Mepivacaine • Local anesthetics alter vascular tone in a dose-
• Prilocaine dependent manner
• Bupivacaine • The vasodilator actions of local anesthetics influence
• Etidocaine efficacy and duration of action
• Ropivacaine Preload with IV fluid to reduce hypotensive effects.
You can see in the amides, there is ‘i’ in the middle. (10 cc per kilo)
Just for you to remember easily. • Blood pressure= Cardiac output x peripheral resistance
• If there’s hypotension, you need to give preload to
C. PHARMACOLOGY OF LOCAL ANESTHETICS increase peripheral resistance to increase blood
Table. Ester and Amides pressure.
Stability in Allergic • The higher the block, the higher the vasodilatation
Metabolism
Solution reaction effect causing hypotension
Esters Plasma esterase Unstable Rare
Amides Hepatic enzymatic Stable Very rare
D. ESTER-DERIVED LOCAL ANESTHETICS
So in elderly, we have to reduce our dosage
COCAINE
because the metabolism for elderly is low. Patients
• High risk for systemic toxicity and addiction
with liver problem also need a reduced dose.
• Produces vasoconstriction when applied as topical
anesthesia

CCetC
Block XX: Local Anesthesia 6 of 9
MD 3
• Used in anesthetizing the nasal mucosa prior to Estimated duration (w/o epinephrine): 1-1.5 hours
nasotracheal intubation Estimated duration (w/ epinephrine): 2-3 hours
The maximum recommended dose of topical cocaine
is 100 mg and its concentration when applied to the MEPIVACAINE
mucus membrane should not exceed 10%. Its • Effects are similar to lidocaine, but lesser toxicity in
primary use is as a topical anesthetic in the upper adults and duration of action is longer
respiratory tract like the intranasal area. 10 ml of 4% • less marked dilatation
cocaine solution delivered by soaking 3 cotton-tipped
applicators to each nostril and waiting for several • Metabolism is markedly prolonged in the fetus and the
minutes to have its effect. Addition of epinephrine is newborn
avoided to prevent cumulative sympathomimetic
effect that can cause serious arrhythmias. PRILOCAINE
• Lesser vasodilatation than lidocaine
PROCAINE • Prilocaine w/o epinephrine has a similar effect as
• Penetrates the tissues poorly, produces anesthesia lidocaine w/ epinephrine
slowly, dissipates rapidly • Least toxic of all the amides – IV regional anesthesia
• Rapid hydrolysis by pseudo-cholinesterase and can be • Can cause methemoglobinemia if given in large doses
used safely in large amounts (>600mg)
• Primary use: infiltration, differential spinal blockade, • not used in obstetric anesthesia
occasionally spinal anesthesia
BUPIVOCAINE
CHLOROPROCAINE • Slow onset long duration, potent and toxic
• Short-acting, not very potent, not likely to cause • In lesser concentrations, it produces excellent sensory
systemic toxicity analgesia with little or no motor impairment
• Shortens the duration of bupivacaine if given as a • More cardiotoxic than lidocaine
mixture in epidural anesthesia Don’t use in patients with heart problems
• Not available
Onset: Rapid
TETRACAINE Maximum dose (mg/kg) w/o epinephrine: 2.5
• Long acting, potent anesthetic Maximum dose (mg/kg) w/ epinephrine: 3.5
• Primary use: spinal anesthesia Estimated duration (w/o epinephrine): 4 hours
• Excellent topical anesthesia, i.e. corneal and Estimated duration (w/ epinephrine): 6-8 hours
endotracheal topical anesthesia
There is a more profound motor blockade in high ETIDOCAINE
spinal compared to bupivacaine • Produces conduction blockade of rapid onset and long
• High spinal-above T1 duration, with the depth and duration of motor blockade
When we do high spinal and we give this, when the exceeding those of sensory block
drug reaches T4, sometimes, the patient cannot • Used primarily for operations where muscle relaxation
breathe. So, at times there is no chest lift with is important
tetracaine. Occasionally, patient loses their voice 
inability to speak. Unlike in etidocaine, even if you ROPIVACAINE
have high spinal, the patient is still capable of • A new amide local anesthetic
respiration. • Its structure is similar to mepivacaine and bupivacaine
• pKa and protein binding similar to bupivacaine but less
E. AMIDES lipid soluble
LIDOCAINE • eliminated more rapidly than bupivacaine resulting in
• Most versatile and commonly used lesser myocardial toxicity
• Potent, rapid onset(5-10 min), moderate duration of
action(1-2 H) F. PHARMACOKINETICS OF LOCAL ANESTHETIC
• Uses: infiltration, peripheral nerve block, spinal or AGENTS
epidural anesthesia, topical in ointment, jelly and ABSORPTION
aerosol forms Factors affecting absorption
Also used for ventricular arrhythmias (V-tach, PVC) Site of injection
• most rapid absorption and greatest peak blood
Onset: Rapid concentrations in the ff. order:
Maximum dose (mg/kg) w/o epinephrine: 3.5 Intercostal nerve block
Maximum dose (mg/kg) w/ epinephrine: 5-6 Caudal block

CCetC
Block XX: Local Anesthesia 7 of 9
MD 3
Lumbar epidural block Effective vasoconstriction can be achieved by as
Brachial plexus block little as 1:800,000 dilution added to the local
Subcutaneous tissue infiltration anesthetic solution
Subarachnoid block Side effects of epinephrine include:
What type of anesthesia: if there are more blood Tachycardia, palpitation, apprehension, headache,
vessels, there is higher tendency of systemic elevated BP
absorption?
Answer: Intercostal block Vasoactive properties
For intercostal nerve block, we pull the rib and give • Physical and chemical properties such as lipid solubility
into the intercostal groove. There is rapid • Ex. Etidocaine produces lesser peak blood
absorption in your intercostal block, followed by concentration than bupivacaine after epidural
your caudal block, epidural, brachial plexus, and administration although they both have similar
subarachnoid block. Subarachnoid block has the vasodilator activity
least absorption.
What will you give to avoid toxicity? DISTRIBUTION
Answer: Epinephrine. Usually, we add epinephrine • Blood concentrations decrease rapidly as the drug is
to our local anesthetics. Especially in intercostal redistributed to well-perfused tissue and more slowly if
block, we give lidocaine and epinephrine taken up elsewhere, metabolized, and excreted
(1:200,000). To prevent systemic absorption, we • What are the four major organs where drug is
give epinephrine.IN axillary block, we usually give distributed? Brain, heart, kidney, liver
epinephrine. During our residency, our consultant
METABOLISM
forgot to give epinephrine, that’s why our patient
• Esters are hydrolyzed in the plasma by
had convulsions.
What will you give if there’s toxicity? pseudocholinesterase
Answer: First stop the procedure, you give oxygen, • Amides are broken down in the liver by N-dealkylation
you give benzodiazepines, then it’s supportive (BP, of the tertiary amine and then hydroxylation of aromatic
etc) nucleus
What do benzodiazepines do?
G. TOXICITY OF LOCAL ANESTHETIC AGENTS
Answer: It facilitates GABA, which are inhibitory CNS TOXICITY
neurotransmitters at the same time it decreased • Initial events
the cerebral requirement for oxygen. Tinnitus
Light-headedness
Addition of vasoconstrictor agents
Confusion
• To retard absorption
Circumoral numbness
• Epinephrine 5μg/mL
• Excitation phase
• Prolongs the duration of anesthesia (about 3 hours)
Tonic-clonic convulsions
• Decreases peak blood concentrations
• Depression phase
• Permits use of larger amounts of anesthesia w/o inc.
Unconsciousness
risk of systemic toxicity
Generalized CNS depression
Respiratory arrest
Epinephrine is added to local anesthetic solutions to
If you have signs of toxicity, stop the procedure then
induce vasoconstriction. Aside from hemostasis, its
give propofol (but sometimes this can also
other beneficial effects are:
exacerbate seizures)
Delays absorption of anesthetic agent with
resultant longer duration of anesthetic action
SYSTEMIC TOXICITY
Hastens the onset of action of the anesthetic
• Occurs when anesthetics are mistakenly injected in
solution
large doses into an artery or vein
the quality of the block because it reduces
• Seizures
absorption and hence more agent is available for
• Coma
the neural blockade
• Arrhythmias
Lessens the discomfort by hastening the onset
• Cardiac arrest
Increases by as much as half the dose of local
Avoided by aspirating before injecting, give
anesthetic that can be safely given
vasoconstrictor (epinephrine)
Sometimes we give lidocaine via IV.

CCetC
Block XX: Local Anesthesia 8 of 9
MD 3
CARDIOVASCULAR TOXICITY d. Meninges
• Occurs at greater blood concentrations than does CNS 4. It is the primary mechanism by which most local
toxicity anesthetics work
• Myocardial depression, vasodilatation, and impaired a. Prevention of sodium influx
cardiac conduction b. Prevention of sodium efflux
c. Prevention of potassium influx
H. TREATMENT OF SYSTEMIC TOXICITY d. Prevention of potassium efflux
• Giving of anesthetic agent is stopped 5. What is the dose of lidocaine that should be
• Oxygenation and reassurance administered when no epinephrine is available?
• If w/ seizures, hyperventilate with 100% O2 (if not a. 4 mg/kg
effective, give Midazolam (1-3mg) or thiopental (25- b. 5 mg/kg
50mg) c. 6 mg/kg
• Secure adequate ventilation d. 7 mg/kg
• Sodium bicarbonate treatment
IF your patient is desaturating, you have to intubate
or ventilate. You should prepare your anesthesia
meds and prepare the machine and intubation set, in Answers: CDCAB
case mag-convulsion. If there is arrest, your
laryngoscope should be ready. Within 3 minutes of REFERENCES
hypoxia, there can already be irreversible neurologic • Doc Chan’s PPT and Lecture
damage to the patient. So, materials should be • Upclass Notes as Supplementary
prepared.

Table. Recommended max doses for single injection


Drug mg/kg w/o Epi Mg/kg w/ Epi
Chlorprocaine 11 14
Lidocaine 4 7
Epivacaine 4 7
Prilocaine 7 8.5
Bupivacaine 2.5 3.2
Etidocaine 6 8

Nice-To-Knows:
• Dictum in local anesthesia: “Aspirate before injecting!”
• Avoid intravascular injection, prevent systemic toxicity.
• CSF
In subarachnoid space
Produced in choroid plexus
120-150 mL

REVIEW QUESTIONS
1. It is the least allergogenic local anesthetic
a. Procaine
b. Cocaine
c. Lidocaine
d. Bupivocaine
2. Which property of local anesthetics is indirectly
proportional to its potency?
a. Protein binding
b. pKa
c. Lipid solubility
d. None of the above
3. When this structure is pierced during anesthesia
administration, a popping sound is heard
a. Vertebra
b. Spinal cord
c. Ligamentum flavum
CCetC
Block XX: Local Anesthesia 9 of 9
MD 3
West Visayas State University – College of Medicine – Batch 2020
Block XX
Module 6 Pain Management
Lecture 9
05/ 16/ 19
Dr. Marcos D. Bito-onon, MD, MPG, FPSA, DPBPS

TOPIC OUTLINE ***start of supplementary notes***


I. Pain A. CONCEPT OF TOTAL PAIN
II. Pain assessment Emotional
III. Nociception and pain Loss of function
IV. Pain physiological pathway
V. Mechanism of perioperative pain
Coping abilities
VI. Mechanism of postoperative pain Financial
VII. Post-operative pain management Direct cost
VIII. Therapeutic options Indirect cost
IX. Benefits of treating pain
X. Summary Physical disability
Review Questions From diseases
References From treatment
Appendices
Spiritual component of pain
Guilt
LECTURER BOOK REFERENCE OLD TRANS
Life-closure / End-of-life issues
***end of supplementary notes***
I. PAIN
• WHO Definition B. CLASSIFICATION OF PAIN
• Intensity
Unpleasant sensory and emotional experience
• Chronicity
It is associated with actual or potential tissue
• Pathophysiology
damage
• Pain pattern
Described in terms of such damage
• What is pain?
PAIN ACCORDING TO INTENSITY
Pain is unpleasant
• Mild
emotion is important
• Moderate
the cause is not always visible
• Severe
pain is what the person says hurt
PAIN ACCORDING TO CHRONICITY
• Acute
• Chronic non-malignant (Chronic persistent)
• Cancer Pain

Table 1. Classification of pain according to duration, cause,


mechanism
Duration Acute or Chronic
Cause Cancer or Non - cancer
Nociceptive (physiological)
Mechanism
Neuropathic (pathological)

Acute pain
Figure 1. The motivational component of pain – sensory, affective, and • Originates from tissue damage
cognitive. • Occurs < 3 months
Because we expect the injury to have healed less
Pain is a “passion of the soul” – Aristotle
than 3 months
Can be an expression of discomfort especially in
• Associated with sympathetic hyperactivity
pediatric patients
• Has Signal value
It is not the responsibility of the patient, or the client
• Correlates with tissue damage
who is complaining of pain, but by the doctor, or
• Diminishes with healing of injury
nurse, or clinical clerk to accept the client’s complain
• Associated with neuroendocrine stress proportional to
of pain
intensity
Dictum: Accept the report of patients of pain
• Serves to detect, localize, and limit tissue damage

CCetC Group No. 38 1 of 14


MD 3 Villanueva, VIllaruel, Visto
By withdrawing from the stimulus Cancer Pain
• Self- limited and resolves with treatment of underlying • Examples:
injury Uterine cervical cancer, breast cancer
• Lasts a few days to weeks Metastases in bone
Example of acute pain is post-operative pain • Features of acute and chronic pain
• Temporarily related to injury and that resolves during • May be acute or chronic
the appropriate healing period • Often mixed nociceptive and neuropathic pain
• It often responds to treatment with analgesic • Usually gets worse over time if untreated
medications and treatment of the precipitating cause • Classification of cancer pain by general etiology:
• If the acute phase of pain is neglected, it may proceed Pain due to direct effects of tumor progression
to chronic pain. ─ Compression or ischemia of painsensitive
Symptom of other disease structures
Pain due to cancer treatment
Chronic Non-Malignant pain (Chronic Persistent) ─ Osteoradionecrosis, neuropathy from
• Pain lasting longer than 3 months; OR lasting after chemotherapy, post-mastectomy syndrome
normal healing Pain due to diagnostic procedures
• May continue during the patient’s lifetime ─ Venipuncture, bone marrow biopsy, lumbar
No longer has signal value, puncture
• Pain is not protective and does not warn of impending Pain due to chronic illness and debility
significant tissue damage ─ Decubitus ulcers, muscle spasm from
NO signal value; pain becomes a disease itself prolonged bedrest
• Pain which persist despite therapy Pre-morbid chronic pain
• Absence of sympathetic hyperactivity ─ Osteoarthritis, chronic radiculitis
• Presence of pain behaviors
Example would be rheumatoid arthritis Non- cancer pain
• Disease of the D’s Acute Non-cancer Pain
Dysfunction • Examples: fractures, appendicitis
Deconditioning • Symptoms of tissue injury
Drug misuse Nociceptive
Depression Occassionally neuropathic (eg. Sciatica)
Disability
• Pain that persists longer than 1 month beyond the
Chronic Non-cancer Pain
normal healing period or that is associated with a
• Examples:
pathological process that causes continuous or
Chronic back pain, arthritis, trigeminal neuralgia,
recurrent pain over months or years – John J. Bonica,
post-herpetic neuralgia, poststroke pain
MD
poststroke pain – because of some nerve
the pain become a disease in itself
recovery
• Damage may not be obvious
Table 2. Acute vs Chronic Pain
• Complex, may be mixed nociceptive or neuropathic
Type of Pain Physiologic Behavioral
• Different pharmacologic treatment may be needed
Acute • Increased • Restlessness
respirations • Distraction Table 3. Cancer vs Non-Cancer Pain
• Increased • Worry • Progressive; may be a mixture of
pulse • Distress Acute and Chronic; Acute
• Increased BP • Sense of exacerbation / long standing pain
• Dilated pupils impending Cancer • Can be due to bone metastasis, pain
• Diaphoresis death Pain secondary to chemotherapy, pain due
• GI disturbances to prolonged immobilization, direct
Chronic • Normal • Reduced or effects of tumor progression,
respirations, absent physical diagnostic procedures
• Pulse, BP, activity Non- Mild limitation means the patient’s
pupil size • Despair Cancer activity is mild to moderately restricted
• No diaphoresis • Depression Pain by pain
• Hopelessness

CCetC
Block XX: Pain Management 2 of 14
MD 3
PAIN ACCORDING TO PATHOPHYSIOLOGY • Breakthrough – it is a subset of incidental pain, it is
Nociceptive pain which is controlled but sometimes it
• physiological pain and caused by stimulation of pain spontaneously recurs and managed by giving 50% of
receptors the usual analgesic dose.
Higher dose if sige2 na gid
Visceral • Question: What is end-of-dose failure?
• Due to injury of sympathetically innervated organs It is when patients experience increased pain
• Characteristics: occurring at the end of a scheduled dose. For
Vague example, you are giving the patient morphine every
Deep 6 hours. On the 5th hour, the pain manifested.
Dull, aching, dragging, squeezing Meaning, the frequency of your dose is inadequate
• Associated with referred or transferred pain due to for the patient. That’s why there is end-of-dose
double innervations failure. So, it’s either you increase the frequency of
Double innervation - from sclerotomal and higher your dose from q6h to q4h because on the 5th
level (autonomic) innervations hour, there is resurgence of pain or you may
• Occurs only on 3 compartments: increase the basic dose of you medication.
Thoracic
Abdominal II. PAIN ASSESSMENT
Pelvic cavities • Pain is always subjective
• Usually starts at the midline specifically at the • Patient’s self-report of pain is the gold standard for
epigastric region assessment
• How severe is the pain?
What is the pain score?
Somatic How is the pain affecting the patient?
• Due to nociceptive activation of the tissues How does it affect the family?
• Characteristics: How does it affect the work and mood?
Constant • What is the pain type?
well-localized Acute or chronic
Throbbing, aching, gnawing Cancer or non-cancer
• Opioid-sensitive (Fentanyl, Demerol) Nociceptive or neuropathic
• Responsive to nerve block • Are there other factors?
Depends on how thorough you extract the history
Neuropathic ─ Physical
• Associated with nerve injury, compression, or invasion ─ Psychological
• Types: • Step 1: Patient self-report of pain
Central – post spinal transection • Step 2: Assumption of Pain
Peripheral – amputation, transection • Step 3: Behavioral indicators of pain
Mixed – post-Herpetic neuralgia • Step 4: Solicit information from caregivers and
• Characteristics family members
Persistent • Step 5: Analgesic trial
Constant • R.A.T Method
Shock-like, shooting, stabbing
Pins and needles R- RECOGNITION
Electric-like observation, ask, guarding behaviors, validate with
• Resistant to opioids the folks
Opioids can work but should be higher in dose in young children: persistent crying, irritability
• Best treated with anti-convulsants, antidepressants, in older children and adults: facial expressions,
and epidural steroid loss/change in appetite, poor school/work
performance, low mood, undue quietness
Psychogenic/Idiopathic
• for secondary gain A - ASSESSMENT
vital signs, inflammatory signs, pain rating
PAIN ACCORDING TO PATTERN Pain is always subjective, and patients’ self-report
• Constant of pain is the gold standard for the measurement
• Intermittent of pain; First and foremost, believe the patient’s
• Incidental – associated with movement report of pain.

CCetC
Block XX: Pain Management 3 of 14
MD 3
T - TREATMENT This scale is used to assess pain intensity in
persons who are able to self-report and is the best
A. ASSESSMENT OF SEVERITY choice for most older adults
• Guides choice of treatment When the patient is unable to describe the pain
• Measures response to treatment numerically by 1 to 10, we can use the value of
Measure and record severity before and after money to quantify the amount of pain the patient is
treatment; If you don’t measure it, you won’t having.
improve it.
• The “fifth” vital sign
• What is the pain score?
At rest
With movement
• How is the pain affecting the patient?
Can the patient move, cough?
Figure 3. Numeric Rating Scale
Can the patient work?
Verbal Descriptor Scale
***start of supplementary notes*** • Pain Thermometer to assess pain intensity for persons
JCAHO STANDARDS able to self-report
Pain is the 5th vital sign
Record patient data in a way that promotes VISUAL RATING SCALES
reassessment Visual Analog Scale
Initial Pain assessment A Visual Analogue Scale (VAS) is a measurement
Regular assessment of pain instrument that tries to measure a characteristic or
Education of health care workers attitude that is believed to range across a continuum
Development of quality improved medications that of values and cannot easily be directly measured.
address pain For example, the amount of pain that a patient feels
***end of supplementary notes*** ranges across a continuum from none to an extreme
amount of pain. Operationally a VAS is usually a
B. PAIN ASSESSMENT TOOL horizontal line, 100 mm in length, anchored by word
descriptors at each end. The patient marks on the
line the point that they feel represents their
perception of their current state. The VAS score is
determined by measuring in millimetres from the left
hand end of the line to the point that the patient
marks. (D. Gould, et al., 2001).
most objective pain rating scale

Figure 2. Pain assessment tool

VERBAL RATING SCALES


Figure 4. Visual Analogue Scale
Numerical rating scales
Perhaps one of the most commonly used pain
Faces Pain Scale
scales in healthcare, the numerical rating scale
• Wong-Baker Faces Pain scale - To assess pain
offers the individual in pain to rate their pain score. It
intensity in persons who are able to self-report but are
is designed to be used by those over the age of 9. In
unable to use a numeric rating scale.
the numerical scale, the user has the option to
• Useful for patients who are aphasic and aged <3 years
verbally rate their scale from 0 to 10 or to place a
old.
mark on a line indicating their level of pain. 0
indicates the absence of pain, while 10 represents
the most intense pain possible. The Numerical
Rating Pain Scale allows the healthcare provider to
rate pain as mild, moderate or severe, which can
indicate a potential disability level.

CCetC
Block XX: Pain Management 4 of 14
MD 3
Nociception involves your pain/nociceptive receptors
which transmit pain into your nerves leading to the
interpretation and pain perception.
It is how pain signals get from injury to the brain.
Along the way it can be modulated as a result we
have varying degrees of pain threshold and
Figure 5. Wong-Baker FACES Pain Scale reactions.
Nociception is different from pain because it how
OTHERS you interpret or perceive the pain stimuli and not the
Functional activity score (FAS) pain itself
• This is an activity-related score.
• Ask your patient to perform an activity related to their
painful area (for example, deep breath and cough for
thoracic injury or move affected leg for lower limb
pain).
• Observe your patient during the chosen activity and
score A, B, or C.
• Relative to baseline refers to any restriction above any
preexisting condition the patient may already have.
• “If you don’t measure it, you won’t improve it.”

Table 4. Functional Activity Score.


No limitation meaning the patient’s activity is
A
unrestricted by pain
Mild limitation means the patient’s activity is mild
B
to moderately restricted by pain Figure 7. Nociception and Pain
Severe limitation means the patient ability to
C
perform the activity is severely limited by pain A. PAIN INFLUENCES
• Cultural background – Asians have higher threshold for
Memorial Pain Assessment Card pain
• Memorial Pain Assessment Card (Fishman 1987) – a • Caucasians demand narcotics more often than a
two-sided card that measures pain intensity and the Filipino patient would.
patient’s mood on one side, and has a modified version • Anticipation of pain
of the Tursky Pain Description Scale on the other. • Previous experience of pain – female previously had
For chronic pain vaginal delivery has a higher threshold.
• Context in which pain occurs - dependent on the
psychological make-up of the patient.
• Emotional and Cognitive responses
• Pain is subjective. What is tolerable to me might not be
tolerable to you.
• Belief to please the gods
• State of the patient - is he anxious, sad, angry
• Psychological Issues
• Other Illnesses like cancer
• Personality - strong, weak
• Social Factors – family up-brining, work, peer pressure
Figure 6. Memorial Pain Assessment Card
IV. PAIN PHYSIOLOGICAL PATHWAY
III. NOCICEPTION AND PAIN • FOUR STEPS:
• Nociception Transduction
How pain signals get from the site of injury to the Transmission
brain Perception
• Pain Modulation
How we percieve or feel pain
• Nociception is not the same as pain

CCetC
Block XX: Pain Management 5 of 14
MD 3
TRANSDUCTION - PERIPHERY You have your fast pain into the somatosensory
• Tissue Injury system and the slow pain to the thalamic system.
• Release of Chemicals (inflammatory soup)
• Stimulation of Pain Receptors (Nociceptors) A. CHEMICAL MEDIATORS OF PAIN
• Signal Travels in A-delta(fast) or C(slow) nerve fibers to • “AKA – Pain/Inflammatory Soup”
the spinal cord
INTRINSIC TO THE NERVOUS SYSTEM
TRANSMISSION - SPINAL CORD • These are released at the site of injury.
• Dorsal horn is the first relay station Substance P
• A-delta and C nerve synapses (connects with second ATP
order nerve) Glutamate
• Second order neuron travels up to the brain Serotonin
• Initial pain modulation can take place in the spinal cord Norepinephrine
(spinothalamic tract reaching the thalamus) Glycine
Aspartate
PERCEPTION - BRAIN
• Thalamus is the second relay station EXTRINSIC TO THE NERVOUS SYSTEM
• Several parts of the brain are interconnected: Leukotriene B4
Frontal cortex – behavior related to pain Interleukin 1-alpha, 1-beta
somatosensory cortex – localization of pain Serotonin
limbic system – emotion on pain Histamine – from mast cells and platelets
brainstem – changes in vital signs Tumor necrosis factor – cases of cancer
• pain perception occurs in the brain Bradykinin
reflex withdrawal from stimuli when pain is percieved Hydrogen ions
Basolateral amygdala is responsible for the reflex Prostaglandins (including PGE2)
action
EXTRINSIC SOURCE
MODULATION • Neutrophils
• Descending pathway from brain to dorsal horn • Leukocytes
• Usually inhibits pain signals from the periphery • Mast cell, activated platelets
A-delta fibers – myelinated, has nodes of ranvier, • Mast cells
and exhibits salutatory conduction, mostly carries • Macrophages
acute and sharp pain. • Plasma
C fibers – nonmyelinated, slower transmission, • Infection
mostly functions in chronic pain.
B. NEUROPATHIC PAIN MECHANISMS
• Pathological Pain
• Abnormality of nociceptive pathway
peripheral nerves
the spinal cord or brain
• Need different pharmacological management
• Abnormal nerve tissue (eg. Amputation neuroma)
• Abnormal firing of pain nerves
• Changes in chemical signaling, nerve connections, and
inhibitory functions in the dorsal horn also influence the
its mechanism
• Associated with nerve injury, compression, or invasion
• Loss of normal inhibitory function
There is hyperexcitability of the peripheral nerves
Figure 8. The Pain Pathway and medication used in each step. and central neurons, loss of inhibitory control
leading to abnormal discharges of the nerve
• You have an injury at the periphery, signals are carried resulting to neuropathic pain
by the peripheral nerve to the dorsal root ganglion and • Resistant to opioids
then to the higher centers: limbic system, • Best treated with anti-convulsants, anti-depressants
somatosensory center and frontal cortex. What is with and epidural steroids
the frontal cortex? It is for behavior towards the pain.
CCetC
Block XX: Pain Management 6 of 14
MD 3
• Abnormalities of the nerve leading to neuropathic pain: • Ligaments
Allodynia – central pain sensitization following Other visceral organs
normally non painful, often repetitive, stimulation;
patient perceives a non-painful stimulus as painful VI. MECHANISM OF POSTOPERATIVE PAIN
Hyperesthesia –abnormal increase in the sensitivity • Peripheral sensitization
to stimuli of the sense, if the stimuli is pain Increased sensitivity to an afferent nerve stimulus
therefore it is felt as if abnormally increased • Constant bombardment of the CNS with noxious input
Dysthetic pain – unpleasant sensation, persistent; • Noxious stimuli processed by the CNS
cannot be describe by the patient; sometimes • Pathologic consequences of the pain
associated by allodynia • Sensitization of the CNS response/ Central
• Neuropathic pain is a disease: Sensitization
From surgical procedures/amputation Refers to a state of spinal neuron hyperexcitability
Exposure to drugs, alcohol, toxins • Induced sensitivity in the nervous system (outlasting
Traumatic nerve injury/compression the stimulus)
Viral infections
Vascular-related/neurodegenerative VII. POST-OPERATIVE PAIN MANAGEMENT
• 77% of patients believe pain is a necessary part of
Nutritional deficiencies
surgery
Chronic illness
• 8% of patients postpone the surgical procedure
Table 5. Nociceptive vs Neuropathic Pain because of concerns associated with pain
Nociceptive The nociceptors in tissues send pain • Pain after surgery is a compilation of several
Pain signal to the CNS unpleasant sensory, emotional, and mental
Neuropathic The damage is within the nerve and experiences, associated with autonomic, endocrine
Pain causes typical pain symptoms metabolic, physiological, and behavioral responses.

THE GOAL IN POST-OPERATIVE PAIN


MANAGEMENT:
V. MECHANISM OF PERIOPERATIVE PAIN
• Mobilize the person as early as possible, get them
eating and drinking as early as appropriate
• Ensure patient are able to cough and do deep
breathing
If patients are not given good post-operative
management, they have an increased risk of chest
infections, hypoxia, cardiac problems, pressure
sores, deep vein thrombosis, depression, anxiety,
anorexia, increased wound infection rates and
others.
• Effective post-operative analgesia reduces the
incidence of post-operative chronic pain, improves the
functioning of organs following surgery and shortens
the hospital stay

ACUTE POST-OPERATIVE PAIN


• Over 80% of patients who had undergone surgery
reported post-operative pain
• Less than half of post-operative patients receive
adequate pain relief
• ACUTE POST-OPERATIVE PAIN may lead to the
Figure 9. Mechanism of Perioperative Pain
development of POST-OPERATIVE PAIN
• Commonly injured tissues during surgery are the • The average pain score is 3-4 out of 10 during the first
following: 2 to 3 days after surgery
Skin • Average pain score can be high as 7-8 with coughing
Muscle or walking in the first 2-3 days
Bone • Temporarily related to injury and that resolves during
Tendon the appropriate healing period

CCetC
Block XX: Pain Management 7 of 14
MD 3
• It often responds to treatment with analgesic • Preventive analgesia is when you give analgesics
medications and treatment of the precipitating cause throughout the operation. Its purpose is to minimize
neuronal sensitization of hypersensitivity extended into
ESSENTIAL STRATEGIC COMPONENTS OF ACUTE the post-operative period.
POST-OPERATIVE PAIN MANAGEMENT
• Multi-modal analgesia
• Procedure-specific analgesia
• Acute rehabilitation
• Opioid + Paracetamol, NSAIDs, Coxibs, A2δ ligands,
Ketamine, Nerve Block  POTENTIATION

ADVERSE PHYSIOLOGIC EFFECTS OF POST-


OPERATIVE PAIN
• Pulmonary System (Decreased Lung Volumes)
Atelectasis
Ventilation-to-perfusion mismatching
Arterial Hypoxemia VIII. THERAPEUTIC OPTIONS
Hypercarbia • Pharmacologic – use of medications
Pneumonia • Non-pharmacologic
• Cardiovascular System (sympathetic Nervous system Physical modalities (Rehab)
Stimulation Psychological modalities
Systemic HPN • Interventional
Tachycardia
Myocardial Ischemia A. PHARMACOLOGIC
Cardiac dysrhythmias • WHO Recommendation
• Immune System (decrease immune function) Right drug
• Coagulation System Right dose
Inc platelet adhesiveness Right diagnosis
Dec fibrinolysis • WHO Guidelines for Pain Relief
Hypercoagulation By the mouth
Deep vein thrombosis By the clock
• GI system ─ Post-op patients should be given round the
Ileus clock medication for the first 48-72 hours
• Genitourinary system By the ladder
Urine retention For the individual
With attention to detail
CONSEQUENCES OF UNRELIEVED PAIN • WHO Stepladder
• In the short term: Designed mainly for treatment of cancer pain i.e
Needless suffering progressive pain requiring increasing medication
Risk of increased post-operative morbidity and This stepwise approach does not work well for
mortality acute severe pain requiring immediate strong
Increases hospital stay and costs of care opioids e.g. morphine
• In the long term: Also does not work well for chronic non-cancer
Chronic pain in the 10-50% of patients pain or neuropathic pain
Severe chronic pain in 2-10% of patients • WHO's cancer pain ladder for adults
Chronic post-operative pain WHO has developed a three-step "ladder" for
cancer pain relief in adults.
PRE-EMPTIVE VS. PREVENTIVE ANALGESIA • If pain occurs, there should be prompt oral
• Preemptive analgesia is based on the hypothesis that administration of drugs in the following order:
treatment that starts before surgical incision would nonopioids (aspirin and paracetamol); then, as
prevent the establishment of central sensitization and necessary, mild opioids (codeine); then strong
reduces subsequent pain experienced by the patient opioids such as morphine, until the patient is free
with the assumption that central sensitization has a of pain. To calm fears and anxiety, additional drugs
major contribution to post-operative pain. – “adjuvants” – should be used. To maintain
freedom from pain, drugs should be given “by the

CCetC
Block XX: Pain Management 8 of 14
MD 3
clock”, that is every 3-6 hours, rather than “on • Rectal
demand” This three-step approach of administering • Transdermal
the right drug in the right dose at the right time is • Nasal
inexpensive and 80-90% effective. Surgical • Intravenous – example ketamine, and IV is indicated
intervention on appropriate nerves may provide for severe pain
further pain relief if drugs are not wholly effective. Intermittent
In the case of cancer pain in children, WHO Continuous
recommends a two step ladder. Patient controlled – infused in your IVF
• Subcutaneous
• Intramuscular – very painful
• PTCA- patient controlled analgesia
• Neuraxial – which can either be epidural or intrathecal
• Peripheral nerve block
• Intrapleural regional analgesia
• Transcutaneous Electrical Nerve Stimulation

TREATMENTS
• Treatments – Periphery
Non-pharmacologic
─ Rest, ice, compression, elevation
Anti-inflammatory agents
Local anesthetics
• Treatments – Brain
Non-pharmacologic treatment:
Figure 10. WHO Pain Relief Ladder ─ Psychological
Pharmacological treatments:
• Treatment of Acute Nociceptive Pain – Reverse WHO
─ Paracetamol
Ladder
─ Opioids
• Mainly useful for severe acute nociceptive pain
─ Amitriptylin
Trauma pain
─ Clonidine
Post-operative pain
• Start at the top and ‘step down the ladder’ as the pain
B. DRUG CLASSIFICATION
improves
• Simple analgesics
Paracetamol (Acetaminophen)
Anti-inflammatory medicines
• Aspirin, Ibuprofen, Diclofenac
• Opioids
Mild – Codeine, Tramadol
Strong – Morphine, Pethidine, Fentanyl,
Oxycodone
• Other analgesics:
Tricyclic antidepressants, e.g. amitriptyline
Anticonvulsants e.g. carbamazepine, gabapentin
Local anaesthetics e.g. Lidocaine, Bupivacaine
• Others e.g. ketamine, clonidine
Figure 11. New Adaptation of the Analgesic Ladder. This model has
been proposed and applied in the treatment of acute pain in
emergency departments and post-operative situations. TYPES OF ANALGESICS
Table 4. Analgesics
Simple Analgesics
ROUTE OF ADMINISTRATION
• Oral—preferred route and is chronic pain Paracetamol / Change prostaglandin levels in
Most common complication of chronic opioid use: Acetaminophen the brain
Constipation Mainly work by changing
Acute complication: Pruritus prostaglandin levels in the
NSAIDs
If opioid reversal: Abdominal pain periphery thereby reducing
• Transmucosal like fentanyl inflammation

CCetC
Block XX: Pain Management 9 of 14
MD 3
Opioid Analgesics
Acts on opioid receptors in the
Codeine
brain and spinal cord
Morphine, Acts on opioid receptors in the
Pethidine, brain and spinal cord
Fentanyl
Others
Increases descending inhibitory
Amitriptyline
signals in the spinal cord
“Membrane stabilizers”,
Anticonvulsants probably work by reducing
abnormal firing of pain nerves
Temporarily block signaling in
Local pain nerves in periphery (e.g.
Anesthetics infiltration or nerve block) or
spinal cord (e.g. spinal block)
Acts weakly on opioid receptors,
also increases descending
Tramadol
inhibitory signals in the spinal
cord
Blocks NMDA receptors in the
Ketamine brain and spinal cord
(especially in dorsal horn)
Increases descending inhibitory
Clonidine
signals in the spinal cord Figure 13. Opioid ladder

Opioids Opioid Therapy (Administration Methods)


• Classification: • Oral – preferred route
Full agonists • Rectal
─ Morphine, Codeine, Fentanyl, Tramadol, • Transdermal
Meperedine, Methadone • Nasal
Partial agonists • Intravenous
─ Buprenorphine • Subcutaneous
Mixed agonist-antagonists • Intramuscular
─ Nalbuphine, Butorphanol, Pentazocine
• Caution: Know the MOA of each drug to avoid giving Monitoring Opioid Therapy
drugs that antagonize each other; don’t give two • Critical Outcomes – 4 A’s
different opioids at the same time Analgesia
• Stepladder vs. Elevator Elevator: Use opioid right Adverse effects
away, low dose. But consider patient condition as well. Activities of daily living
• Most common mistake in the use opioids in the wards Aberrant opioid-related behavior
is the mixing of these medications. For example, if your
patient is already on tramadol (which is a full agonist) Opioid Therapy (Management of Side Effects)
and you also gave nalbuphine (mixed agonist- • Constipation
antagonist) PRN, nalbuphine will antagonize the action • Sedation
of tramadol. • Nausea and Vomiting
• Do not mix full agonists with mixed agonist-antagonists! • Respiratory depression
Opioids don’t have ceiling dose/ maximum dose. But • Others:
you have to limit if adverse effects are noticed (eg. Hallucination
Cardiorespiratory depression) Pruritus
Urinary retention
Sexual dysfunction
• The most common complaint of a patient on reversal
therapy is abdominal pain.

CCetC
Block XX: Pain Management 10 of 14
MD 3
ALTERNATIVE ALGORITHM (ANALGESIC D. NON-PHARMACOLOGIC TREATMENT
ELEVATOR) • Physical – rest, ice, compression, elevation,
• Mild pain • Psychological – explanation, reassurance, counseling
Initiate non-opioid treatment if inadequate control • Current recommendation of American Pain Society:
→ low dose “strong” opioids give anticonvulsants and other medications like opioids
• Moderate pain for treatment of postoperative pain, meaning approach
Start low dose “strong” opioids with or without non- is multimodal.
opioids
• Severe pain E. COMPLEMENTARY/ALTERNATIVE THERAPIES
Immediate use of “strong” opioids with or without • Hypnosis, psychotherapy, relaxation, biofeedback,
non-opioids thermotherapy, cryotherapy, TENS, acupuncture

ADJUVANT DRUG THERAPIES IX. BENEFITS OF TREATING PAIN


• Antidepressants • For the patient
• Corticosteroids Physical
• Caffeine ─ Better sleep, improved appetite
• Clonidine ─ Fewer medical complications (e.g. cardiac
• Anticonvulsants complications, pneumonia)
• Radiopharmaceuticals – Strontium-89 Psychological
• Antispasmodics ─ Reduced suffering
• Calcitonin ─ Less depression, anxiety
• Benzodiazepines • For the family
• Local Anesthetics Improved function as a member of the family (e.g.
• Psychostimulants as a father/mother)
• Muscle Relaxants Able to keep working
• For society
C. MULTIMODAL ANALGESIA Reduced health costs (e.g. shorter hospital stay)
Able to contribute to the community

X. SUMMARY
• ANESTHESIA
Pneumonic: YAMPLE
─ Y – Why? – Indication
─ Anesthetic Experience
─ Medications
─ Past History
─ Last Meal
─ Examinations (ASA, MC)
• GOING ABOUT PAIN HISTORY
Pneumonic: OLDCARTS
─ Onset—(temporal pattern) of pain
─ Location—site of pain
BENEFITS OF MULTI-MODAL ANALGESIA ─ Duration—constant or incidental
• Decreased dose of each analgesic ─ Character—quality of pain
• Improved anti-nociception due to additive or synergistic ─ Associated signs and symptoms
effect ─ Relieving and aggravation factors
• Decreased severity of side effects of each drug ─ Treatment
─ Score—intensity (you get the worst pain, the
usual pain, the least pain and the pain at the
time of your examination)
Pneumonic: SOCRATES
─ Site—location of pain
─ Onset—when did it start? How long?
─ Character—description of the pain
─ Radiation—where does the pain go?
Figure 14. Why should the treatment be multimodal?

CCetC
Block XX: Pain Management 11 of 14
MD 3
─ Associated signs and symptoms—impact on c. Present analgesia
quality of life d. Preventive Analgesia
─ Time course—temporal pattern
4. In WHO’s Recommendation, which is not included?
─ Exacerbating and relieving factors
─ Severity—intensity/ pain score A. Rite Med
• Pain Diagnosis B. Right Drug
Duration of pain C. Right Dose
Cause D. Right Diagnosis
Mechanism
─ ex. non-cancer pain, mixed nociceptive and 5. What would you give for the relief of mild to
moderately severe pain, where the use of an opioid
neuropathic
analgesic is appropriate, can be given by itself or
• Characteristics of pain combined in a syrup with other drugs and is used as a
Location cough suppressant in adults aged 18 and above?
Severity – at rest and movement
Quality A. Fentanyl B. Codeine
Duration C. Morphine D. Pethidine
• Classification
Duration: Acute or Chronic Answers: CCBAB
Cause: Cancer/Non-cancer REFERENCES
Mechanism: Nociceptive/Neuropathic/Mixed • Upclass notes
• Pre-existing Factors • Doctor’s lecture
Physical
Psychological
Socio-cultural
Spiritual
• Treatment
Non-drug treatments
Drug treatments – Nociceptive
Drug treatments – Neuropathic

REVIEW QUESTIONS

1. Indication for Analgesia and Fever would work


peripherally to block pain impulse generation; may also
inhibit prostaglandin synthesis in CNS. A good example
of this would be acetaminophen, what will be the
recommended dosage for Angel Locsin, a 34 year old
stunner of Bulacan to have a regular strength immediate
release?
a. 100-150mg PO/PR q4hr PRN
b. 525-750 mg PO/PR q4hr PRN
C. 325-650 mg PO/PR q4hr PRN
d. Ang bato ni Darna ang Solusyon

2. Wisdom is nothing but healed pain, which of the


following though is an example of a pain that is
described to be “shooting” with direct damage to nerve?
a. Superficial Burn
b. Cancer Pain
c. Phantom Pain
d. Tooth di Ache

3. Treatment starts before surgical incision would


prevent the establishment of central sensitization and
reduces subsequent pain experienced by the patient
a.Periemptive
b. Preemptive analgesia

CCetC
Block XX: Pain Management 12 of 14
MD 3
APPENDICES

Localization Characteristics Fibers Involved Examples Treatment


Somatic pain Localized Pinprick, A-delta fibers Superficial IM injections,
stabbing, sharp activity located in lacerations, venous access,
the periphery superficial burns, otitis media,
stomatitis and
extensive
abrasions
Cold packs,
tactile
stimulation,
analgesics,
NSAIDs,
opioids
Visceral Pain Generalized Ache, pressure, C-fibers involving Colic, muscle Opioids,
sharp deeper spasms; NSAIDs;
innervations periosteum, intraspinal
muscle and joint and local
injuries; sickle cell injections;
crisis, paracetamol
appendicitis and
kidney disease

CCetC
Block XX: Pain Management 13 of 14
MD 3
CCetC
Block XX: Pain Management 14 of 14
MD 3

You might also like